Inter 1st Year Maths 1B Applications of Derivatives Important Questions

Inter 1st Year Maths 1B Applications of Derivatives Important Questions

Students get through Maths 1B Important Questions Inter 1st Year Maths 1B Applications of Derivatives Important Questions which are most likely to be asked in the exam.

Intermediate 1st Year Maths 1B Applications of Derivatives Important Questions

Question 1.
Find dy and ∆y of y = f(x) = x2 + x at x = 10 when ∆x = 0.1.
Solution:
As change in y – f(x) is given by ∆y = f(x + ∆x) – f(x), this change at x = 10 with ∆x = 0.1 is
∆y = f(10.1) – f(10)
= {(10.1)2 + 10.1} – {102 + 10}
= 2.11.
Since dy = f'(x) ∆x, dy at x = 10 with ∆x = 0.1 is dy = {(2)(10) + 1} 0.1 = 2.1
(since \(\frac{\mathrm{d} y}{\mathrm{dx}}\) = 2x + 1).

Inter 1st Year Maths 1B Applications of Derivatives Important Questions

Question 2.
Find ∆y and dy for the function y = cos (x) at x = 60° with ∆x = 1° .
Solution:
For the given problem ∆y and dy at x = 60° with ∆x = 1° are
∆y = cos (60° + 1°) – cos (60°) …………. (1)
and dy = -sin(60°) (1°) ………….. (2)
Cos (60°)= 0.5,
Cos (61°)= 0.4848,
Sin (60°) = 0.8660,
1° = 0.0174 radians
∴ ∆y = -0.0152 and
dy = -0.0150.

Question 3.
The side of a square is increased from 3 cm to 3.01 cm. Find the approximate increase in the area of the square.
Solution:
Let x be the side of a square and A be its area.
Then A = x2. …… (1)
Clearly A is a function of x. As the side is increased from 3 cm to 3.01 cm we can take x – 3 and ∆x = 0.01 to compute the approximate increase in the area of square. The approximate value of change in area is
∆A ≈ \(\frac{\mathrm{dA}}{\mathrm{dx}}\) ∆x
In view of equation (1), the equation (2) becomes
∆A ≈ 2x∆x
Hence the approximate increase in the area when the side is increased from 3 to 3.01 is
∆A ≈ 2(3)(0.01) = 0.06

Question 4.
If the radius of a sphere is increased from 7 cm to 7.02 cm then find the approximate increase in the volume of the sphere.
Solution:
Let r be the radius of a sphere and V be its volume. Then
V = \(\frac{4 \pi \pi^{2}}{3}\) …………….. (1)
Here V is a function of r. As the radius is increased from 7 cm to 7.02, we can take r = 7 cm and ∆r = 0.02 cm. Now we have to find the approximate increase in the volume of the sphere.
∴ ∆V ≈ \(\frac{\mathrm{dV}}{\mathrm{dr}}\) ∆r = 4πr2 ∆r.
Thus, the approximate increase in the volume of the sphere is \(\frac{4(22)(7)(7)(0.02)}{7}\) = 12.32 cm3.

Inter 1st Year Maths 1B Applications of Derivatives Important Questions

Question 5.
If y = f(x) = k xn then show that the approximate relative error (or increase) in y is n times the relative error (or increase) in x where n and k are constants.
Solution:
The approximate relative error (or increase) in y by the equation (2) of if a number A is very close to a number B but it is not equal to B then A is called an approximate value of B is (\(\frac{f^{\prime}(x)}{f(x)}\)) ∆x = \(\frac{k n x^{n-1}}{k x^{n}}\) ∆x = n(\(\frac{\Delta x}{x}\) = n)
Hence the approximate relative error in y = kxn is n times the relative error in x.

Question 6.
If the increase in the side of a square is 2% then find the approximate percentage of increase in its area.
Solution:
Let x be the side of a square and A be its area.
Then A = x2.
Approximate percentage error in area A
= (\(\frac{\frac{\mathrm{dA}}{\mathrm{dx}}}{\mathrm{A}}\)) × 100 × ∆x(by (3) of if a number
A is very close to a number B but it is not equal to B then A is called an approximate value of B with f = A)
= \(\frac{100(2 x) \Delta x}{x^{2}}\) = \(\frac{200 \Delta x}{x}\) = 2(2) = 4
(∵ \(\frac{\Delta x}{x}\) × 100 = 2

Question 7.
If an error of 0.01 cm is made in measuring the perimeter of a circle and the perimeter is measured as 44 cm then find the approximate error and relative error in its area.
Solution:
Let r, p and A be the radius, perimeter and area of the circle respectively. Given that p = 44 cm and ∆p = 0.01. We have to find approximation of ∆A and \(\frac{\Delta \mathrm{A}}{\mathrm{A}}\). Note that A = πr2 which is a function of r. As p and ∆p are given we have to transform A = πr2 into the form A = f(p). This can be achieved by using the relation, perimeter 2πr = p.
∴ A = π(\(\frac{p}{2 \pi}\))2 = \(\frac{p^{2}}{4 \pi}\)
Hence the approximate error in
A = \(\frac{d A}{d p}\)∆p = \(\frac{2 p}{4 \pi}\)∆p = \(\frac{P}{2 \pi}\)∆p
The approximate error in A when p = 44 and ∆p = 0.01 = \(\) (0.01) = 0.07
The approximate relative error
Inter 1st Year Maths 1B Applications of Derivatives Important Questions 1
= 0.0004545.

Inter 1st Year Maths 1B Applications of Derivatives Important Questions

Question 8.
Find the approximate value of \(\sqrt[3]{999}\)
Solution:
This problem can be answered by
f(x + ∆x) ≈ f(x) + f'(x) ∆x …………. (1)
with x = 1000 and ∆x = -1. The reason for taking x = 1000 is to make the calculation of f(x) simpler when f(x) = \(\sqrt[3]{x}\), Suppose
y = f(x) = \(\sqrt[3]{x}\)
The equation (1) becomes
f(x + ∆x) ≈ f(x) = \(\frac{1}{3 x^{\frac{2}{3}}}\) ∆x
Hence f(1000 – 1)
≈ f(1000) + \(\frac{1}{3(1000)^{2 / 3}}\) (-1) = 9.9967.

Question 9.
Find the slope of the tangent to the following curves at the points as indicated.
i) y = 5x2 at (-1, 5)
ii) y = \(\frac{1}{x – 1}\) (x ≠ 1) at [3, \(\frac{1}{2}\)]
iii) x = a secθ, y = a tanθ at θ = \(\frac{\pi}{6}\)
iv) (\(\frac{x}{a}\))n + (\(\frac{x}{b}\))n = 2 at (a, b)
Solution:
i) y = 5x2, then \(\frac{\mathrm{dy}}{\mathrm{dx}}\) = 10x
Slope of the tangent at the given points
(\(\frac{\mathrm{dy}}{\mathrm{dx}}\))(-1.5) -10(-1) = -10

ii) y = \(\frac{1}{x – 1}\)
\(\frac{\mathrm{dy}}{\mathrm{dx}}\) = \(\frac{-1}{(x-1)^{2}}\)
Slope of the tangent at the given point is
Inter 1st Year Maths 1B Applications of Derivatives Important Questions 2
Inter 1st Year Maths 1B Applications of Derivatives Important Questions 3

Inter 1st Year Maths 1B Applications of Derivatives Important Questions

Question 10.
Find the equations of the tangent and the normal to the curve y = 5x4 at the point (1, 5)
Solution:
y = 5x4 implies that \(\frac{\mathrm{dy}}{\mathrm{dx}}\) = 20x3
Slope of the tangent to the curve at (1, 5) is
(\(\frac{\mathrm{dy}}{\mathrm{dx}}\))(1, 5) = 20(1)5 = 20
Equation of the normal of (1, 5) is
y – 5 = 20 (x – 1) = 20x – 20
y = 20x – 15
Slope of the normal to the curve at (1, 5) is
–\(\frac{1}{m}\) = –\(\frac{1}{20}\)
Equation of the normal of (1, 5) is
y – 5 = –\(\frac{1}{20}\) (x – 1)
20 y – 100 = -x + 1
x + 20y = 101
(or) 20y = 101 – x

Question 11.
Find the equations of the tangent and the normal to the curve y4 = ax3. at (a, a).
Solution:
Given curve is y4 = ax3
Differentiating w.r.to. x
4y3 . \(\frac{\mathrm{dy}}{\mathrm{dx}}\) = 3ax2
\(\frac{\mathrm{dy}}{\mathrm{dx}}\) = \(\frac{3 a x^{2}}{4 y^{3}}\)
Slope of the tangent at (a, a) = \(\frac{3 a \cdot a^{2}}{4 a^{3}}\) = \(\frac{3}{4}\)
Slope of the normal at (a, a) = –\(\frac{1}{m}\) = –\(\frac{4}{3}\)
Equation of the tangent at (a, a) is
y – a = \(\frac{3}{4}\)(x – a)
4y – 4a = 3x – 3a
4y = 3x + a
Equation of the normal at (a, a) is 4
y – a = – \(\frac{4}{3}\)(x – a)
3y – 3a = -4x + 4a
3y + 4x = 7a

Question 12.
Find the equations of the tangent to the curve y = 3x2 – x3, where it meets the X-axis.
Solution:
Equation of the curve is y = 3x2 – x3
Equation of X – axis is y = 0
For points is intersection of the curve and X-axis
3x2 – x3 = 0 ⇒ x2 (3 – x) = 0
x = 0, x = 3
The curve crosses X-axis 0(0, 0) and A(3, 0)
y = -3x2 – x3
\(\frac{\mathrm{dy}}{\mathrm{dx}}\) = 6x – 3x2
At O(0, 0), slope of the tangent = (\(\frac{\mathrm{dy}}{\mathrm{dx}}\))(0, 0) = 0
Equation of the tangent at (0, 0) is y – 0, 0(x – 0) i.e., y = 0
i.e., x-axis is the tangent to the curve at O(0, 0)
At A(3, 0), slope of the tangent = (\(\frac{\mathrm{dy}}{\mathrm{dx}}\))(3, 0)
= 6.3 – 3.32
= 18 – 27
= -9
Equation of the tangent at A(3, 0) is
y – 0 = -9(x – 3) = -9x + 27
(or) 9x + y = 27

Inter 1st Year Maths 1B Applications of Derivatives Important Questions

Question 13.
Find the points at which the curve t y = sin x has horizontal tangents.
Solution:
y = sin x
Inter 1st Year Maths 1B Applications of Derivatives Important Questions 4
A tangent is horizontal if and anal its slope is
cos x = 0 ⇒ x = (2n + 1)\(\frac{\pi}{2}\), n ∈ Z
Hence the given curve has horizontal tangents at points (x0, y0)
⇔ x0 = (2n + 1) . \(\frac{\pi}{2}\) and
y0 = (-1)n for same n ∈ Z

Question 14.
Verify whether the curve y = f(x) = x1/3 has a vertical tangent at the point with x = 0.
Solution:
Inter 1st Year Maths 1B Applications of Derivatives Important Questions 5
The function has a verified tangent at the point whose x co-ordinate is 0.

Question 15.
Find whether the curve y = f(x)= x2/3 has a vertical tangent at x = 0.
Solution:
Inter 1st Year Maths 1B Applications of Derivatives Important Questions 6
Thus left handed be normal \(\frac{1}{h^{1 / 3}}\) as h → 0 is -∞
While the right handed limit is ∞.
Hence Inter 1st Year Maths 1B Applications of Derivatives Important Questions 7 does not exist. The vertical tangent does not exist.
At the point x = 0.

Inter 1st Year Maths 1B Applications of Derivatives Important Questions

Question 16.
Show that the tangent at any point 0 on the curve x = c sec θ, y = c tan θ is y sin θ = x – c cos θ.
Solution:
x = c sec θ, y = c tan θ
Inter 1st Year Maths 1B Applications of Derivatives Important Questions 8
Inter 1st Year Maths 1B Applications of Derivatives Important Questions 9

Question 17.
Show that the area of the triable formed by the tangent at any point on the curve xy = c (c ≠ 0) with the coordinate axis is constant.
Solution:
Observe that c ≠ 0
If c = 0 the equation xy = 0 represent the coordinate circle which is against the definite.
Let P (x1, y1) be a point on the curve xy = c
y = \(\frac{c}{x}\) = 1, \(\frac{\mathrm{dy}}{\mathrm{dx}}\) = – \(\frac{c}{x^{2}}\)
Equation of the tangent at (x1, y1) is
y – y1 = – \(\frac{c}{x^{2}}\) (x – x1)
x2y – x12 = -cx + cx1
cx + x12 . y = x12 + cx1
= cx1 + cx1 (x1y1 = c)
= 2cx1
\(\frac{c x}{2 c x_{1}}\) + \(\frac{x_{1}^{2} y}{2 c x_{1}}\) = 1
\(\frac{x}{2 x_{1}}\) + \(\frac{y}{\left(\frac{2 c}{x_{1}}\right)}\) = 1
Area of the triangle formed with co-ordinate axes
= \(\frac{1}{2}\) |OA . OB|
= \(\frac{1}{2}\) (2x1) (\(\frac{2 c}{x_{1}}\)) = 2c = constant

Inter 1st Year Maths 1B Applications of Derivatives Important Questions

Question 18.
Show that the equation of the tangent to the curve (\(\frac{x}{a}\))n + (\(\frac{y}{b}\))n = 2 (a ≠ 0, b ≠ 0) at the point (a, b) is \(\frac{x}{a}\) + \(\frac{y}{b}\) = 2
Solution:
Inter 1st Year Maths 1B Applications of Derivatives Important Questions 10
Equation of the tangent to the curve at the point (a, b) is
y – b = \(\frac{-b}{a}\) (x – a)
\(\frac{y}{b}\) – 1 = – \(\frac{x}{a}\) + 1
\(\frac{x}{a}\) + \(\frac{y}{b}\) = 2

Question 19.
Show that the length of the sub normal at point on the curve y2 = 4ax is a constant.
Solution:
Equation of the curve is y2 = 4ax
Differentiating w.r.to x
2y \(\frac{\mathrm{dy}}{\mathrm{dx}}\) = 4a
\(\frac{\mathrm{dy}}{\mathrm{dx}}\) = \(\frac{4a}{2y}\) = \(\frac{2a}{y}\)
Length of the sub-normal \(\left|\frac{y d y}{d x}\right|=\left|y \cdot \frac{2 a}{y}\right|\)
= 2a = constant

Question 20.
Show that the length of the Sub tangent at any point on the curve y = ax (a > 0) is a constant.
Solution:
Equation of the curve is y = ax
Differentiating w.r.to x
\(\frac{\mathrm{dy}}{\mathrm{dx}}\) = ax log a = y. log a
Length of the sub-tangent
= \(\left|\frac{y}{\left(\frac{d y}{d x}\right)}\right|\) = \(\left|\frac{y}{\left(\frac{d y}{d x}\right)}\right|\) = \(\frac{1}{\log a}\) constant

Inter 1st Year Maths 1B Applications of Derivatives Important Questions

Question 21.
Show that the square of the length of subtangent at any point on the curve by2 = (x + a)3 (b ≠ 0) varies with the length of the subnormal at that point.
Solution:
Differentiating by2 = (x + a)3
w.r.t x, we get
2by y’ = 3(x + a)2
∴ The length of the subnormal at any point (x, y) on the curve
= |y y’| = |\(\frac{3}{2 b}\)(x + a)2| ………………… (1)
The square of the length of subtangent
Inter 1st Year Maths 1B Applications of Derivatives Important Questions 11
∴ (length of the subnormal)2 ∝ (length of subnormal).

Question 22.
Find the value of k, so that the length of the subnormal at any point on the curve y = a1 – kxk is a constant.
Solution:
Differentiating y = a1 – kxk with respect to x,
we get y’ = ka1 – kxk – 1
Length of subnormal at any point P(x, y) on the curve
= |y y’| = |yka1 – kxk – 1|
= |k a1 – kxka1 – kxk – 1|
= |ka2-2kx2k-1|
In order to make these values a constant, we should have 2k – 1 = 0 i.e., k = \(\frac{1}{2}\).

Question 23.
Find the angle between the curves xy = 2 and x2 + 4y = 0.
Solution:
First we find the points of intersection of xy = 2 and x2 + 4y = 0
y = \(\frac{-x^{2}}{4}\)
But xy = 2 ⇒ x(\(\frac{-x^{2}}{4}\)) = 2 ⇒ x3 = -8
x = -2
y = \(\frac{-x^{2}}{4}\) = – \(\frac{4}{4}\) = -1
Point of intersection is P(-2, -1)
Inter 1st Year Maths 1B Applications of Derivatives Important Questions 12
= \(\left|\frac{-\frac{3}{2}}{\frac{1}{2}}\right|\) = 3
Φ = tan-1(3)

Inter 1st Year Maths 1B Applications of Derivatives Important Questions

Question 24.
Find the angle between the curve 2y = e\(\frac{-x}{2}\) and Y-axis.
Solution:
Equation of Y-axis is x = 0.
The point of intersection of the curve
2y = e\(\frac{-x}{2}\) and x = 0 is P(0, \(\frac{1}{2}\))
Let ψ be the angle between the given curves
2y = e\(\frac{-x}{2}\) at P with X – axis is given by
tan ψ = \(\left.\frac{\mathrm{dy}}{\mathrm{dx}}\right|_{\left(0, \frac{1}{2}\right)}=\left.\frac{-1}{4} \mathrm{e}^{\frac{-x}{2}}\right|_{\left(0, \frac{1}{2}\right)}=\frac{-1}{4}\)
Further, if Φ is the angle between the Y – axis and 2y = e\(\frac{-x}{2}\), then we have
tan Φ = |tan (\(\frac{\pi}{2}\) – ψ)| = |cot ψ| = 4
∴ The angle between the curve and the Y-axis is tan-1 4.

Question 25.
Show that the condition of the orthogonality of the curves ax2 + by2 = 1 and a1x2 + b1y2 = 1 is \(\frac{1}{a}\) – \(\frac{1}{b}\) = \(\frac{1}{a_{1}}\) – \(\frac{1}{b_{1}}\).
Solution:
Let the curves ax2 + by2 = 1 and a1x2 + b1y2 = 1 intersect at p(x1, y1) so that
ax12 + by12 = 1 and a1x12 + b1y12 = 1, from which we get,
\(\frac{x_{1}^{2}}{b_{1}-b}\) = \(\frac{y_{1}^{2}}{a_{1}-a}\) = \(\frac{1}{a b_{1}-a_{1} b}\) …………. (1)
Differentiating ax2 + by2 = 1 with respect to x,
we get \(\frac{\mathrm{dy}}{\mathrm{dx}}\) = \(\frac{-a x}{b y}\)
Hence, if mt is the slope of the tangent at P(x1, y1) to the curve
ax2 + by2 = 1, m1 = \(\frac{-a x_{1}}{b y_{1}}\)
Similarly, the slope (m2) of the tangent at P to
a1x2 + b1y2 = 1 is given by m2 = \(\frac{-a_{1} x_{1}}{b_{1} y_{1}}\)
Since the curves cut orthogonally we have m1m2 = -1,
i.e., \(\frac{\mathrm{a} a_{1} x_{1}^{2}}{\mathrm{~b} \mathrm{~b}_{1} y_{1}^{2}}\) = -1 or \(\frac{x_{1}^{2}}{y_{1}^{2}}=\frac{-b_{1}}{a a_{1}}\) ………………….. (2)
Now from (1) and (2), the condition for the orthogonality of the given curves is
\(\frac{b_{1}-b}{a-a_{1}}\) = \(\frac{b b_{1}}{a a_{1}}\)
or (b – a) a1b1 = (b1 – a1) ab
or \(\frac{1}{a}\) – \(\frac{1}{b}\) = \(\frac{1}{a_{1}}\) – \(\frac{1}{b_{1}}\)

Question 26.
Show that the curves y2 = 4(x + 1)and y2 = 36 (9 – x) intersect orthogonally. [Mar 11, May 06, 05]
Solution:
Solving y2 = 4(x + 1) and y2 = 36 (9 – x) for the points of intersection, we get
4(x + 1) = 36 (9 – x) 10x = 80 or x = 8
i.e., y2 = 4(x + 1) ⇒ y2 = 4(9) = 36 ⇒ y = ± 6
The points of intersection of the two curves are P(8, 6), Q(8, -6)
y2 = 4(x + 1) ⇒ \(\frac{\mathrm{dy}}{\mathrm{dx}}\) = \(\frac{2}{y}\)
y2 = 36 (9 – x) ⇒ \(\frac{\mathrm{dy}}{\mathrm{dx}}\) = \(\frac{-18}{y}\)
Slope of the tangent to the curve
y2 = 4(x + 1) at P is
m1 = \(\frac{2}{6}\) = \(\frac{1}{3}\)
Slope of the tangent to the curve
y2 = 36 (9 – x) at P is
m2 = \(\frac{-18}{6}\) = -3
m1m2 = \(\frac{1}{3}\) × -3 = -1
⇒ the curves intersect orthogonally at P.
We can prove, similarly,that the curves intersect orthogonally at Q also.

Inter 1st Year Maths 1B Applications of Derivatives Important Questions

Question 27.
Find the average rate of change of s = f (t) = 2t2 + 3 between t = 2 and t = 4.
Solution:
The average rate of change of s between t = 2 and t = 4 is \(\frac{f(4)-f(2)}{4-2}\) = \(\frac{35-11}{4-2}\) = 12.

Question 28.
Find the rate of change of area of a circle w.r.t. radius when r = 5 cm.
Solution:
Let A be the area of the circle with radius r.
Then A = πr2. Now, the rate of change of area A w.r.t. r is given by \(\frac{\mathrm{dA}}{\mathrm{dr}}\) = 2πr. When r = 5 cm.
\(\frac{\mathrm{dA}}{\mathrm{dr}}\) = 10 π.
Thus, the area of the circle is changing at the rate of 10 π cm2/cm.

Question 29.
The volume of a cube is increasing at a rate of 9 cubic centimeters per second. How fast is the surface area increasing when the length of the edge is 10 centimeters ?
Solution:
Let x be the length of the edge of the cube, V be its volume and S be its surface area. Then, V = x3 and S = 6x2. Given that rate of change of volume is 9 cm3/sec.
Therefore, \(\frac{\mathrm{dV}}{\mathrm{dt}}\) = 9 cm3/sec.
Now differentiating V w.r.t. t, we get,
Inter 1st Year Maths 1B Applications of Derivatives Important Questions 13

Inter 1st Year Maths 1B Applications of Derivatives Important Questions

Question 30.
A particle is moving in a straight line so that after t seconds its distance is s (in cms) from a fixed point on the line is given by s = f(t) = 8t + t3. Find (i) the velocity at time t = 2 sec (ii) the initial velocity (iii) acceleration at t = 2 sec.
Solution:
The distance s and time t are connected by the relation.
s = f(t) = 8t + t3 …………. (1)
∴ velocity (ν) = 8 + 3t2 ……………. (2)
and the acceleration is given by
a = \(\frac{\mathrm{d}^{2} \mathrm{~s}}{\mathrm{dt}^{2}}\) = 6t
i) The velocity at
t = 2 is 8 + 3 (4) = 20cm/sec.
ii) The initial velocity (t = 0) is 8 cm/sec.
iii) The acceleration at t = 2 is 6(2) = 12 cm/sec2

Question 31.
A container in the shape of an inverted cone has height 12 cm and radius 6 cm at the top. If it is filled with water at the rate of 12 cm3/sec., what is the rate of change in the height of water level when the tank is filled 8 cm ?
Solution:
Let OC be height to water level at t sec. The triangles OAB ad OCD are similar triangles.
Inter 1st Year Maths 1B Applications of Derivatives Important Questions 14
Let OC = h and CD = r. Given that AB = 6 cm, OA = 12 cm.
\(\frac{r}{6}\) = \(\frac{h}{12}\)
Inter 1st Year Maths 1B Applications of Derivatives Important Questions 15
Hence, the rate of change of water level is \(\frac{3}{4 \pi}\) cm/sec when the water level of the tank is 8 cm.

Question 32.
A particle is moving along a line according to s = f (t) = 4t3 – 3t2 + 5t -1 where s is measured in meters and t is measured in seconds. Find the velocity and acceleration at time t. At what time ‘ the acceleration is zero.
Solution:
Since f(t) = 4t3 – 3t2 + 5t – 1, the velocity at time t is
v = \(\frac{\mathrm{ds}}{\mathrm{dt}}\) = 12t2 – 6t + 5
and the acceleration at time t is a = \(\frac{\mathrm{d}^{2} \mathrm{~s}}{\mathrm{dt}^{2}}\) = 24t – 6.
The acceleration is 0 if 24t – 6 = 0
i.e., t = \(\frac{1}{4}\)
The acceleration of the particle is zero at
t = \(\frac{1}{4}\) sec.

Inter 1st Year Maths 1B Applications of Derivatives Important Questions

Question 33.
The quantity (in mg) of a drug in the blood at time t (sec) is given by q = 3(0.4)t. Find the instantaneous rate of change at t = 2 sec.
Solution:
Given that q = 3(0.4)t
∴ \(\frac{\mathrm{dQ}}{\mathrm{dx}}\) = 3(0.4)t loge(0.4) is the instaneous rate of change in q. Hence the instaneous rate of q at time t = 2 sec. is given by
\(\left(\frac{\mathrm{dQ}}{\mathrm{dx}}\right)_{t=2}\) = 3(0.4)2 loge (0.4).

Question 34.
Let a kind of bacteria grow by t3 (t in sec). At what time the rate of growth of the bacteria is 300 bacteria per sec ?
Solution:
Let g be the amount of growth of bacteria at t sec. Then
g(t) = t3 ………… (1)
The growth rate at time t is given by
g'(t) = 3t2
300 = 3t2 (given that growth rate is 300)
t = 10 sec.
∴ After t = 10 sec, the growth rate of bacteria should be 300 bacteria/sec.

Question 35.
The total cost C(x) in rupees associated with production of x units of an item is given by C(x) = 0.005 x3 – 0.02x2 + 30x + 500. Find the marginal cost when 3 units are produced (marginal cost is the rate of change of total cost).
Solution:
Let M represent the marginal cost. Then
M = \(\frac{\mathrm{dC}}{\mathrm{dx}}\)
Hence,
M = \(\frac{\mathrm{d}}{\mathrm{dx}}\)(0.005x3 – 0.02x2 + 30x + 500) dx
= 0.005(3x2) – 0.02(2x) + 30
∴ The Marginal cost at x = 3 is
(M)x = 3 = 0.005 (27) – 0.02 (6) + 30 = 30.015.
Hence the required marginal cost is Rs. 30.02 to produce 3 units.

Inter 1st Year Maths 1B Applications of Derivatives Important Questions

Question 36.
The total revenue in rupees received from the sale of x units of a product is given by R(x) = 3x2 + 36x + 5. Find the marginal revenue when x = 5 (marginal revenue is the rate of change of total revenue).
Solution:
Let m denote the marginal revenue. Then
m = \(\frac{\mathrm{dR}}{\mathrm{dx}}\) (since the total revenue is R(x))
Given that R(x) = 3x2 + 36x + 5
∴ m = 6x + 36
The marginal revenue at x = 5 is
[m = \(\frac{\mathrm{dR}}{\mathrm{dx}}\)]x = 5 = 30 + 36 = 66
Hence the required marginal revenue is Rs. 66.

Question 37.
Verify Rolle’s theorem for the function y = f(x) = x2 + 4 in [-3, 3].
Solution:
Here f(x) = x2 = 4. f is continuous on [-3, 3] as x2 + 4 is a polynomial which is continuous on any closed interval. Further f(3) = f(-3) = 13 and f is differentiable on [-3, 3].
∴ By Rolle’s theorem ∃ c ∈ (-3, 3) such that f'(c) = 0
The point c = 0 ∈ (-3, 3). Thus Rolle’s theorem is verified.

Question 38.
Verify Rolle’s theorem for the function f(x) = x(x + 3)e-x/2 in [-3, 0].
Solution:
Here f(-3) = 0 and f(0) = 0.
We have
f'(x) = \(\frac{\left(-x^{2}+x+6\right)}{2} e^{\frac{-x}{2}}\)
f'(x) = 0 ⇔ -x2 + x + 6 = 0 ⇔ x = -2 or 3. Of these two values -2 is in the open interval (-3, 0) which satisfies the conclusion of Rolle’s theorem.

Question 39.
Let f(x) = (x -1) (x – 2) (x – 3). Prove that there is more than one ‘c’ in (1, 3) such that f'(c) = 0. [Mar 13]
Solution:
Observe that f is continuous on (1, 3) differentiable in (1, 3) and f(1) = f(3) = 0.
f'(x) = (x – 1) (x – 2) + (x – 1) (x – 3) + (x – 2) (x – 3)
= 3x2 – 12x + 11
\(\frac{12 \pm \sqrt{144-132}}{6}\) = 2 ± \(\frac{1}{\sqrt{3}}\)
Both these roots lie in the open interval (1, 3) and are such that the derivative vanishes at these points.

Inter 1st Year Maths 1B Applications of Derivatives Important Questions

Question 40.
On the curve y = x2, find a point at which the tangent is parallel to the chord joining (0, 0) and (1, 1).
Solution:
The slope of the chord is \(\frac{1-0}{1-0}\) = 1.
The derivative is \(\frac{\mathrm{dy}}{\mathrm{dx}}\) = 2x.
We want x such that 2x = 1
i.e., x = \(\frac{1}{2}\)
we not that \(\frac{1}{2}\) is in the open interval (0, 1), as required in the Lagrange’s mean value theorem.
The corresponding point on the curve is (\(\frac{1}{2}\), \(\frac{1}{4}\)).

Question 41.
Show that f(x) = 8x + 2 is a strictly increasing function on R without using the graph of y = f(x).
Solution:
Let x1, x2 ∈ R with x1 < x2. Then 8x1 < 8x2. Adding 2 to both sides of this inequality, we have 8x1 + 2 < 8x2 + 2. i.e., f(x1) < f(x2).
Thus
x1 < x2 ⇒ f(x1) < f(x2) ∀ x1, x2 ∈ R.
Therefore, the given function f is strictly increasing on R.

Question 42.
Show that f (x) = ex is strictly increasing on R (without graph).
Solution:
Let x1, x2 ∈ R such that x1 < x2. we know that if a > b then ea > eb
∴ x1 < x2 ⇒ ex1 < ex2
i.e., f(x1) < f(x2).
Hence the given function f is a strictly increasing function.

Question 43.
Show that f(x) = – x + 2 is strictly decreasing on R.
Solution:
Let x1, x2 ∈ R x1 < x2.
Then x1 < x2
⇒ -x1 > -x2
⇒ -x1 + 2 > -x2 + 2
⇒ f(x1) > f(x2).
Therefore the given function f is strictly decreasing on R.

Inter 1st Year Maths 1B Applications of Derivatives Important Questions

Question 44.
Find the intervals on which
f(x) = x2 – 3x + 8 is increasing or decreasing ?
Solution:
Given fucntion is f(x) = x2 – 3x + 8.
Differentiating it w.r.t. x, we get f'(x) = 2x – 3
f(x) = 0 for x = 3/2.
Inter 1st Year Maths 1B Applications of Derivatives Important Questions 16
since f'(x) < 0 in (-∞, 3/2) the function f(x) is strictly decreasing on (-∞, \(\frac{3}{2}\)) Further since f'(x) > 0 in (\(\frac{3}{2}\), -∞), the function f(x) is a strictly increasing function (\(\frac{3}{2}\), -∞).

Question 45.
Show that f(x) = |x| is strictly decreasing on (-∞, 0) and strictly increasing on (0, ∞).
Solution:
The given function is f(x) = |x| i.e.,
Inter 1st Year Maths 1B Applications of Derivatives Important Questions 25
Thus f'(c) = 1 if c > 0, f'(c) = -1 if c < 0. Since f'(c) > 0 on (0, ∞), the function f(x) is strictly increasing on (0, ∞). Since f'(c) < 0 on (-∞, 0), the function f(x) is strictly decreasing on (-∞, 0).

Question 46.
Find the intervals on which the function f(x) = x3 + 5x2 – 8x + 1 is a strictly increasing function.
Solution:
Given that f(x) = x3 + 5x2 – 8x + 1.
∴ f'(x) = 3x2 + 10x – 8 = (3x – 2) (x + 4)
= 3(x – \(\frac{2}{3}\)) (x – (-4)).
f'(x) is negative in (-4, \(\frac{2}{3}\)) and positive in (-∞, -4) ∪ (\(\frac{2}{3}\), ∞) .
∴ The function is strictly deceasing in (-4, \(\frac{2}{3}\)) and is strictly decreasing in (-∞, -4)and (\(\frac{2}{3}\), ∞)

Inter 1st Year Maths 1B Applications of Derivatives Important Questions

Question 47.
Find the Intervals on which f(x) = xx (x > 0) is increasing and decreasing.
Solution:
Taking logarithms on both sides of f(x) = xx
we get
log (f(x)) = x log x. Differenetiating it w.r.t. x
we have \(\frac{1}{f(x)}\) f'(x)= 1 + log x
∴ f'(x) = xx( 1 + log x)
f'(x) = 0 ⇒ xx(1 + log x) = 0 ……………. (1)
⇒ 1 + log x = 0
⇒ x = 1/e
Inter 1st Year Maths 1B Applications of Derivatives Important Questions 17
Suppose x < 1/e log x < log (1/e) (since the base e > 1). i.e., log x < -1
1 + log x < 0 ⇒ xx (1 + log x) < 0. i.e., f'(x) < 0 Now suppose, x > 1/e. Then log x > log (1/e)
i.e., log x > – 1.
⇒ 1 + log x > 0
⇒ xx (1 + log x) > 0
⇒ f'(x) > 0
Hence, f is strictly decreasing on (o, 1/e) and it is strictly increasin on (1/e, ∞).

Question 48.
Determine the intervals in which f(x) = \(\frac{2}{(x-1)}\) + 18x ∀ x ∈ R \ {0} is strictly increasing and decreasing.
Solution:
Given that f(x) = \(\frac{2}{(x-1)}\) + 18x. Differenetiating
it w.r.t. x, we get
f'(x) = \(\frac{-1}{(x-1)^{2}}\) . 2 + 18 and f'(x) = 0
⇒ \(\frac{2}{(x-1)^{2}}\) = 18 ⇒ (x – 1)2 = 1/9.
∴ f'(x) = 0 if x – 1 = 1/3 or x- 1 = -(1/3).
i.e., x = 4/3 or x – 2/3.
The derivative of f(x) can be expressed as
f'(x) = \(\frac{18}{(x-1)^{2}}\) . (x – /3) (x – 4/3)
Inter 1st Year Maths 1B Applications of Derivatives Important Questions 18
∴ The given function f(x) is strictly increasing on (-∞, \(\frac{2}{3}\)) and (\(\frac{4}{3}\), ∞) and it is strictly decreasing on (\(\frac{2}{3}\), \(\frac{4}{3}\)).

Question 49.
Let f(x) = sin x – cos x be defined on [0, 2π]. Determine the intervals in which f(x) is strictly decreasing and strictly increasing.
Solution:
Given that f(x) = sin x – cos x.
∴ f'(x) = cos x + sin x
∴ f'(x) = \(\sqrt{2}\) . sin(x + π/4)
Let 0 < x < 3π/4. Then π/4 < x + π/4 < π. ∴ sin (x + π/4) > 0 i.e., f'(x) > 0.
Similarly it can be shown that f'(x) < 0 in (3π/4 . 7π/4) and f'(x) > 0 in (7π/4, 2π).
Inter 1st Year Maths 1B Applications of Derivatives Important Questions 19
Thus the function f(x) strictly increasing in (0, \(\frac{3\pi}{24}\) and (\(\frac{7\pi}{4}\), 2π) it is strictly decreasing in (\(\frac{3\pi}{4}\), \(\frac{7\pi}{4}\)).

Inter 1st Year Maths 1B Applications of Derivatives Important Questions

Question 50.
If 0 ≤ x ≤ \(\frac{\pi}{2}\) then show that x ≥ sinx.
Solution:
Let f(x) = x – sin x. Then f'(x) = 1 – cos x ≥ 0 ∀ x
∴ f is an increasing function for all x.
Now, f(0) = 0. Hence f(x) ≥ f(0) for all x ∈ (0, \(\frac{\pi}{2}\)). Therefore, x ≥ in x.

Question 51.
Let f : R → R be defined by f(x) = 4x2 – 4x + 11. Find the global minimum value and a point of global minimum.
Solution:
We have to look for a value c e R(domain) such that
f(x) ≥ f(c) ∀ x ∈ R
so that f(c) is the global minimum value of f. Consider
f(x) = 4x2 – 4x + 11 = (2x – 1)2 + 10 ≥ ∀ x ∈ R ……………(1)
Now, f(1/2) = 10
Also f(x) ≥ f(1/2) ∀ x ∈ R
Hence, f(1/2) = 10 is the global minimum value of f(x), and a point of global minimum is x = 1/2.

Question 52.
Let f : [-2, 2] → R be defined by f(x) = |x|. Find the global maximum of f(x) and a point of global minimum.
Solution:
We know that |x| = \(\left\{\begin{array}{ccc}
x & \text { if } & x \geq 0 \\
-x & \text { if } & x<0
\end{array}\right.\)
Therefore, from the graph of the function f on [-2, 2] clearly f(x) ≤ f(2) and f(x) ≤ f(-2) ∀ x ∈ [-2, 2].
Inter 1st Year Maths 1B Applications of Derivatives Important Questions 20
∴ f(2) = f(-2) = 2 is the global maximum of f(x), 2 and -2 are the points of global maximum.

Question 53.
Find the global maximum and global minimum of the function f : R → R defined by f (x) = x2.
Solution:
We have f(x) ≥ f(0) ∀ x ∈ R.
Hence th global minimum value of f(x) is 0 and a point of global minimum is x = 0.
Suppose f has global maximum at x0 ∈ R (x0 > 0). Then as per out assumption we have.
f(x0) ≥ f(x) ∀ x ∈ R
Inter 1st Year Maths 1B Applications of Derivatives Important Questions 21
Choose x1 = x0 + 1. Then x1 ∈ R and x0 < x1
∴ x02 < x12
Hence f(x0) < (fx1)
Thus we got f(x1) such that f(x0) > f(x0) which is a contradition to Therefore, f(x) has no global maximum on R.

Inter 1st Year Maths 1B Applications of Derivatives Important Questions

Question 54.
Find the stationary points of f(x) = 3x4 – 4x3 + 1, ∀ x ∈ R and state whether the function has local maxima or local minima at those points.
Solution:
Given that f(x) = 3x4 – 4x3 +1 and the domain of f is R. Differentiating the function w.r.t. x we have
f'(x) = 12x2(x – 1) ……………………… (1)
The stationary points are the roots of f (x) = 0 i.e., 12x2(x – 1) = 0. Hence x = 0 and x = 1 are the stationary points. Now, we test whether the stationary point x = 1 is a local extreme point or not. For.
f'(0.9) = 12(0.9)2 (0.9 – 1) ⇒ f'(0.9) is negative
f'(1.1) = 12(1.1)2 (1.1 – 1) ⇒ f'(1.1) is positive
and f(x) is defined in the neighbourhood i.e., (0.8, 1.2) of x = 1 with 8 = 0.2.
By theorem cis a point of local maximum if f(x) changes sign from positive to negative at x = c.
c is a point of local minimum if f'(x) changes sign from negative to positive at x = c.
The given function has local (relative) minimum at x = 1. Hence x = 1 is a local extreme point.
We will now test whether x = 0 is a local extreme point or not.
The function f(x) is defined in the neighbourhood of (-0.2, 0.2).
f'(-0.1) = 12(-0.1)2 (-0.1 – 1) ⇒ f'(-0.1) is negative
f'(-0.1) = 12(0.1)2 (0.1 -1) ⇒ f'(0.1) is negative
Thus, f(x) has no change in sign at x = 0. Therefore, the function f has no local maximum and no local minimum. Hence, x = 0 is not a local extreme point.

Question 55.
Find the points (if any) of local maxima and local minima of the function f (x) = x3 – 6x2 + 12x – 8 ∀ x ∈ R.
Solution:
Given function is f(x) = x3 – 6x2 + 12x – 8 and the domain of f is R.
Differentiating the given function w.r.t. x, we get
f'(x) = 3x2 – 12x + 12 i.e., f'(x) = 3(x – 2)2.
The stationary point of f(x) is x = 2, since 2 is a root of f'(x) = 0.
Choose δ = 0.2 The 0.2- neighbourhood of 2 is (1.8, 2.2). Now
f'(1.9) = 3(1.9 – 2)2 ⇒ f'(1.9) is positive
f'(2.1) = 3(2.1- 2)2 ⇒ f'(2.1) is positive
Thus f(x) does not change the sign at x = 2. By Theorem c is neither a point of local maximum nor a point of local minimum if f'(x) does not change sign at x = c.
x = 2 is neithere a local maximum nor a local minimum.

Inter 1st Year Maths 1B Applications of Derivatives Important Questions

Question 56.
Find the points of local minimum and local maximum of the function f(x) = sin 2x ∀ x ∈ [0, 2π]
Solution:
The given function is f(x) = sin 2x and domain is [0, 2π].
f'(x) = 2cos 2x …………… (1)
The critical points are the roots of 2 cos 2x = 0 and lying in the domain [0, 2π].
They are \(\frac{\pi}{4}\) and \(\frac{3\pi}{4}\) .
Now we apply the first derivative test at x = \(\frac{\pi}{4}\)
Clearly (\(\frac{\pi}{4}\) – 0.1 . \(\frac{\pi}{4}\) + 0.1) is a neighbourhood of \(\frac{\pi}{4}\) and the given f is defined on it.
Now
f'(\(\frac{\pi}{4}\) – 0.05) = 2 cos(\(\frac{\pi}{2}\) – 0.1) > 0
f'(\(\frac{\pi}{4}\) + 0.05) = 2 cos(\(\frac{\pi}{2}\) + 0.1) < 0
Thus f'(x) changes sign from positive to negative at x = \(\frac{\pi}{4}\). Therefore f has a local maximum.
Now we apply the first derivative test at x = \(\frac{3\pi}{4}\).
Clearly (\(\frac{3\pi}{4}\) – 0.1 . \(\frac{3\pi}{4}\) + 0.1) is a neighbourhood of \(\frac{3\pi}{4}\) and the given f is defined on it.
Now
f'(\(\frac{3\pi}{4}\) – 0.05) = 2 cos(\(\frac{3\pi}{4}\) – 0.1) < 0 f'(\(\frac{3\pi}{4}\) + 0.05) = 2 cos(\(\frac{3\pi}{4}\) + 0.1) > 0
Thus f'(x) changes sign from positive to negative at x = \(\frac{3\pi}{4}\). Therefore f has a local maximum at x = \(\frac{3\pi}{4}\).

Question 57.
Find the points of local extrema of the function f(x) = x3 – 9x2 – 48x + 6 ∀ x ∈ R Also find its local extrema.
Solution:
Given function is
f(x) = x3 – 9x2 – 48x + 6 …………… (1)
and the domain of the function is R.
Differentiating (1) w.r.t. x we get
f'(x) = 3x2 – 18x – 48 = 3(x – 8) (x + 2) ………….. (2)
Thus the stationary points are – 2 and 8.
Differentiating (2) w.r.t.x we get,
f'(x) = 6(x – 3) ………….. (3)
Let x1 = -2 and x2 = 8. Now we have to find f’ at each of these points to know the sign of second derivative.
At x1 = -2, f'(-2) = – 30. The sign of it is negative.
∴ x1 = – 2 is a point of local maximum of f and its local maximum value is f(-2) – 58.
Now, at x2 = 8 f'(8) = 30. Thus the sign of f”(x2) is positive. Therefore, x = 8 is a point of local minimum of f and its local minimum value is
f(8) = – 442.

Inter 1st Year Maths 1B Applications of Derivatives Important Questions

Question 58.
Find the points of local extrema of f(x) = x6 ∀ x ∈ R. Also find its local extrema.
Solution:
f(x) = x6 ………………….. (1)
Differentiating (1) w.r.t. x we get,
f'(x) = 6x5 ……………. (2)
and again differentiating (2) w.r.t. x we get
f'(x) = 30x4 …………………… (3)
The stationary point of f(x) is x = 0 only (since f'(x) = 0 only at x = 0).
Now f'(0) = 0. At x = 0, we can not conclude anything about the local extrema by the second derivative test. Therefore, we apply the first derivative test. As the domain of f is R, the function f is defined on (-0.2, 0.2) which is a neighbourhood of x = 0. Now
f'(-0.1) = 6(-0.1)5 < 0, f'(0, 1) = 6(0.1) 5 > 0.
Thus f'(x) changes sign form negative of positive at x = 0.
∴ x = 0 is a point of local minimum and its local minimum value is f(0) = 0.

Question 59.
Find the points of local extrema and local extrema for the function f(x) = cos 4x defined on (0, \(\frac{\pi}{2}\))
Solution:
Here f(x) = cos 4x ……………. (1)
and its domain is (0, \(\frac{\pi}{2}\))
∴ f’(x) = -4 sin 4x ………….. (2)
and f”(x) = -16 cos 4x ……………… (3)
The stationary points are the roots of
f'(x) = 0 and lying in the domain (o, \(\frac{\pi}{2}\)).
f'(x) = 0 ⇒ 4 sin 4x = 0
⇒ 4x = 0, π, 2π, 3π, 4π ………………….
⇒ x = 0, π/4, π/2, 3π/4, π …………………..
The point lying in the domain is x = \(\frac{\pi}{4}\) only.
Thus x = \(\frac{\pi}{4}\) is the stationary point of the given function. Now
f”(\(\frac{\pi}{4}\)) = -16 cos(π) = 16 > 0.
The function f has local minimum at x = \(\frac{\pi}{4}\) and its local minimum value is
f(\(\frac{\pi}{4}\)) = -1.

Question 60.
Find two positive number whose sum is 15 so that the sum of their squares is minimum.
Solution:
Suppose one numbers is x and the other number 15 – x. Let S be the sum of squares of these numbers. Then S = x2 + (15 – x)2 ………………… (1)
Note that the quantity S, to be minimized, is a function of x.
Differentiating (1) w.r.t. x, we get
\(\frac{\mathrm{dS}}{\mathrm{dx}}\) = 2x + 2(15 – x) (-1)
= 4x – 30 ………………. (2)
and again differentiating (2) w.r.t.x, we get
\(\frac{\mathrm{d}^{2} \mathrm{~S}}{\mathrm{dx}^{2}}\) = 4 ……………….. (3)
The stationary point can be obtained by solving \(\frac{\mathrm{dS}}{\mathrm{dx}}\) = 0 i.e., 4x – 30 = 0.
∴ x = 15/2 is the stationary point of (1).
Since \(\frac{\mathrm{d}^{2} \mathrm{~S}}{\mathrm{dx}^{2}}\) = 4 > 0, S is minimum at x = \(\frac{15}{2}\)
∴ The two numbers are \(\frac{15}{2}\), 15 – \(\frac{15}{2}\) i.e., \(\frac{15}{2}\) and \(\frac{15}{2}\).

Inter 1st Year Maths 1B Applications of Derivatives Important Questions

Question 61.
Find the maximum area of the rectangle that can be formed with fixed perimeter 20.
Solution:
Let x and y denote the length and the breadth of a rectangle respectively. Given that the perimeter of the rectangle is 20.
i.e., 2(x + y) = 20
i.e., x + y = 10 ……………. (1)
Let A denote the area of rectangle.
Then A = xy ………….. (2)
Which is to be minimized. Equation (1) can be expressed as
y = 10 – x …………… (3)
From (3) and (2), we have
A = x (10 – x)
A = 10x – x2 ……………… (4)
Differentiating (4) w.r.t. x we get
\(\frac{\mathrm{dA}}{\mathrm{dx}}\) = 10 – 2x ……………….. (5)
The stationary point is a root of 10 – 2x = 0
∴ x = 5 is the stationary point.
Differentiating (5) w.r.t. x, we get
\(\frac{\mathrm{d}^{2} \mathrm{~A}}{\mathrm{dx}^{2}}\) = -2
which is negative. Therefore by second derivative test the area A is maximized at x = 5 and hence y = 10 – 5 = 5, and the maximum area is A = 5(5) = 25.

Question 62.
Find the point on the graph y2 = x which is the nearest to the point (4, 0).
Solution:
Inter 1st Year Maths 1B Applications of Derivatives Important Questions 22
Let P(x, y) be any point on y2 = x and A(4, 0). We have to find P such that PA is minimum
Suppose PA = D. The quantity to be minimized is D.
D = (\(\sqrt{(x-4)^{2}+(y-0)^{2}}\)) ………………. (1)
P(x, y) lies on the curve, therefore
y2 = x …………………… (2)
From (1) and (2), we have
D = \(\sqrt{\left((x-4)^{2}+x\right)}\)
D = \(\sqrt{\left(x^{2}-7 x+16\right)}\) ……………………. (3)
Differentiating (3) w.r.t. x, we get
\(\frac{\mathrm{dD}}{\mathrm{dx}}\) = \(=\frac{2 x-7}{2} \cdot \frac{1}{\sqrt{x^{2}-7 x+16}}\)
Now \(\frac{\mathrm{dD}}{\mathrm{dx}}\) = 0
gives x = \(\frac{7}{2}\). Thus \(\frac{7}{2}\) is a stationary point of the function D. We apply the first derivative test to verify whether D is minimum at x = \(\frac{1}{2}\)
Inter 1st Year Maths 1B Applications of Derivatives Important Questions 23

Inter 1st Year Maths 1B Applications of Derivatives Important Questions

Question 63.
Prove that the radius of the right circular cylinder of greatest curved surface area which can be inscribed in a given cone is half of that of the cone.
Solution:
Let O be the centre of the circular base of the cone and its height be h. Let r be the radius of the circular base of the cone.
Then AO = h, OC = r.
Let a cylinder with radius x(OE) be inscribed in the given cone. Let its height be u.
Inter 1st Year Maths 1B Applications of Derivatives Important Questions 24
i.e., RO = QE = PD = u
Now the triangles AOC and QEC are similar.
Therefore,
\(\frac{\mathrm{QE}}{\mathrm{OA}}\) = \(\frac{\mathrm{EC}}{\mathrm{OC}}\)
i.e., \(\frac{u}{h}\) = \(\frac{r-x}{r}\)
∴ u = \(\frac{h(r-x)}{r}\) ………………… (1)
Let S denote the curved surface area of the chosen cylinder.Then
S = 2 π xu.
As the cone is fixed one, the values of r and h are constants. Thus S is function of x only. Now,
\(\frac{\mathrm{dS}}{\mathrm{dx}}\) = 2 πh (r – 2x)/r and \(\frac{\mathrm{d}^{2} \mathrm{~S}}{\mathrm{dx}^{2}}\) = -4πh/r.
The stationary point of S is a root of
\(\frac{\mathrm{dS}}{\mathrm{dx}}\) = 0
i.e., π(r – 2x)/r = 0
i.e., x = r/2
\(\frac{\mathrm{d}^{2} \mathrm{~S}}{\mathrm{dx}^{2}}\) < 0 for all x, Therefore (\(\frac{\mathrm{d}^{2} \mathrm{~S}}{\mathrm{dx}^{2}}\))x=r/2 < 0.
Hence, the radius of the cylinder of greatest curved surface area which can be inscribed in a given cone is r/2.

Question 64.
The profit function P(x) of a company, selling x items per day is given by P(x) = (150 – x)x – 1600. Find the number of items that the company should sell to get maximum profit. Also find the maximum profit.
Solution:
Given that the profit function is
P(x) = ( 150 – x)x – 1600 ………….. (1)
For maxima or minima \(\frac{\mathrm{dP}(\mathrm{x})}{\mathrm{dx}}\) = 0
∴ (150 – x) (1) + x (-1) = 0
i.e., x = 75
Now \(\frac{d^{2} P(x)}{d x^{2}}\) = -2 and \(\left[\frac{d^{2} P(x)}{d x^{2}}\right]_{x=75}\) < 0.
∴ The profit P(x) is maximum for x = 75.
∴ The company should sell 75 items a day to make maximum profit.
The maximum profit will be P(75) = 4025.

Question 65.
A manufacturer can sell x items at a price of rupees (5 – x/100) each. The cost price of x items is Rs. (x/5 + 500). Find the number of items that the manufacturer should sell to earn maximum profits.
Solution:
Let S(x) be the selling price of x items and C(x) be the cost price of x items. Then, we have
S(x) = {cost of each item}, x .
∴ S(x) = (5 – x/100) x = 5x – x2/100 and C(x) = x/5 + 500
Let P(x) denote the profit function. Then
P(x) = S(x) – C(x)
P(x) = (5x – x2/100) – (x/5 + 500)
– (24x/5) – (x2/100) – 500 …………….. (1)
For maxima or minima
\(\frac{\mathrm{dP}(\mathrm{x})}{\mathrm{d} x}\) = 0
i.e., 24/5 – x/50 = 0
The stationary point of P(x) is x = 240 and
\(\left[\frac{d^{2} P(x)}{d x^{2}}\right]\) = –\(\frac{1}{50}\) for all x.
Hence the manufacturer can earn maximum profit if he sells 240 items.

Inter 1st Year Maths 1B Applications of Derivatives Important Questions

Question 66.
Find the absolute extrement of f(x) = x2 defined on [-2, 2].
Solution:
The given function f(x) = x2 is continuous on [-2, 2]. It can be shown that it has only local minimum and the point of local minimum is 0. The absolute(global) maximum of f is the largest value of f(-2), f(0) and f(2) i.e., 4, 0, 4.
Hence, the absolute maximum value is 4. Similarly the absolute minimum is the least value of 4, 0, 4. Hence 0 is the absolute minimum value.

Question 67.
Find the absolute maximum of x40 – x20 on the interval [0, 1]. Find also its absolute maximum value.
Solution:
Let f(x) = x40 – x20 ∀ x ∈ [0,1] ………… (1)
The function f is continuous on [0,1 j and the interval [0,1] is closed.
From (1) we have
f'(x) = 40 x39 – 20 x19 = 20x19 (2x20 – 1).
Thus f'(x) = 0 at x = 0 or
x = \(\left(\frac{1}{2}\right)^{\frac{1}{20}}\)
Therefore, the critical points of f are and \(\left(\frac{1}{2}\right)^{\frac{1}{20}}\) and 0 is one of the end points of the domain. Therefore no local maximum exists at x = 0. Now
f'(x) = 40(39) x38 – 20(19) x18
= 20x18 (78 x20 – 19)
[f”(x)]x = \(\left(\frac{1}{2}\right)^{\frac{1}{20}}\) = 20 (1/2)(18/20)[39 – 19] > 0.
Therefore f has local minimum at
x = (1/2)(1/20)
and its value is \(f\left(\left(\frac{1}{2}\right)^{\frac{1}{20}}\right)\) = –\(\frac{1}{4}\)
Therefore the absolute maximum value of the function f is the largest value of f(0), f(1) and \(f\left(\left(\frac{1}{2}\right)^{\frac{1}{20}}\right)\) i.e., the largest value of {0, 0, –\(\frac{1}{4}\)}
Hence, the absolute maximum of f is 0 and the points of absolute maximum are 0 and 1. Further the absolute minimum is the least of 0, 0, -latex]\frac{1}{4}[/latex].
Hence the absolute minimum is -latex]\frac{1}{4}[/latex] and the point of absolute minimum is x = \(\left(\frac{1}{2}\right)^{\frac{1}{20}}\)

Inter 1st Year Maths 1B The Plane Solutions Ex 7(a)

Practicing the Intermediate 1st Year Maths 1B Textbook Solutions Inter 1st Year Maths 1B The Plane Solutions Exercise 7(a) will help students to clear their doubts quickly.

Intermediate 1st Year Maths 1B The Plane Solutions Exercise 7(a)

I.

Question 1.
Find the equation of the plane if the foot of the perpendicular from origin to the plane is (1, 3, -5).
Solution:
OP is the normal to the plane D. Rs of op are 1, 3, -5
The plane passes through P( 1, 3, -5) equation of the plane is
1(x – 1) + 3(y – 3) – 5(z + 5) = 0
x – 1 + 3y – 9 – 5z – 25 = 0
x + 3y – 5z – 35 = 0
Inter 1st Year Maths 1B The Plane Solutions Ex 7(a) 1

Question 2.
Reduce the equation x + 2y – 3z – 6 = 0 of the plane to the normal form.
Solution:
Equation of the plane is x + 2y – 3z – 6 = 0
i.e., x + 2y – 3z = 6
Dividing in the
\(\sqrt{1^{2}+2^{2}+(-3)^{2}}=\sqrt{1+4+9}=\sqrt{14}\)
The equation of the plane in the normal form is
Inter 1st Year Maths 1B The Plane Solutions Ex 7(a) 2

Question 3.
Find the equation of the plane. Whose intercepts on X, Y, Z – axis are 1, 2, 4 respectively.
Solution:
Equation of the plane in the intercept form is
\(\frac{x}{a}+\frac{y}{b}+\frac{z}{c}\) = 1
Given a = 1, b = 2, c = 4.
Equation of the required plane in the intercept form is
\(\frac{x}{1}+\frac{y}{2}+\frac{z}{4}\) = 1
Multiplying with 4, we get
4x + 2y + z = 4

Inter 1st Year Maths 1B The Plane Solutions Ex 7(a)

Question 4.
Find the intercepts of the plane 4x + 3y – 2z + 2 = 0 on the co-ordinate axes.
Solution:
Equation of the plane is 4x + 3y – 2z + 2 = 0
– 4x – 3y + 2z = 2
Inter 1st Year Maths 1B The Plane Solutions Ex 7(a) 3
x – intercept = -1/2, y – intercept = -2/3, z intercept =1.

Question 5.
Find the d.c.’s of the normal to the plane x + 2y + 2z – 4 = 0.
Solution:
Equation of the plane is x + 2y + z- 4 = 0
d.r.’s of the normal are (1, 2, 2)
Dividing with \(\sqrt{1+4+4}\) = 3,
d.c.’s of the normal to the plane are (\(\frac{1}{3}, \frac{2}{3}, \frac{2}{3}\))

Question 6.
Find the equation of the plane passing through the point (-2, 1, 3) and having (3, -5, 4) as d.r.’s of its normal.
Solution:
d.r.’s of the normal are (3, -5, 4) and the plane passes through (-2, 1, 3).
Equation of the plane is 3(x + 2) – 5(y – 1) + 4(z – 3) = 0
3x + 6 – 5y + 5 + 4z – 12 = 0
3x -5y+ 4z – 1 = 0.

Question 7.
Write the equation of the plane 4x – 4y + 2z + 5 = 0 in the intercept form.
Solution:
Equation of the plane is
4x – 4y + 2z + 5 = 0
– 4x + 4y – 2z = 5
\(-\frac{4x}{5}, \frac{4y}{5}, \frac{2z}{5}\)
Intercept form is
Inter 1st Year Maths 1B The Plane Solutions Ex 7(a) 4
x – intercept = \(\frac{5}{4}\), y – intercept = \(\frac{5}{4}\), z – intercept = –\(\frac{5}{2}\)

Question 8.
Find the angle between the planes x + 2y + 2z-5 = 0 and 3x + 3y + 2z – 8 = 0.
Solution:
Equation of the planes are
x + 2y + 2z – 5 = 0
3x + 3y + 2z – 8 = 0
Inter 1st Year Maths 1B The Plane Solutions Ex 7(a) 5

II.

Question 1.
Find the equation of the plane passing through the point (1,1,1) and parallel to the plane x +.2y + 3z – 7 = 0.
Solution:
Equation of the given plane is x + 2y + 3z – 7 = 0.
Equation of the parallel plane is x + 2y + 3z = k.
This plane passes through the point P (1, 1, 1)
1 + 2 + 3 = k ⇒ k = -6
Equation of the required plane is x + 2y + 3z = 6

Question 2.
Find the equation of the plane passing through (2, 3, 4) and perpendicular to x – axis.
Solution:
The plane is perpendicular to x – axis
∴ x – axis is the normal to the plane
d.c.’s of x -axis are 1, 0, 0
Equation of the required plane is x = k
This plane passes through the point P(2, 3, 4)
∴ 2 = k
Equation of the required plane is x = 2.

Inter 1st Year Maths 1B The Plane Solutions Ex 7(a)

Question 3.
Show that 2x + 3y + 7 = 0 represents a plane perpendicular to XY – plane.
Solution:
Equation of the given plane is 2x + 3y + 7 = 0
Equation of xy – plane is z = 0
a1a2 + b1b2 + c1c2 = 2.0 + 3.0 + 0.1 = 0
+0+0=0
The plane 2x + 3y + 7 = 0 is perpendicular to XY – plane.

Question 4.
Find the constant k so that the planes x – 2y + kz = 0 and 2x + 5y – z = 0 are at right angles. Find the equation of the plane through (1, -1, -1) and perpen-dicular to these planes.
Solution:
Equations of the given planes are x – 2y + kz = 0 and 2x + 5y – z = 0
These the planes are perpendicular
1.2 – 2.5 + k (-1) = 0
2 – 10 = k ⇒ k = -8
Equation of the planes
x – 2y – 8z = 0 ………. (1)
2x + 5y – z = 0 ……….. (2)
The required plane passes through (1, -1, -1)
∴ Equation of the plane can be taken as
a(x – 1) + b(y + 1) + c(z + 1) = 0 ………… (3)
This plane is perpendicular to the planes (1) and (2)
a – 2b – 8c = 0
2a + 5b – c = 0
Inter 1st Year Maths 1B The Plane Solutions Ex 7(a) 6
Substituting in (3), equation of the required planes
42 (x – 1) – 15(y + 1) + 9(z + 1) = 0
42x – 42 – 15y – 15 + 9z + 9 = 0
42x – 15y + 9z – 48 = 0.

Question 5.
Find the equation of the plane through (-1, 6, 2) and perpendicular to the join of (1, 2, 3) and (-2, 3, 4).
Solution:
The plane is perpendicular to the line joining A(1, 2, 3) and B(-2, 3, 4).
d.r.’s of AB are 1 + 2, 2 – 3, 3 – 4
i.e., 3, -1, -1
Inter 1st Year Maths 1B The Plane Solutions Ex 7(a) 7
AB is normal to the plane and the plane passes through the point P(-1, 6, 2)
Equation of the required plane is 3(x + 1) – 1(y – 6) -1(z – 2) = 0
3x + 3 – y + 6- z + 2 = 0
3x – y – z + 11 = 0

Question 6.
Find the equation of the plane bisecting the line segment joining (2, 0, 6) and (-6, 2, 4) and perpendicular to it.
Solution:
A (2, 0, 1), B(-6, 2, 4) are the given points ‘o’ is the mid point of AB
Inter 1st Year Maths 1B The Plane Solutions Ex 7(a) 8
Co-ordinates of O are
\(\left(\frac{2-6}{2}, \frac{0+2}{2}, \frac{6+4}{2}\right)\) = (-2, 1, 5)
The plane is perpendicular to AB
d.r.’s of the normal to the plane are
2 + 6, 0 – 2, 6 – 4
8, -2, 2
Equation of the required plane is
+8 (x + 2) – 2(y – 1) + 2 (z – 5) = 0
8x + 16 – 2y + 2 + 2z – 10 = 0
8x – 2y + 2z + 8 = 0

Question 7.
Find the equation of the plane passing through (0, 0, -4) and perpendicular to the line joining the points (1, -2, 2) and (-3, 1, -2).
Solution:
A(1, -2, 2), B(-3, 1, -2) are the given points
d.r.’s of AB are 1 + 3, -2 -1, 2 + 2 i.e., 4, -3, 4
AB is normal to the plane and the plane passes through the point P(0, 0, -4).
Equation of the required plane is
4(x – 0) – 3(y – 0) + 4(z + 4) = 0
4x – 3y + 4z + 16 = 0

Inter 1st Year Maths 1B The Plane Solutions Ex 7(a)

Question 8.
Find the equation of the plane through (4, 4, 0) and perpendicular to the planes 2x + y + 2 z + 3 = 0 and 3x + 3y + 2z – 8 = 0.
Solution:
Equation of the plane passing through P(4, 4, 0) is
a(x – 4) + b(y – 4) + c(z – 0) = 0 ………. (1)
This plane is perpendicular to
2x + y + 2z – 3 = 0
3x + 3y + 2z – 8 = 0
∴ 2a + b + 2c = 0 ………… (2)
3a + 3b + 2c = 0 ………….(3)
Inter 1st Year Maths 1B The Plane Solutions Ex 7(a) 9
Substituting in (1), equation of the required plane is
-4 (x – 4) + 2(y – 4) + 3(z – 0) = 0
-4x + 16 + 2y – 8 + 3z = 0
-4x + 2y + 3z + 8 = 0

III.

Question 1.
Find the equation of the plane through the points (2, 2, -1), (3, 4, 2), (7, 0, 6).
Solution:
A(2, 2, -1), B(3, 4, 2), C(7, 0, 6) are the given points.
Equation of the plane passing through A(2, 2, -1) is
a(x – 2) + b(y – 2) + c(z + 1) = 0
This plane passes through B(3, 4, 2) and C(7, 0, 6)
a(3 – 2) + b(4 – 2) +c(2 + 1) = 0
a + 2b + 3c = 0 ……….. (2)
a(7 – 2) + b(0 – 2) + c(6 + 1) = 0
5a – 2b + 7c = 0 ……….. (3)
From (2) and (3)
Inter 1st Year Maths 1B The Plane Solutions Ex 7(a) 10
Substituting in (1) equation of the required plane is
5(x- 2) + 2(y- 2) – 3(z +1) = 0
5x – 10 + 2y – 4 – 3z – 3 = 0
5x + 2y – 3z -17 = 0 or 5x + 2y- 3z = 17

Question 2.
Show that the points (0, -1, 0), (2, 1, -1), (1, 1, 1), (3, 3, 0) are coplanar.
Solution:
Equation of the plane through A(0, -1, 0) is
ax + b(y + 1) + cz = 0 …….. (1)
This plane passes through B(2, 1, -1) and C(1, 1, 1)
2a + 2b – c = 0 …………. (2)
a + 2b + c = 0 ………….. (3)
(2) – (3) gives a – 2c = 0 ⇒ a = 2c ⇒ \(\frac{a}{2}=\frac{c}{1}\)
(2) + (3) gives 3a + 4b = 0 ⇒ 3a = – 4b
⇒ \(\frac{a}{4}=\frac{b}{-3}\)
∴ \(\frac{a}{4}=\frac{b}{-3}=\frac{c}{2}\)
Substitutes in (1) equation of the plane ABC is
4x – 3(y + 1) + 2(z – 0) = 0
4x – 3y + 2z – 3 = 0
4x – 3y + 2z – 3 = 4.3 – 3.3. + 0.3
= 12 – 9 – 3 = 0
The given points A, B, C, D are coplanar.

Inter 1st Year Maths 1B The Plane Solutions Ex 7(a)

Question 3.
Find the equation of the plane through (6, – 4, 3), (0, 4, -3) and cutting of inter-cepts whose sum is zero.
Solution:
Suppose a, b, c are the intercepts of the plane.
Equation of the plane is \(\frac{x}{a}+\frac{y}{b}+\frac{z}{c}\) = 1
Given a + b + c = 0
c = – (a + b)
The plane passes through
P(6, -4, 3), Q(0, 4, -3)
Inter 1st Year Maths 1B The Plane Solutions Ex 7(a) 11
c = -a – b = -3 – b
4(-3 – b) – 3b = b(-3 – b)
-12 – 4b – 3b = – 3b – b²
b² – 4b – 12 = 0
(b – 6) (b + 2) = 0 ⇒ b = 6 – 2

Case i) b = 6
c = -3 – b = -3 – 6 = -9
Equation of the plane is
\(\frac{x}{3}+\frac{y}{6}+\frac{z}{9}\) = 1
6x + 3y – 2z = 18

Case ii) b = -2
c = -3 – b = -3 + 2 = -1
Equation of the plane is
\(\frac{x}{3}+\frac{y}{-2}+\frac{z}{-1}\) = 1

Question 4.
A plane meets the co-ordinate axes in A, B, C. If the centroid of ∆ABC is (a, b, c). Show that the equation of the plane is
\(\frac{x}{a}+\frac{y}{b}+\frac{z}{c}\) = 1
Solution:
Suppose α, β, γ are the intercepts of the plane ABC.
Equation of the plane is the intercept form is
\(\frac{x}{\alpha}+\frac{y}{\beta}+\frac{z}{\gamma}=1\) ………… (1)
Inter 1st Year Maths 1B The Plane Solutions Ex 7(a) 12
Co-ordinates of A are (α, 0, 0), B are (0, β, 0) and C are (0, 0, γ)
G is the centroid of ∆ABC
Co-ordinates of Gare =
Inter 1st Year Maths 1B The Plane Solutions Ex 7(a) 13
α = 3a, β = 3b, γ = 3c
Substituting in (1), equation of the plane ABC is
Inter 1st Year Maths 1B The Plane Solutions Ex 7(a) 14

Question 5.
Show that the plane through (1, 1, 1) , (1, -1, 1) and (-7, -3, -5) is parallel . to Y-axis.
Solution:
Equation of the plane through A (1, 1, 1) can be taken as
a(x – 1) +b(y – 1) + c(z – 1) = 0 ……….. (1)
This plane passes through B(1, -1, 1) and C(-7, -3, -5)
0 – 2b + 0 = 0 ⇒ b – 0
Equation of zy – plane is y = 0
0.x + 1.y + 0.z = 0
a.0 + 0.1 + c.0 = 0
The required plane is perpendicular to zx – plane hence it is parallel to Y – axis.

Inter 1st Year Maths 1B The Plane Solutions Ex 7(a)

Question 6.
Show that the equations ax + by + r =0, by + cz + p = 0, cz + ax + q = 0 represent planes perpendicular to xy,yz, zx planes respectively.
Solution:
Equation of the given plane is
ax + by + c = Q
d.rs of the normal are (a, b, c)
Equation of XY – plane is z = 0
d.rs of the normal are (0, 0, 1)
a.0 + b.0 + 0.1 = 0
∴ ax + by + r = 0 is perpendicular to xy – plane.
Similarly we can show that by + cz + p = 0 is perpendicular to yz – plane and cz + ax + q – 0 is perpendicular to zx – plane.

Inter 1st Year Maths 1B The Straight Line Important Questions

Inter 1st Year Maths 1B The Straight Line Important Questions

Students get through Maths 1B Important Questions Inter 1st Year Maths 1B The Straight Line Important Questions which are most likely to be asked in the exam.

Intermediate 1st Year Maths 1B The Straight Line Important Questions

Question 1.
Find the equation of the straight line pass-ing through the point (2, 3) and making non-zero intercepts on the axes of co-ordinates whose sum is zero. [Mar 12]
Sol. Equation of the line in the intercept form is
\(\frac{x}{a}\) + \(\frac{y}{b}\) = 1
Given b = -a
Equation of the line is \(\frac{x}{a}\) – \(\frac{y}{a}\) = 1
⇒ x – y = a
This line passes through (2, 3)
2 – 3 = a ⇒ a = -1
Equation of the line is
x – y = -1 or x – y + 1 = 0

Inter 1st Year Maths 1B The Straight Line Important Questions

Question 2.
Find the equation of the straight line passing through the points (at12, 2at1 ) and (at22, 2at1 ).
Solution:
Equation of the given points.
(x – x1) (y – y2) = (y – y1) (x1 – x2)
( x -at12) (2at1 – 2at2)
= (y – 2at1) (at, — at2)
(x – at12) .2a(t1 – t2) = (y – 2at1) a. (t12 – t22)
2x – 2at12 = y(t1 + t2) – 2at12 + 2at1t2 = 0
2x – (t1 + t2)y + 2at1t2 = 0

Question 3.
Find the equation of the straight line passing through A (-1, 3) and i) parallel ii) perpendicular to the straight line passing through B(2, – 5) and C(4, 6). [Mar 11]
Solution:
Slope of BC = \(\frac{-5-6}{2-4}\) = \(\frac{-11}{-2}\) = \(\frac{11}{2}\)
i) The required line is parallel to BC and passes through A(- 1, 3)
Equation of the parallel line is
y – 3 = \(\frac{11}{2}\)(x + 1)
2y – 6 = 11x + 11
11x – 2y + 17 = 0

ii) The required line is perpendicular to BC.
Slope of the required line = –\(\frac{1}{m}\) = –\(\frac{2}{11}\)
This line passes through A (-1, 3)
Equation of the required line is
y – 3 = –\(\frac{2}{11}\)(x + 1)
11y – 33 = -2x – 2
2x + 11y – 31 = 0

Inter 1st Year Maths 1B The Straight Line Important Questions

Question 4.
Prove that the points (1, 11), (2, 15) and (-3, -5) are collinear and find the equation of the straight line containing them.
Solution:
A(1, 11), B(2, 15) and C (-3, -5) are the given points.
Equation of AB is
(y – y1) (x1 – x2) = (x – x1) (y1 – y2)
(y – 11) (1 -2) = (x – 1) (11 – 15)
-(y – 11) = -4 (x – 1)
-y + 11 = – 4x + 4
4x – y + 7 = 0
C (-3, -5) .
4x – y + 7 = 4(-3) + 5 + 7
= -12 + 12 = 0
C lines on AB ⇒ A, B, C are collinear.
Equation of the line containing them is 4x – y + 7 = 0

Question 5.
A straight line passing through A(1, -2) makes an angle tan-1 \(\frac{4}{3}\) with the positive direction of the X – axis in the anticlock-wise sense. Find the points on the straight line whose distance from A is 5.
Solution:
Inter 1st Year Maths 1B The Straight Line Important Questions 1
Given α = tan-1 \(\frac{4}{3}\) ⇒ tan α = \(\frac{4}{3}\)
Inter 1st Year Maths 1B The Straight Line Important Questions 2
cos α = \(\frac{3}{5}\), sin α = \(\frac{4}{5}\)
(x1, y1) = (1, -2) = x1 = 1, y1 = -2
Case i) .
r = 5
x = x1 + r cos α = 1 + 5 . \(\frac{4}{5}\) = 1 + 4 = 5
y = y1 + r sin α= – 2 + 5 . \(\frac{3}{5}\) = -2 + 3 = 1
Co-ordinates of B are (5, 1)

Case ii) r = -5
x = x1 + r cos α = 1 – 5 . \(\frac{4}{5}\) = 1 – 4 = -3
y = y1 + r sin α = -2 – 5 . \(\frac{3}{5}\) = -2 – 3 = -5
Co-ordinates of C are (- 3, – 5)

Question 6.
A straight line parallel to the line y = \(\sqrt{3} x\) passes through Q(2, 3) and cuts the line 2x + 4y – 27 = 0 at P. Find the length of PQ.
Solution:
PQ is parallel to the straight line y = \(\sqrt{3} x\)
tan α = \(\sqrt{3}\) = tan 60°
α = 60°
Q(2, 3) is a given point
Inter 1st Year Maths 1B The Straight Line Important Questions 3
Co-ordinates of any point P are
(x1 + r cos α, y1 + r sin α)
(2 + r cos 60°, 3 + r sin 60°)
Inter 1st Year Maths 1B The Straight Line Important Questions 4

Inter 1st Year Maths 1B The Straight Line Important Questions

Question 7.
Transform the equation
3x + 4y+ 12 = 0 into
i) slope—intercept form
ii) Intercept form and
in) normal form
Solution:
The given equation is 3x + 4y + 12=0
i) Slope-intercept form
4y = -3x – 12
y = (-\(\frac{3}{4}\))x + (-3)
Slope = –\(\frac{3}{4}\) , y – intercept = -3.

ii) Intercept form
-3x – 4y = 12
–\(\frac{3 x}{12}\) – \(\frac{4 y}{12}\) = 1
\(\frac{x}{(-4)}\) + \(\frac{y}{(-3)}\) = 1
x – intercept = -4, y – intercept = -3

iii) Normal form
-3x – 4y = 12
Dividing with \(\sqrt{9+16}\) = 5
(-\(\frac{3}{5}\))x + (-\(\frac{4}{5}\))y = \(\frac{12}{5}\)
Let cos α = \(\frac{-3}{5}\) and sin α = –\(\frac{4}{5}\)
p = \(\frac{12}{5}\) so that
x cos α + y sin α = p
α lies in third quadrant so that
α = π + tan-1(\(\frac{4}{3}\))

Question 8.
If the area of the triangle formed by the straight line x = 0, y = 0 and 3x + 4y = a (a > 0) is 6, find the value of a. [May 11]
Solution:
Equation of the line is 3x + 4y = a
Inter 1st Year Maths 1B The Straight Line Important Questions 5
Given \(\frac{a^{2}}{2 a}\) = 6 ⇒ a2 = 144
a = ±12
But a > 0
a = 12

Inter 1st Year Maths 1B The Straight Line Important Questions

Question 9.
Find the value of k, if the lines 2x – 3y + k = 0, 3x – 4y -13 = 0 and 8x – 11y – 33 = 0 are concurrent.
Solution:
Let L1, L2, L3 be the straight lines whose equations are respectively
2x – 3y + k = 0 ………………. (1)
3x – 4y – 13 = 0 ……………….. (2)
8x – 11y – 33 = 0 ………………. (3)
Solving (2) and (3) for x and y
Inter 1st Year Maths 1B The Straight Line Important Questions 6
Point of the lines (2) and (3) is (11, 5)
The given lines L1, L2, L3 are concurrent
∴ L1 contain (11, 5)
∴ 2(11) – 3(5) + k = 0
k = -7

Question 10.
If the straight lines ax + by + c = 0, bx + cy + a = 0 and cx + ay + b = 0 are concurrent, then prove that a3 + b3 + c3 = 3abc.
Solution:
The equation of the given lines are
ax + by +c = 0 ………………… (1)
bx + cy + a = 0 ……………….. (2)
cx + ay + b = 0 ……………….. (3)
Solving (1) and (2) point of intersection is got by
Inter 1st Year Maths 1B The Straight Line Important Questions 7
c(ab – c2) +a (bc – a2) + b(ca – b2) = 0
abc – c3 + abc – a3 + abc – b3 = 0
∴ a3 + b3 + c3 = 3abc.

Question 11.
A variable straight line drawn through the point of intersection of the straight lines \(\frac{x}{a}\) + \(\frac{y}{b}\) = 1 and \(\frac{x}{b}\) + \(\frac{y}{a}\) = 1 meets the co-ordinate axes at A and B. Show that the locus of the mid point of \(\overline{\mathrm{AB}}\) is 2(a + b) xy = ab(x + y)
Solution:
Equations of the given lines are \(\frac{x}{a}\) + \(\frac{y}{b}\) = 1
and \(\frac{x}{b}\) + \(\frac{y}{a}\) = 1
Solving the point of intersection P(\(\frac{a b}{a+b}\), \(\frac{a b}{a+b}\))
Q (x0, y0) is any point on the locus
⇔ The line with x – intercept 2x0, y – intercept 2y0, passes through P
⇔ P lies on the straight line \(\frac{x}{2 x_{0}}\) + \(\frac{y}{2 y_{0}}\) = 1
i.e., \(\frac{a b}{a+b}\)(\(\frac{1}{2 x_{0}}\) + \(\frac{1}{2 y_{0}}\)) = 1
⇒ \(\frac{a b}{a+b}\) . \(\frac{x_{0}+y_{0}}{2 x_{0} y_{0}}\) = 0
ab(x0 + y0) = 2(a + b) x0y0
Q(x0, y0) lies on the curve
2(a + b)xy = ab(x + y)
Locks the mid of point of
AB is 2(a + b)xy = ab(x +y)

Inter 1st Year Maths 1B The Straight Line Important Questions

Question 12.
If a, b, c are in arithmetic progression, then show that the equation ax + by + c = 0 represents a family of concurrent lines and find the point of concurrency.
Solution:
a, b, c are in A.P.
2b = a + c
a – 2b + c = 0
a.1 + b(-2) + c = 0
Each number of family of straight lines
ax + by + c = 0
passes through the fixed point (1,-2)
∴ Set of lines ax + by + c = 0 for parametric values of a, b and c is a family of concurrent lines.
∴ Point of concurrency is (1, -2).

Question 13.
Find the value of k, if the angle between the straight lines 4x – y + 7 = 0 and kx – 5y – 9 = 0 is 45°.
Solution:
Inter 1st Year Maths 1B The Straight Line Important Questions 8
Squaring and cross multiplying
2(4k + 5)2 = 17(k2 + 25)
2(16k2 + 40k + 25) = 17k2 + 425
32k2 + 80k + 50 = 17k2 + 425
15k2 + 80k – 375 = 0
3k2 + 16k – 75 = 0
(k – 3) (3k + 25) = 0
k = 3 or -25/3

Question 14.
Find the equation of the straight line passing through (x0, y0) and (i) parallel (ii) perpendicular to the straight line
ax + by + c = 0.
Solution:
Equation of the given line is ax + by + c = 0
i) Equation of the parallel line is ax + by = k ………………… (1)
This line passes through P(x0, y0) ⇒ ax0 + by0 = k ……………… (2)
Subtracting (2) from (1) equation of the required line is a(x – x0) + b(y – y0) = 0

ii) Equation of the perpendicular line is bx – ay = k.
This line passes through P(x0, y0) ⇒ bx0 – ay0 = k
Subtracting, equation of the required line is b(x – x0) – a(y – y0) = 0

Inter 1st Year Maths 1B The Straight Line Important Questions

Question 15.
Find the equation of the straight line perpendicular to the line 5x – 2y = 7 and passing through the point of intersection of the lines 2x + 3y = 1 and 3x + 4y = 6.
Solution:
Given lines are L1 = 2x + 3y – 1 = 0
L2 = 3x + 4y – 6 = 0
Equation of the line passing through the intersection of L1 = 0, L2 = 0 is
L1 + kL2 = 0
(2x + 3y – 1) + k(3x + 4y – 6) = 0
(2 + 3k)x + (3 + 4k)y – (1 + 6k) = 0 …………………. (1)
This line is perpendicular to 5x – 2y = 7 ………………… (2)
a1a2 + b1b2 = 0
5(2 + 3k) – 2(3 + 4k) = 0
10 + 15k – 6 – 8k = 0
7k = -4 ⇒ k = – 4/7
Substituting in (1) equation of the required lines
(2 – \(\frac{12}{7}\))x + (3 – \(\frac{16}{7}\))x – (1 – \(\frac{24}{7}\)) = 0
\(\frac{2}{7}\)x + \(\frac{5}{7}\)y + \(\frac{17}{7}\) = 0 ⇒ 2x + 5y + 17 = 0

Question 16.
If 2x – 3y – 5 = 0 is the perpendicular bi-sector of the line segment joining (3, – 4) and (α, β) find α + β.
Solution:
(α, β) is the reflection of (3, -4) is the line
2x – 3y – 5 = 0
\(\frac{\alpha-3}{2}\) = \(\frac{\beta+4}{-3}\) = \(\frac{-2(6+12-5)}{4+9}\) = -2
α – 3 = -4 ⇒ α = -1
β + 4 = 6 ⇒ β = 2
α + β = -1 + 2 = 1

Inter 1st Year Maths 1B The Straight Line Important Questions

Question 17.
If the four straight lines ax + by + p = 0, ax + by + q = 0, cx + dy + r = 0 and cx + dy + s = 0 form a parallelogram, show that the area of the parallelogram so formed is \(\left|\frac{(p-a)(r-s)}{b c-a d}\right|\).
Solution:
Let L1, L2, L3, L4 be the lines given by
L1 = ax + by + p = 0
L2 = ax + by + q = 0
L3 = cx + dy + r = 0
L4 = cx + dy + s = 0
L1 and L2 are parallel: L3 and L4 are parallel
Area of the parallelogram = \(\frac{\mathrm{d}_{1} \mathrm{~d}_{2}}{\sin \theta}\)
d1 = distance between L1 and L2 = \(\frac{|p-a|}{\sqrt{a^{2}+b^{2}}}\)
d2 = distance between L3 and L4 = \(\frac{|r-s|}{\sqrt{c^{2}+d^{2}}}\)
Inter 1st Year Maths 1B The Straight Line Important Questions 9

Question 18.
The hypotenuse of a right angled isosceles triangle has its ends at the points (1, 3) and (-4, 1). Find the equations of the legs of the triangle.
Sol:
LetA =(1, 3)and B = (-4, 1) and ABC b a right isosceles triangle with \(\stackrel{\leftrightarrow}{A B}\) as hypotenuse.
We require, therefore, the equations of
\(\stackrel{\leftrightarrow}{A C}\) and \(\stackrel{\leftrightarrow}{B C}\)
Slope of \(\stackrel{\leftrightarrow}{A B}\) is \(\frac{1-3}{-4-1}\) = \(\frac{2}{5}\)
Since the slope of \(\stackrel{\leftrightarrow}{A B}\) is \(\), neither \(\frac{2}{5}\) nor \(\stackrel{\leftrightarrow}{A C}\) is vertical.
Inter 1st Year Maths 1B The Straight Line Important Questions 10
Taking the slope of \(\stackrel{\leftrightarrow}{A C}\) as \(\frac{7}{3}\), the slope of \(\stackrel{\leftrightarrow}{B C}\) would be –\(\frac{3}{7}\). Therefore, the equations of \(\stackrel{\leftrightarrow}{A C}\) and \(\stackrel{\leftrightarrow}{B C}\) are respectively.
y – 3 = \(\frac{7}{3}\) (x – 1) and y – 1 = – \(\frac{3}{7}\) (x + 4),
which become 7x – 3y + 2 = 0 and 3x + 7y + 5 = 0
If the lines drawn through A and B respectively parallel to \(\stackrel{\leftrightarrow}{B C}\) and \(\stackrel{\leftrightarrow}{A C}\) meet at D, then ∆ABD is also right isosceles, having \(\stackrel{\leftrightarrow}{A B}\) as its hypotenuse.
Therefore, the equations of \(\stackrel{\leftrightarrow}{A D}\) and \(\stackrel{\leftrightarrow}{B D}\) are respectively,
3(x – 1) + 7(y – 3) = 0 and 7(x + 4) – 3(y – 1) = 0
⇒ 3x + 7y – 24 = 0 and 7x – 3y + 31 = 0.
Therefore, the two pairs of legs required are
7x – 3y + 2 = 0, 3x + 7y + 5 = 0 and
3x + 7y – 24 = 0, 7x – 3y + 31 = 0.
Note : ADBC is square.

Inter 1st Year Maths 1B The Straight Line Important Questions

Question 19.
A line is such that its segment between the lines 5x – y + 4 = 0 and 3x + 4y – 4 = 0 is . bisected at the point (1, 5). Obtain its equation.
Solution:
Let the required line meet 3x + 4y – 4 = 0 at A and 5x – y + 4 = 0 at B, so that AB is the segment between the given lines, with its mid-point at C = (1, 5).
The equation 5x – y + 4 = 0 can be written as
y = 5x + 4 so that any point on \(\stackrel{\leftrightarrow}{B X}\) is (t, 5t + 4) for all real t.
∴ B = (t, 5t + 4) for some t. Since (1, 5) is the mid-point of \(\stackrel{\leftrightarrow}{A B}\)
A = [2 – t, 10 – (5t + 4)]
= [2 – t, 6 – 5t]
Since A lies on 3x + 4y – 4 = 0,
3(2 – 1) + 4(6 – 5t) – 4 = 0
⇒ -23t + 26 = 0
⇒ t = \(\frac{26}{23}\)
Inter 1st Year Maths 1B The Straight Line Important Questions 11
equation of \(\stackrel{\leftrightarrow}{A B}\) is y – 5 = \(\frac{107}{3}\) (x – 1)
⇒ 3y – 15 = 107x – 107
⇒ 107x – 3y – 92 = 0.

Question 20.
An equilateral triangle has its incentre at the origin and one side as x + y – 2 = 0. Find the vertex opposite to x + y – 2 = 0.
Solution:
Let ABC be the equilateral triangle and
x + y – 2 = 0 represent side \(\stackrel{\leftrightarrow}{B C}\).
Since O is the incentre of the triangle, \(\stackrel{\leftrightarrow}{A D}\) is the bisector of ∠BAC .
Since the triangle is equilateral, \(\stackrel{\leftrightarrow}{A D}\) is the perpendicular bisector of \(\stackrel{\leftrightarrow}{B C}\).
Since O is also the centroid, AO : OD = 2 : 1. [The centroid, circumcentre incentre and orthocentre coincide]
Let D = (h, k).
Since D is the foot of the perpendicular from O onto \(\stackrel{\leftrightarrow}{B C}\), D is given by
Inter 1st Year Maths 1B The Straight Line Important Questions 12
∴ x1 = -2, y1 = -2.
∴ A = (-2, -2), the required vertex.

Inter 1st Year Maths 1B The Straight Line Important Questions

Question 21.
Find the orthocentre of the triangle whose vertices are (-5,-7) (13, 2) and (-5, 6)
Solution:
Let A(-5, -7), B(13,2) and C (-5, 6) be the vertices of a triangle. Let \(\stackrel{\leftrightarrow}{A D}\) be the perpendicular drawn from A to \(\stackrel{\leftrightarrow}{B C}\) and \(\stackrel{\leftrightarrow}{B E}\) be the perpendicular drawn from B to \(\stackrel{\leftrightarrow}{A C}\).
Inter 1st Year Maths 1B The Straight Line Important Questions 13
Now slope of \(\stackrel{\leftrightarrow}{B C}\) = \(\frac{6-2}{-5-13}\) = \(\frac{-2}{9}\)
Since \(\stackrel{\leftrightarrow}{A D}\) ⊥ \(\stackrel{\leftrightarrow}{B C}\) , slope of \(\stackrel{\leftrightarrow}{A D}\) = \(\frac{9}{2}\) and so, the equation of \(\stackrel{\leftrightarrow}{A D}\) is
9x – 2y = – 45 + 14 = -31 ………………… (1)
Equation of \(\stackrel{\leftrightarrow}{A C}\) is x = -5 which is a vertical line and therefore equation of \(\stackrel{\leftrightarrow}{B E}\) is y = 2. ……………… (2)
Point of intersection of the lines (1) and (2) is (-3, 2) which is the orthocentre of ∆ ABC.

Question 22.
If the equations of the sides of a triangle are 7x + y – 10 = 0, x – 2y + 5 = 0 and x + y + 2 = 0, find the orthocentre of the triangle.
Solution:
Let ∆ ABC be the given triangle
Let the equations x – 2y + 5 = 0 ……………… (1)
7x + y – 10 = 0 ……………… (2)
and x + y + 2 = 0 ……………….. (3)
represent the sides \(\overleftrightarrow{\mathrm{AB}}\), \(\overleftrightarrow{\mathrm{BC}}\), and \(\overleftrightarrow{\mathrm{CA}}\)
Inter 1st Year Maths 1B The Straight Line Important Questions 14
Let \(\overleftrightarrow{\mathrm{AD}}\) and \(\overleftrightarrow{\mathrm{BE}}\) be the altitudes drawn from A and B respectively to the sides \(\overleftrightarrow{\mathrm{BC}}\) & \(\overleftrightarrow{\mathrm{CA}}\)
Solving the equations (1) and (3), we obtain A =(-3,1).
Since \(\overleftrightarrow{\mathrm{AD}}\) ⊥ \(\overleftrightarrow{\mathrm{BC}}\), the equations of \(\overleftrightarrow{\mathrm{AD}}\) is
x – 7y = -3 – 7 = -10 …………………… (4)
Solving the equation (1) and (2), we obtain B = (1, 3).
Since \(\overleftrightarrow{\mathrm{BE}}\) ⊥ \(\overleftrightarrow{\mathrm{AC}}\) , the equation of BE is
x – y = 1 – 3 = -2 …………… (5)
Point of intersection of the lines (4) and (5) is H(\(\frac{-2}{3}\), \(\frac{4}{3}\)) which is the orthocentre of ∆ ABC.

Inter 1st Year Maths 1B The Straight Line Important Questions

Question 23.
Find the circumcentre of the triangle whose vertices are (1, 3), (-3, 5) and (5, -1).
Solution:
Let the vertices of the triangle be
A(1, 3), B(-3, 5) and (5,-1).
The midpoints of the sides \(\overline{\mathrm{BC}}\), \(\overline{\mathrm{CA}}\) are respectively D(1, 2) and E(3, 1).
Let S be the point of intersection of the perpendicular bisectors of the sides \overline{\mathrm{BC}} and \overline{\mathrm{CA}}
Inter 1st Year Maths 1B The Straight Line Important Questions 15
Since the slope of \(\overleftrightarrow{\mathrm{BC}}\) = \(\frac{5+1}{-3-5}\) = \(\frac{-3}{4}\), the slope \(\overleftrightarrow{\mathrm{SD}}\) is \(\frac{4}{3}\) and therefore its equation is 4x – 3y = 4 – 6 = -2 …………….. (1)
Slope of \(\overleftrightarrow{\mathrm{AC}}\) = \(\frac{3+1}{1-5}\) = -1 ⇒ Slope of –\(\sqrt{3x}\) = 1
∴ Equation of \(\overleftrightarrow{\mathrm{SE}}\) is x – y = 3 – 1 = 2 ………………….. (2),
Solving the equations (1) and (2),we obtain S = (-8, -10) which is the circumcentre of ∆ ABC.

Question 24.
Find the circumcentre of the triangle whose sides are 3x – y – 5 = 0, x + 2y – 4 = 0 and 5x + 3y + 1 = 0. [May 11, 05; Mar. 06]
Solution:
Let the given equations 3x – y – 5 = 0, x + 2y – 4 = 0 and 5x + 3y + 1 = 0 represent the sides \(\overleftrightarrow{\mathrm{BC}}\), \(\overleftrightarrow{\mathrm{CA}}\) and \(\overleftrightarrow{\mathrm{AB}}\) respectively of ∆ ABC.
Solving the above equations two by two, we obtain the vertices A(-2, 3), B(1, -2) and (2, 1) of the given triangle.
The midpoints of the sides \(\overline{\mathrm{BC}}\) and \(\overline{\mathrm{CA}}\) are respectively D = (\(\frac{3}{2}\), \(\frac{-1}{2}\)) and E = (0, 2).
Inter 1st Year Maths 1B The Straight Line Important Questions 16
Equation of \(\overleftrightarrow{\mathrm{SD}}\) the perpendicular bisector of \(\overline{\mathrm{BC}}\) is x + 3y = 0 and the equation of \(\overleftrightarrow{\mathrm{SE}}\) the perpendicular bisector of \(\overline{\mathrm{AC}}\) is 2x – y + 2 = 0. Solving these two equations, we obtain the point of intersection of the lines \(\overleftrightarrow{\mathrm{SD}}\) and \(\overleftrightarrow{\mathrm{SE}}\) which is S(\(\frac{-6}{7}\), \(\frac{2}{7}\)) the circumcentre of ∆ ABC.

Inter 1st Year Maths 1B The Straight Line Important Questions

Question 25.
Find the incentre of the triangle formed by the straight lines y = \(\sqrt{3}\)x, y = –\(\sqrt{3}\)x and y = 3.
Solution:
Inter 1st Year Maths 1B The Straight Line Important Questions 17
The straight lines y = \(\sqrt{3x}\) and y = –\(\sqrt{3x}\) respectively make angles 60° and 120° with the positive direction of X – axis.
Since y = 3 is a horizontal line, the triangle formed by the three given lines is equilateral. So in-centre is same and centroid.
Vertices of the triangle and O(0, 0), A(\(\sqrt{3}\), 3) and D(\(\sqrt{3}\), 3).
∴ Incentre is (\(\frac{0+\sqrt{3}-\sqrt{3}}{3}\), \(\frac{0+3+3}{3}\)) = (0, 2)

Question 26.
Find the equation of the straight line whose distance from the origin is 4, If the normal ray from the origin to the straight line makes an angle of 135° with the positive direction of the x-axis.
Solution:
The equation of the given line is
x cos α + y sin α = p where p = 4 and α = 135°
∴ x(\(\frac{-1}{\sqrt{2}}\)) + y(\(\frac{1}{\sqrt{2}}\)) = 4 ⇒ or x – y + 4\(\sqrt{2}\) = 0

(i). Transform the equation x + y + 1 = 0 in to normal from.
Solution:
x + y + 1 = 0
⇔ (\(\frac{-1}{\sqrt{2}}\))n + (\(\frac{-1}{\sqrt{2}}\)) y = \(\frac{1}{\sqrt{2}}\)
⇔ x cos \(\frac{5\pi}{42}\) + y sin \(\frac{5\pi}{4}\) = \(\frac{1}{\sqrt{2}}\)
Hence the normal form of the equation of the given straight line is x cos \(\frac{5\pi}{4}\) + y sin \(\frac{5\pi}{4}\) = \(\frac{1}{\sqrt{2}}\) and the distance of the line from the origin is = \(\frac{1}{\sqrt{2}}\)

ii) A straight line passing through A(1, -2) makes and angle Tan-1\(\frac{4}{3}\) with the positive direction of the x – axis in the anti-clock wise sense. Find the points of the straight line whose distance from A is 5.
Solution:
The paramatic equation of the lines A (1, -2)
Slope is \(\frac{4}{3}\) (∵ tan θ = \(\frac{4}{3}\))
are x = 1 + r cos θ = 1 + r(\(\frac{3}{5}\)) and y
= -2 + r sinθ
= -2 + r (\(\frac{4}{5}\))
The points on the above line a + a distance of (r) = 5 correspond to r = ±5 in the above equation and are
∴ (4, 2) and (-2, -6)

Inter 1st Year Maths 1B The Straight Line Important Questions

Question 27.
Trans form the equation 3x + 4y + 12 = 0 in to (i) slope – intercept from (ii) intercept form (iii) normal form.
Solution:
(i) slop – intercept form
3x + 4y + 12 = 0
⇔ 4y = -3x – 12
⇔ y = (\(\frac{-3}{4}\)) x + (-3)
∴ Slope = (\(\frac{-3}{4}\)) and y – intercept = -3,

ii) Intercept form :
3x = 4y + 12 = 0
⇔ \(\frac{-3y}{12}\) – \(\frac{-4y}{12}\) = 1
⇔ \(\frac{x}{(-4)}\) + \(\frac{y}{(-3)}\) = 1
∴ x-intercept of the line is -4, and tan y-intercept is -3.

iii) Normal form
3x = 4y + 12 = 0
⇔ -3x + -4y – 12 = 0
⇔ (\(\frac{-3}{5}\))x + (\(\frac{-4}{5}\))y = \(\frac{12}{5}\)
⇔ x cos α + y sin α = p ⇒ p\(\frac{12}{5}\) and cos α = \(\frac{-3}{5}\), sin α = \(\frac{-4}{5}\) determine the angle α in (0, 2π).

Question 28.
Find the angle between the lines 2x + y + 4 = 0 and y – 3x = 7.
Solution:
The angle between the given lines
= cos-1 \(\frac{-6+1}{\sqrt{5 \times 10}}\)
= cos-1 [latex]\frac{5}{\sqrt{2}}[/latex] = cos-1(\(\frac{1}{\sqrt{2}}\)) = \(\frac{\pi}{4}\)

Inter 1st Year Maths 1B The Straight Line Important Questions

Question 29.
Find Q(h,k) in the foot of the perpendicular from p(x1, y1) on the straight lines ax + by + c = 0 then (h – x1) ; a = (k – y1); b = -(ax1 + by1 + c); (a2 + b2)
Solution:
Equation of \(\stackrel{\leftrightarrow}{P Q}\) which is normal to the given straight line
L : ax + by + c = 0
bx – ay = bx1 – ay1
Inter 1st Year Maths 1B The Straight Line Important Questions 18
∴ Q ∈ \(\stackrel{\leftrightarrow}{P Q}\) we have
bh – ak = bx1 – ay1
∴ b(h – x1) = a(k – y1)
or (h – x1) a = (k – y1); b.
But, this implus that h = aλ + x1, and k = bλ + y1
for some λ ∈ R. sine Q(h, k) in point on L.
a(aλ + x1) + b(bλ + y1) + c = 0
i.e., λ = \(\frac{\left(a x_{1}+b y_{1}+c\right.}{\left(a^{2}+b^{2}\right)}\)
∴ (h – x1) ; a = (k – y1); b
= – (ax1 + by1 + c); (a2 + b2)

Question 30.
Find the distance between the parallel straight lines 3x + 4y – 3 = 0 and 6x + 8y -1 = 0.
Solution:
The equation of the given straight lines is 3x + 4y – 3 = 0 and 6x + 8y – 1 = 0.
formula : \(\frac{c_{1}-c_{2}}{\sqrt{a^{2}+b^{2}}}\) = \(\frac{-6+1}{\sqrt{6^{2}+8^{2}}}\) = \(\frac{5}{10}\) = \(\frac{1}{2}\)

Question 31.
Find the condition for the points (a, o) (h, k) and (o, b) when ab ≠ 0 to be collinear. [Mar 10]
Solution:
The equation of the line passing through (a, o) and (o, b) is \(\frac{x}{a}\) + \(\frac{y}{b}\) = 1
The given points are collinear ⇒ (h, k) lies on the above line ⇒ \(\frac{h}{a}\) + \(\frac{k}{b}\) = 1
⇒ hb + ka = ab

Inter 1st Year Maths 1B The Straight Line Important Questions

Question 32.
Find the area of the triangle formed by the straight lines x cos a + y sin a = p and the axes of co-ordinates. [Mar 10]
Solution:
The area of the triangle formed by the line ax + by + c = 0
and the co-ordinate axes is \(\frac{c^{2}}{2|a b|}\)
∴ Area of the triangle = \(\frac{p^{2}}{2|\cos \alpha \cdot \sin \alpha|}\)
= \(\frac{p^{2}}{|\sin 2 \alpha|}\)

Inter 1st Year Maths 1A Trigonometric Ratios up to Transformations Important Questions

Inter 1st Year Maths 1A Trigonometric Ratios up to Transformations Important Questions

Students get through Maths 1A Important Questions Inter 1st Year Maths 1A Trigonometric Ratios up to Transformations Important Questions which are most likely to be asked in the exam.

Intermediate 1st Year Maths 1A Trigonometric Ratios up to Transformations Important Questions

Question 1.
Find the values of
(i) sin \(\frac{5 \pi}{3}\)
(ii) tan (855°)
(iii) sec \(\left(\frac{13 \pi}{3}\right)\)
Solution:
i) sin \(\frac{5 \pi}{3}\) = sin \(\left(2 \pi-\frac{\pi}{3}\right)\) = -sin \(\frac{\pi}{3}\) = \(-\frac{\sqrt{3}}{2}\)

ii) tan (855°) = tan (2 × 360° + 135°)
= tan (135°)
= tan (180° – 45°)
= -tan 45° = -1

iii) sec \(\left(13 \frac{\pi}{3}\right)\) = sec \(\left(4 \pi+\frac{\pi}{3}\right)\)
= sec \(\frac{\pi}{3}\) = 2

Question 2.
Simplify.
i) Cot (θ – \(\frac{13 \pi}{2}\))
ii) tan \(\left(-23 \frac{\pi}{3}\right)\)
Solution:
Inter 1st Year Maths 1A Trigonometric Ratios up to Transformations Important Questions 1

Inter 1st Year Maths 1A Trigonometric Ratios up to Transformations Important Questions

Question 3.
Find the value of sin2 \(\frac{\pi}{10}\) + sin2 \(\frac{4 \pi}{10}\) + sin2 \(\frac{6 \pi}{10}\) + sin2 \(\frac{9 \pi}{10}\)
Solution:
Inter 1st Year Maths 1A Trigonometric Ratios up to Transformations Important Questions 2
Inter 1st Year Maths 1A Trigonometric Ratios up to Transformations Important Questions 3

Question 4.
If sin θ = \(\frac{4}{5}\) and θ is not in the first qua-drant, find the value of cos θ.
Solution:
∵ sin θ = \(\frac{4}{5}\) and θ is not in the first quadrant
⇒ θ lies in 2nd quadrant, ∵ sin θ is +ve
Inter 1st Year Maths 1A Trigonometric Ratios up to Transformations Important Questions 4

Question 5.
If sec θ + tan θ = \(\frac{2}{3}\), find the value of sin θ and determine the quadrant in which θ lies.
Solution:
∵ sec θ + tan θ = \(\frac{2}{3}\)
Inter 1st Year Maths 1A Trigonometric Ratios up to Transformations Important Questions 6
∵ tan θ is negative, sec θ is positive
⇒ θ lies in fourth quadrant.

Question 6.
Prove that
cot\(\frac{\pi}{16}\).cot\(\frac{2 \pi}{16}\).cot\(\frac{3 \pi}{16}\)…..cot\(\frac{7 \pi}{16}\) = 1
Solution:
Inter 1st Year Maths 1A Trigonometric Ratios up to Transformations Important Questions 7
Inter 1st Year Maths 1A Trigonometric Ratios up to Transformations Important Questions 8

Inter 1st Year Maths 1A Trigonometric Ratios up to Transformations Important Questions

Question 7.
If 3 sin θ + 4 cos θ = 5, then find the value of 4 sin θ – 3 cos θ.
Solution:
Given 3 sin θ + 4 cos θ = 5 and let 4 sin θ – 3 cos θ = x
By squaring and adding, we get
(3 sin θ + 4 cos θ)2 + (4 sin θ – 3 cos θ)2 = 52 + x2
⇒ 9 sin2 θ + 16 cos2 θ + 24 sin θ cos θ + 16 sin2 θ + 9 cos2 θ – 24 sin θ cos θ = 25 + x2
⇒ 9 + 16 = 25 + x2
⇒ x2 = 0 ⇒ x = 0
∴ 4 sin θ – 3 cos θ = 0.

Question 8.
If cos θ + sin θ = \(\sqrt{2}\) cos θ, prove that cos θ – sin θ = \(\sqrt{2}\) sin θ. (May ’11)
Solution:
Given cos θ + sin θ = \(\sqrt{2}\) cos θ
⇒ sin θ = (\(\sqrt{2}\) – 1) cos θ
Now multiplying both sides by (\(\sqrt{2}\) + 1)
Inter 1st Year Maths 1A Trigonometric Ratios up to Transformations Important Questions 9

Question 9.
Find the value of 2(sin2 θ + cos6 θ) – 3(sin4 θ + cos4 θ)
Solution:
2 (sin6 θ + cos6 θ) – 3(sin4 θ + cos4 θ)
= 2[(sin2 θ)3 + (cos2 θ)3] – 3[(sin2 θ)2 + (cos2 θ)2]
= 2[(sin2 θ + cos2 θ)3 – 3 sin2 θ cos2 θ (sin2 θ + cos2 θ)] – 3[(sin2 θ + cos2 θ)2 – 2 sin2 θ cos2 θ]
= 2[1 – 3 sin2 θ cos2 θ] – 3[1 – 2 sin2 θ cos2 θ]
= 2 – 6 sin2 θ cos2 θ – 3 + 6 sin2 θ cos2 θ
= -1

Question 10.
Prove that (tan θ + cot θ)2 = sec2 θ + cosec2 θ = sec2 θ cosec2 θ.
Solution:
(tan θ + cot θ)2
=tan2 θ + cot 2 θ + 2 tan θ cot θ
= tan2 θ + cot2 θ + 2
= (1 + tan2 θ) + (1 + cot2 θ)
= sec2 θ + cosec2 θ
Inter 1st Year Maths 1A Trigonometric Ratios up to Transformations Important Questions 10

Question 11.
If cos θ > 0, tan θ + sin θ = m and tan θ – sin θ = n, then show that m2 – n2 = n, then show that m2 – n2 = 4\(\sqrt{m n}\)
Solution:
Given that m = tan θ + sin θ
n = tan θ – sin θ
⇒ m + n = 2 tan θ and m – n = 2 sin θ
Now(m + n)(m – n) = 4 tan θ sin θ
Inter 1st Year Maths 1A Trigonometric Ratios up to Transformations Important Questions 11

Question 12.
If tan 20° = λ, then show that \(\frac{\tan 160^{\circ}-\tan 110^{\circ}}{1+\tan 160^{\circ} \cdot \tan 110^{\circ}}\) = \(\frac{1-\lambda^{2}}{2 \lambda}\) (A.P.) Mar. ’16
Solution:
Given tan 20° = λ
Inter 1st Year Maths 1A Trigonometric Ratios up to Transformations Important Questions 12

Question 13.
Find the values of sin 75°, cos 75°, tan 75° and cot 75°.
Solution:
i) sin 75° = sin (45° + 30°)
= sin 45°. cos 30° + cos 45°. sin 30°
Inter 1st Year Maths 1A Trigonometric Ratios up to Transformations Important Questions 13

Inter 1st Year Maths 1A Trigonometric Ratios up to Transformations Important Questions

ii) cos (75°) = cos (45° + 30°)
= cos 45° cos 30° – sin 45° sin 30°
Inter 1st Year Maths 1A Trigonometric Ratios up to Transformations Important Questions 14

Inter 1st Year Maths 1A Trigonometric Ratios up to Transformations Important Questions 16

Question 14.
If 0 < A, B < 90°.cos A = \(\frac{5}{13}\) and sin B = \(\frac{4}{5}\)-then find sin(A + B).
Solution:
0 < A < 90°and cos A = \(\frac{5}{13}\) ⇒ sin A = \(\frac{12}{13}\)
0 < B < 90°and sin B = \(\frac{4}{5}\) ⇒ cos B = \(\frac{3}{5}\)
∴ sin (A + B) = sin A cos B + cos A sin B
= \(\frac{12}{13}\) • \(\frac{3}{5}\) + \(\frac{5}{13}\) • \(\frac{4}{5}\) = \(\frac{56}{65}\)

Question 15.
Prove that
sin2 \(\left(52 \frac{1}{2}\right)^{\circ}\) – sin2 \(\left(22 \frac{1}{2}\right)^{\circ}\) = \(\frac{\sqrt{3}+1}{4 \sqrt{2}}\)
Solution:
Inter 1st Year Maths 1A Trigonometric Ratios up to Transformations Important Questions 17

Question 16.
Prove that tan 70° – tan 20° = 2 tan 50°.
Solution:
tan 50° = tan (70° – 20°)
= \(\frac{\tan 70^{\circ}-\tan 20^{\circ}}{1+\tan 20^{\circ} \cdot \tan 70^{\circ}}\)
⇒ tan 70° – tan 20°
= tan 50° [1 + tan 20° . tan (90° – 20°)]
= tan 70° – tan 20° = tan 50°[1 + tan 20° cot 20°]
⇒ tan 70° – tan 20° = 2 tan 50°.

Question 17.
If A + B = π/4, then prove that
i)(1 + tan A)(1 + tan B) = 2. (May ’11; Mar. ’07)
ii) (cot A – 1)(cot B – 1) = 2
Solution:
i) Given that A + B = π/4 (T.S) (Mar. ’16)
⇒ tan (A + B) = tan (π/4)
⇒ \(\frac{\tan A+\tan B}{1-\tan A \tan B}\) = 1
⇒ tan A + tan B = 1 – tan A tan B
⇒ tan A + tan B + tan A tan B = 1
Add 1 on both sides
⇒ 1 + tan A + tan B + tan A tan B = 1 + 1
⇒ (1 + tan A)(1 + tan B) = 2

ii) Given A + B = π/4
⇒ cot (A + B) = cot π/4
⇒ \(\frac{\cot A \cot B-1}{\cot B+\cot A}\) = 1
⇒ cot A cot B – 1 = çot B + cot A
⇒ cot A cot B – cot A – cot B = 1
Again add 1 on both sides
cot A cot B – cot A – cot B + 1 = 1 + 1
∴ (cot A – 1) (cot B – 1) = 2.

Question 18.
If sin α = \(\frac{1}{\sqrt{10}}\), sin β = \(\frac{1}{\sqrt{5}}\) and α, β are acute, show that α + β = π/4.
Solution:
Given α is acute and sin α = \(\frac{1}{\sqrt{10}}\)
⇒ tan α = \(\frac{1}{3}\)
β is acute and sin β = \(\frac{1}{\sqrt{5}}\) ⇒ tan β = \(\frac{1}{2} .\)
Inter 1st Year Maths 1A Trigonometric Ratios up to Transformations Important Questions 18

Question 19.
If sin A = \(\frac{12}{13}\), cos B = \(\frac{3}{5}\) and neither A nor B is in the first quadrant, then find the quadrant in which A + B lies.
Solution:
sin A = \(\frac{12}{13}\) and A is not in first quadrant
⇒ A lies in second quadrant, ∵ sin A is +ve
cos B = \(\frac{3}{5}\) and B is not in first quadrant
⇒ B lies in fourth quadrant, cos B is +ve
∵ sin A = \(\frac{12}{13}\) ⇒ cos A = –\(\frac{5}{13}\)
cos B = \(\frac{3}{5}\) ⇒ sin B = –\(\frac{4}{5}\)
sin (A + B) = sin A cos B + cos A sin B
Inter 1st Year Maths 1A Trigonometric Ratios up to Transformations Important Questions 19
cos (A + B) = cos A cos B – sin A sin B
Inter 1st Year Maths 1A Trigonometric Ratios up to Transformations Important Questions 20
∵ sin (A + B) and cos (A + B) are positive
⇒ (A + B) lies in first quadrant.

Question 20.
Find (i) tan \(\left(\frac{\pi}{4}+\mathbf{A}\right)\) interms of tan A
and (ii) cot \(\left(\frac{\pi}{4}+\mathbf{A}\right)\) interms of cot A.
Solution:
Inter 1st Year Maths 1A Trigonometric Ratios up to Transformations Important Questions 21

Inter 1st Year Maths 1A Trigonometric Ratios up to Transformations Important Questions

Question 21.
Prove that \(\frac{\cos 9^{\circ}+\sin 9^{\circ}}{\cos 9^{\circ}-\sin 9^{\circ}}\) = cot 36°. (A.P) (Mar ’15, Mar. ’11)
Solution:
Inter 1st Year Maths 1A Trigonometric Ratios up to Transformations Important Questions 22

Question 22.
Show that
cos 42° + cos 78° + cos 162° = 0 (May. ’11)
Solution:
L.H.S. = cos 42° + cos 78° + cos 162°
= 2 cos \(\left(\frac{42^{\circ}+78^{\circ}}{2}\right)\). cos \(\left(\frac{42^{\circ}-78^{\circ}}{2}\right)\) + cos (180° – 18°)
= 2 cos 60° . cos (-18°) + cos (180° – 18°)
= 2\(\left(\frac{1}{2}\right)\) cos 18° – cos 18° = 0
∴ cos 42° + cos 78° + cos 162° = 0

Question 23.
Express \(\sqrt{3}\) sin θ + cos θ as a sine of an angle.
Solution:
\(\sqrt{3}\) sin θ + cos θ = 2(\(\frac{\sqrt{3}}{2}\) sin θ + cos θ)
= 2(cos \(\frac{\pi}{6}\) sin θ + sin \(\frac{\pi}{6}\) cos θ)
= 2. sin[θ + \(\frac{\pi}{6}\)]

Question 24.
Prove that sin2 θ + sin2 [θ – \(\frac{\pi}{3}\)] + sin2 [θ – \(\frac{\pi}{3}\)] = \(\frac{3}{2} .\)
Solution:
Inter 1st Year Maths 1A Trigonometric Ratios up to Transformations Important Questions 23

Question 25.
If A, B, C are angles of a triangle and if none of them is equal to \(\frac{\pi}{2}\), then prove that
i) tan A + tan B + tan C = tan A tan B tan C
ii) cot A cot B + cot B cot C + cot C cot A = 1
Solution:
i) ∵ A, B, C are angles of a triangle
⇒ A + B + C = 180°
⇒ A + B = 180° – C
⇒ tan (A + B) = tan (180° – C)
⇒ \(\frac{\tan A+\tan B}{1-\tan A \tan B}\) = -tan C
⇒ tan A + tan B = -tan C + tan A tan Btan C
⇒ tan A + tan B + tan C = tan A tan B tan C

ii) ∵ A + B + C = 180°
⇒ A + B = 180° – C
⇒ cot (A + B) = cot (180° – C)
⇒ \(\frac{\cot A \cot B-1}{\cot B+\cot A}\) = -cot C
⇒ cot A cot B – 1 = -cot B cot C – cot C. cot A
⇒ cot A cot B + cot B cot C + cot C cot A = 1

Question 26.
Let ABC be a triangle such that cot A + cot B + cot C = \(\sqrt{3}\). Then prove that ABC is an equilateral triangle.
Solution:
Given that A + B + C = 180°
Then we know that
cot A cot B + cot B cot C + cot C cot A = 1
i.e., Σ(cot A cot B) = 1 —– (1)
Now Σ (cot A – cot B)2
= Σ (cot2 A + cot2 B – 2 cot A cot B)
= 2 cot2 A + 2 cot2 B + 2 cot2 C – 2 cot A cot B – 2 cot B cot C – 2 cot C cot A
= 2 [cot A + cot B + cot C]2 – 6 (cot A cot B + cot B cot C + cot C cot A)
= 2
= 6 – 6 = 0
∴ Σ (cot A – cot B)2 = 0 .
⇒ cot A = cot B = cot C
⇒ cot A = cot B = cot C = \(\frac{\sqrt{3}}{3}\) = \(\frac{1}{\sqrt{3}}\)
(∵ cot A + cot B + cot C = \(\sqrt{3})\))
⇒ A = B = C = 60°
∴ ΔABC is an equilateral triangle.

Question 27.
Suppose x = tan A, y = tan B, z = tan C. Suppose none of A, B, C, A – B, B – C, C – A is an odd multiple of \(\frac{\pi}{2}\).Then prove that \(\sum\left(\frac{x-y}{1+x y}\right)\) = \(\pi\left(\frac{x-y}{1+x y}\right)\)
Solution:
∵ x = tan A, y = tan B, z = tan C
Inter 1st Year Maths 1A Trigonometric Ratios up to Transformations Important Questions 24
Write P = A – B, Q = B – C, R = C – A.
Then P + Q + R = O
⇒ tan (P + Q) = tan (-R)
⇒ \(\frac{\tan P+\tan Q}{1-\tan P \tan Q}\) = -tan R
⇒ tan P + tan Q = -tan R + tan P tan Q tan R
⇒ tan P + tan Q + tan R = tan P tan Q tan R
⇒ Σ(tan P) = π (tan P)
⇒ Σtan (A – B) = π tan (A – B)
∴ Σ\(\left(\frac{x-y}{1+x y}\right)\) = π\(\left(\frac{x-y}{1+x y}\right)\)

Question 28.
Find the values of
i) sin 22\(\frac{1}{2}\)°
ii) cos 22\(\frac{1}{2}\)°
iii) tan 22\(\frac{1}{2}\)°
iv) cot 22\(\frac{1}{2}\)°
Solution:
Inter 1st Year Maths 1A Trigonometric Ratios up to Transformations Important Questions 25
Inter 1st Year Maths 1A Trigonometric Ratios up to Transformations Important Questions 26

Question 29.
Find the values of
i) sin 67\(\frac{1}{2}\)°
ii) cos 67\(\frac{1}{2}\)°
iii) tan 67\(\frac{1}{2}\)°
iv) cot 67\(\frac{1}{2}\)°
Solution:
Let A = 67\(\frac{1}{2}\)° ⇒ = 2A = 135°
Inter 1st Year Maths 1A Trigonometric Ratios up to Transformations Important Questions 27
Inter 1st Year Maths 1A Trigonometric Ratios up to Transformations Important Questions 28

Inter 1st Year Maths 1A Trigonometric Ratios up to Transformations Important Questions

Question 30.
Simplify: \(\frac{1-\cos 2 \theta}{\sin 2 \theta}\)
Solution:
\(\frac{1-\cos 2 \theta}{\sin 2 \theta}\) = \(\frac{2 \sin ^{2} \theta}{2 \sin \theta \cos \theta}\) = \(\frac{\sin \theta}{\cos \theta}\) = tan θ

Question 31.
If cos A = \(\sqrt{\frac{\sqrt{2}+1}{2 \sqrt{2}}}\), find the value of cos 2A.
Solution:
cos 2A = 2 cos2A – 1 = 2 \(\left(\frac{\sqrt{2}+1}{2 \sqrt{2}}\right)\) – 1
= \(\frac{\sqrt{2}+1}{\sqrt{2}}\) – 1 = \(\frac{1}{\sqrt{2}}\)

Question 32.
If cos θ = \(\frac{-5}{13}\) and \(\frac{\pi}{2}\) < θ < π, find the value of sin 2θ.
Solution:
\(\frac{\pi}{2}\) < θ < π ⇒ sin θ > 0 and cos θ = –\(\frac{5}{13}\)
⇒ Sin θ = \(\frac{12}{13}\)
∴ sin 2θ = 2 sin θ cos θ
= 2 . \(\frac{12}{13}\)(-\(\frac{5}{13}\)) = –\(\frac{120}{169}\)

Question 33.
For what values of x in the first quadrant \(\frac{2 \tan x}{1-\tan ^{2} x}\) is positive?
Solution:
\(\frac{2 \tan x}{1-\tan ^{2} x}\) > 0 ⇒ tan 2x > 0
⇒ 0 < 2x < \(\frac{\pi}{2}\) (since x is in the first quadrant)
⇒ 0 < x < \(\frac{\pi}{4}\)

Question 34.
If cos θ = \(\frac{-3}{5}\) and π < θ < \(\frac{3 \pi}{2}\), find the value of tan θ/2.
Solution:
Inter 1st Year Maths 1A Trigonometric Ratios up to Transformations Important Questions 29

Question 35.
If A is not an integral multiple of \(\frac{\pi}{2}\), prove that
i) tan A + cot A = 2 cosec 2A
ii) cot A – tan A = 2 cot 2A
Solution:
i) tan A + cot A = \(\frac{\sin A}{\cos A}\) + \(\frac{\cos A}{\sin A}\)
= \(\frac{\sin ^{2} A+\cos ^{2} A}{\sin A \cos A}\)
Inter 1st Year Maths 1A Trigonometric Ratios up to Transformations Important Questions 30

Question 36.
If A is not an integral multiple of prove that
i) tan A + cot A = 2 cosec 2A
ii) cot A – tan A = 2 cot 2A
Solution:
Inter 1st Year Maths 1A Trigonometric Ratios up to Transformations Important Questions 31

Question 37.
If θ is not an integral multiple of \(\frac{\pi}{2}\), prove that tan θ + 2 tan 2θ + 4 tan 4θ +
8 cot 8θ = cot θ.
Solution:
From cot A – tan A = 2 cot 2A above tan A = cot A – 2 cot 2A ….(1)
∴ tan θ + 2 tan 2θ + 4 tan 4θ + 8 cot 8θ = (cot θ – 2 cot 2θ) + 2 (cot 2θ – 2 cot 4θ) + 4 (cot 4θ – 2 cot 8θ) + 8 cot 8θ (by (1) above)
= cot θ

Question 38.
For A ∈ R, prove that
i) sin A.sin(π/3 +A).sin(π/3 – A) = \(\frac{1}{4}\) . sin3A
ii) cosA . cos (π/3 + A). cos(π/3 – A)
= \(\frac{1}{4}\) cos 3A and hence deduce that
iii) sin 20° sin 40° sin 60° sin 80° = \(\frac{3}{16}\)
iv) cos \(\frac{\pi}{9}\), cos \(\frac{2 \pi}{9}\), cos \(\frac{3 \pi}{9}\). cos \(\frac{4 \pi}{9}\) = \(\frac{1}{16}\).
Solution:
i) sin A. sin (π/3 + A). sin (π/3 – A)
= sin A [sin2 π/3 – sin2 A]
Inter 1st Year Maths 1A Trigonometric Ratios up to Transformations Important Questions 32
Inter 1st Year Maths 1A Trigonometric Ratios up to Transformations Important Questions 33
iii) ∵ sin A. sin (60° + A). sin (60° – A)
= \(\frac{1}{4}\) sin 3A
Put A = 20°
⇒ sin 20°. sin (60° + 20°). sin (60° – 20°)
= \(\frac{1}{4}\). sin(3 × 20°) .
⇒ sin 20°. sin 40°. sin 80° = \(\frac{1}{4}\) sin 60°
Multiplying on both sides with sin 60°
We get, sin 20° sin 40° sin 60° sin 80°
= \(\frac{1}{4}\) sin2 60°
= \(\frac{1}{4}\left(\frac{\sqrt{3}}{2}\right)^{2}[latex] = [latex]\frac{3}{16}\)

iv) ∵ cos A. cos (60° + A). cos (60° – A) = \(\frac{1}{4}\)cos 3A
Put A = 20°
⇒ cos 20°. cos (60° + 20°) cos (60° – 20°)
= \(\frac{1}{4}\). cos(3 × 20°)
⇒ cos 20°. cos 40°. cos 80° = cos 60°
On multiplying both sides by cos 60°, we get cos 20°. cos 40°. cos 60°. cos 80°
= \(\frac{1}{4}\). cos2 60°
Inter 1st Year Maths 1A Trigonometric Ratios up to Transformations Important Questions 34

Question 39.
If 3A is not an odd multiple of \(\frac{\pi}{2}\), prove that tan A. tan (60° + A). tan (60° – A)
= tan 3A and hence find the value of tan 6°. tan 42°. tan 66°. tan 78°.
Solution:
Inter 1st Year Maths 1A Trigonometric Ratios up to Transformations Important Questions 35
∴ tan A . tan \(\left(\frac{\pi}{3}+\mathrm{A}\right)\) tan \(\left(\frac{\pi}{3}-A\right)\) = tan 3A
i.e., tan A tan (60° – A) tan (60° + A) = tan 3A —— (1)
Put A = 6°
⇒ tan 6° tan 54° tan 66° = tan 18° ——- (2)
Put A = 18° in (1)
tan 18° tan 42° tan 78° = tan 54° ——- (3)
put (2) in (3),
(tan 6° tan 54° tan 66°) tan 42° tan 78°
= tan 54
⇒ tan 6° tan 42° tan 66° tan 78° = 1

Question 40.
For α, β ∈ R, prove that (cos α + cos β)2 + (sin α + sin β)2 = 4 cos2 \(\left(\frac{\alpha-\beta}{2}\right)\).
Solution:
LH.S. = (cos α + cos β)2 + (sin α + sin β)2
= (cos2 α + cos2 β + 2 cos α cos β + (sin2 α + sin2 β + 2 sin α sin β)
= 1 + 1 + 2 (cos α cos β + sin α sin β)
= 2 + 2 cos (α – β) = 2 (1 + cos (α – β)]
= 2[2cos2\(\left(\frac{\alpha-\beta}{2}\right)\)]
= 4.cos2\(\left(\frac{\alpha-\beta}{2}\right)\); [∵ 1 + cos θ = 2 cos2 \(\frac{\theta}{2}\)]

Question 43.
If a, b, c are non zero real numbers and α, β are solutions of the equation a cos θ + b sin θ = c then show that
(i) sin α + sin β = \(\frac{2 b c}{a^{2}+b^{2}}\) and
(ii) sin α. sin β = \(\frac{c^{2}-a^{2}}{a^{2}+b^{2}}\)
Solution:
∵ a cos θ + b sin θ = c
= a cos θ = c – b sin θ
⇒ (a cos θ)2 = (c – b sin θ)2
⇒ a2 cos2 θ = c2 + b2 sin2 θ – 2bc sinθ
⇒ a2 (1 – sin2 θ) = c2 + b2 sin2 θ – 2bc sin θ
⇒ (a2 + b2) sin2 θ – 2bc sin θ + (c2 – a2) = 0
This is a quadratic equatioñ in sin θ, whose roots are sin α and sin β (given that α, β are two solutions of θ)
∴ Sum of the roots sin α + sin β = \(\frac{2 b c}{a^{2}+b^{2}}\)
Product of the roots sin α sin β = \(\frac{c^{2}-a^{2}}{a^{2}+b^{2}}\)

Inter 1st Year Maths 1A Trigonometric Ratios up to Transformations Important Questions

Question 42.
If θ is not an odd multiple of \(\frac{\pi}{2}\) and cos θ ≠ \(\frac{-1}{2}\), prove that
\(\frac{\sin \theta+\sin 2 \theta}{1+\cos \theta+\cos 2 \theta}\) = tan θ.
Solution:
Inter 1st Year Maths 1A Trigonometric Ratios up to Transformations Important Questions 36

Question 43.
Prove that sin4 \(\frac{\pi}{8}\) + sin4 \(\frac{3 \pi}{8}\) + sin4 \(\frac{5 \pi}{8}\) + sin4 \(\frac{7 \pi}{8}\) = \(\frac{3}{2}\)
Solution:
Inter 1st Year Maths 1A Trigonometric Ratios up to Transformations Important Questions 37

Question 44.
If none of 2A and 3A is an odd multiple of \(\frac{\pi}{2}\), then prove that tan 3A. tan 2A. tanA = tan 3A – tan 2A – tan A.
Solution:
∵ tan 3A = tan (2A + A)
= \(\frac{\tan 2 A+\tan A}{1-\tan 2 A \tan A}\)
⇒ tan 3A(1 – tan 2A tan A) = tan 2A + tan A
⇒ tan 3A – tan A tan 2A tan 3A
= tan 2A + tan A
∴ tan 3A – tan 2A – tan A
= tan A tan 2A tan 3A

Question 45.
Prove that sin 78° + cos 132° = \(\frac{\sqrt{5}-1}{4}\).
Solution:
L.H.S. = sin 78° + cos 132°
= sin 78° + cos (90° + 42°)
= sin 78° – sin 42°
Inter 1st Year Maths 1A Trigonometric Ratios up to Transformations Important Questions 38

Question 46.
Prove that sin 21° cos 9° – cos 84° cos 6° = \(\frac{1}{4}\).
Solution:
LH.S. = sin 21° cos 9° – cos 84° cos 6°
= \(\frac{1}{2}\)[2 sin 21° cos 9° – 2 cos 84° cos 6°]
= \(\frac{1}{2}\)[sin (21° + 9°) + sin (21° – 9°) – 2 cos (90° – 6°) cos 6°]
= \(\frac{1}{2}\)[sin 30° + sin 12° – 2 sin 6° cos 6°]
= \(\frac{1}{2}\)[\(\frac{1}{2}\) + sin 12° – sin (2 × 16°)
= \(\frac{1}{4}\) = R.H.S.

Question 47.
Find the value of sin 34° + cos 64° – cos 4°.
Solution:
sin 34° + (cos 64° – cos 4°)
Inter 1st Year Maths 1A Trigonometric Ratios up to Transformations Important Questions 39

Question 48.
Prove that cos2 76° + cos2 16° – cos 76° cos 16° = \(\frac{3}{4} .\)
Solution:
L.H.S. = cos2 76° + cos2 16° – cos 76° cos 16°
= cos2 76° + (1 – sin2 16°) – \(\frac{1}{2}\)(2 cos 76° cos 16°)
= 1 + (cos2 76° – sin2 16°) – \(\frac{1}{2}\)[cos (76° + 16°) + cos (76° – 16°)]
= 1 + cos (76° + 16°). cos (76° – 16°) – \(\frac{1}{2}\)[cos 92° + cos 60°]
= 1 + cos (92°). cos 60° – \(\frac{1}{2}\)cos 92° – \(\frac{1}{2}\)cos 60°
= 1 + \(\frac{1}{2}\). cos 92° – \(\frac{1}{2}\). cos 92° – \(\frac{1}{2}\) × \(\frac{1}{2}\)
= 1 – \(\frac{1}{4}\) = \(\frac{3}{4}\) = R.H.S.

Question 49.
If a, b, ≠ 0 and sin x + sin y = a and cos x + cos y = b, find two values of
i) tan \(\left(\frac{x+y}{2}\right)\)
ii) sin \(\left(\frac{x-y}{.2}\right)\) interms of a and b.
Solution:
Inter 1st Year Maths 1A Trigonometric Ratios up to Transformations Important Questions 40

First method:

ii) a2 + b2 = (sin x + sin y)2 + (cos x + cos y)2
= sin2 x + sin2 y + 2 sin x sin y + cos2 x + cos2 y + 2 cos x cos y
= 2 + 2(cos x cos y + sin x sin y)
= 2 + 2 cos (x – y)
= 2[1 + cos (x – y)]
a2 + b2 – 2 = 2 cos (x – y)
Inter 1st Year Maths 1A Trigonometric Ratios up to Transformations Important Questions 41

Second method:

Inter 1st Year Maths 1A Trigonometric Ratios up to Transformations Important Questions 42

Question 50.
Prove that cos 12° + cos 84° + cos 132° + cos 156° = –\(\frac{1}{2}\).
Solution:
L.H.S. = cos 12° + cos 84° + cos 132° + cos 156°
Inter 1st Year Maths 1A Trigonometric Ratios up to Transformations Important Questions 43

Inter 1st Year Maths 1A Trigonometric Ratios up to Transformations Important Questions

Question 51.
Show that for any θ ∈ R
4 sin \(\frac{5 \theta}{2}\) cos \(\frac{3 \theta}{2}\) cos 3θ = sin θ – sin 2θ + sin 4θ + sin 7θ
Solution:
R.H.S. = 4 sin \(\frac{5 \theta}{2}\) cos \(\frac{3 \theta}{2}\) cos 3θ
Inter 1st Year Maths 1A Trigonometric Ratios up to Transformations Important Questions 44
= 2 [(sin 4θ + sin θ) cos 3θ]
= 2 sin 4θ cos 3θ + 2 sinθ cos 3θ
= sin (4θ + 3θ) + sin (4θ – 3θ) + sin (θ + 3θ) + sin(θ – 3θ)
= sin 7θ + sin θ + sin 4θ – sin 2θ = R.H.S.

Question 52.
If none of A, B, A + B is an integral multiple of π, then prove that
Inter 1st Year Maths 1A Trigonometric Ratios up to Transformations Important Questions 45
Solution:
Inter 1st Year Maths 1A Trigonometric Ratios up to Transformations Important Questions 46
Inter 1st Year Maths 1A Trigonometric Ratios up to Transformations Important Questions 47

Question 53.
For any α ∈ R, provethat cos2 (α – π/4) + cos2 (α + π/12) – cos2 (α – π/12) = \(\frac{1}{2}\).
Solution:
Inter 1st Year Maths 1A Trigonometric Ratios up to Transformations Important Questions 48

Question 54.
Suppose (α – β) is not an odd multiple of \(\frac{\pi}{2}\), m is a non – zero real number such that m ≠ -1 and \(\frac{\sin (\alpha+\beta)}{\cos (\alpha-\beta)}\) = \(\frac{1-m}{1+m}\), Then prove that tan \(\left(\frac{\pi}{4}-\alpha\right)\) = m. tan \(\left(\frac{\pi}{4}+\beta\right)\)
Solution:
Inter 1st Year Maths 1A Trigonometric Ratios up to Transformations Important Questions 49
Inter 1st Year Maths 1A Trigonometric Ratios up to Transformations Important Questions 50

Question 55.
If A, B, C are the angles of a triangle, prove that
i) sin 2A + sin 2B + sin 2C = 4 sin A sin B sin C
ii) sin 2A + sin 2B – sin 2C = 4 cos A cos B sin C
Solution:
i) ∵ A, B, C are angles of a triangle
⇒ A + B + C = 180° ——- (1)
L.H.S. = sin 2A + sin 2B + sin 2C
Inter 1st Year Maths 1A Trigonometric Ratios up to Transformations Important Questions 51
= 2 sin (A + B) cos (A – B) + sin 2C
= 2 sin (180° – C) cos (A – B) + sin 2C
By(1)
= 2 sin C. cos (A — B) + 2 sin C. cos C
= 2 sin C [cos (A — B) + cos C]
= 2 sin C [cos (A — B) + cos (180° — \(\overline{A+B}\))]
= 2 sin C [cos (A – B) — cos (A + B)]
= 2 sin C (2 sin A sin B)
= 4 sin A sin B sin C = R.H.S.

ii) LH.S. = sin 2A + sin 28 — sin 2C
Inter 1st Year Maths 1A Trigonometric Ratios up to Transformations Important Questions 52
= 2 sin(A+ B)cos(A—B)—sin2C
= 2 sin (180° — C) cos (A — B) — sin 2C
= 2 sin C. cos(A – B) – 2 sin C cos C
= 2 sin C [cos (A – B) – cos C]
= 2 sin C [cos (A — B)—cos (180°— \(\overline{A+B}\))]
By(1)
= 2 sin C [cos (A — B) + cos (A + B)]
= 2 sin C [2 cos A cas B]
= 4 cos A cos B sin C = R.H.S.

Question 56.
If A, B, C are angles of a triangle, prove that
i) cos 2A + cos 2B + cos 2C
= -4 cos A cos B cos C – 1

ii) cos 2A + cos 2B – cos 2C
= 1 – 4 sin A sin B cos C
Solution:
i) ∵ A, B, C are angles of a triangle
⇒ A + B + C = 180° ———— (1)
L.H.S. = (cos 2A + cos 2B) + cos 2C
= 2 cos(A + B) cos (A – B) + 2 cos2 C – 1
= -2 cos C cos (A – B) – 2 cos C cos (A + B) – 1
[∵ cos (A + B) = -cos C]
= – 1 – 2 cos C[cos (A – B) + cos(A – B)]
= – 1 – 2 cos C[2 cos A cos B]
= -1 – 4 cos A cos B cos C = R.H.S.

ii) L.H.S. = cos 2A + cos 2B – cos 2C
Inter 1st Year Maths 1A Trigonometric Ratios up to Transformations Important Questions 53
= 2 cos (A + B). cos (A – B) – (2 cos2 C – 1)
= 2 cos (180° – C) cos (A – B) – 2 cos2 C + 1
= 1 – 2 cos C cos (A – B) – 2 cos2 C
= 1 – 2 cos C [cos (A – B) + cos C]
= 1 – 2 cos C[cos(A – B) + cos(180° – \(\overline{\mathrm{A}+\mathrm{B}}\))]
= 1 – 2 cos C [cos (A – B) – cos (A + B)]
= 1 – 2 cos C [2 sin A sin B]
= 1 – 4 sin A sin B cos C = R.H.S.

Inter 1st Year Maths 1A Trigonometric Ratios up to Transformations Important Questions

Question 57.
If A B, C are angles in a triangle, then prove that
i) sin A + sin B + sin C = 4 cos \(\frac{A}{2}\) cos \(\frac{B}{2}\) cos \(\frac{C}{2}\)
ii) cos A + cos B + cos C = 1 + 4 sin \(\frac{A}{2}\) sin \(\frac{B}{2}\) sin \(\frac{C}{2}\)
Solution:
Inter 1st Year Maths 1A Trigonometric Ratios up to Transformations Important Questions 54
Inter 1st Year Maths 1A Trigonometric Ratios up to Transformations Important Questions 55
Inter 1st Year Maths 1A Trigonometric Ratios up to Transformations Important Questions 56

Question 58.
If A + B + C = π/2, then show that
i) sin2 A + sin2 B + sin2 C = 1 – 2 sin A sin B sin C
ii) sin 2A + sin 2B + sin 2C = 4 cos A cos B cos C
Solution:
A + B + C = π/2 ——– (1)
L.H.S. = sin2 A + sin2 B + sin2 C
= \(\frac{1}{2}\)(1 – cos2A + 1 – cos 2B + 1 – cos 2C]
= \(\frac{1}{2}\)[3 – (cos 2A + cos 2B + cos 2C)]
= \(\frac{1}{2}\)[3 – (1 + 4 sin A sin B sin C)
(By Problem No. 57(ii)]
(∵ 2A + 2B + 2C = 180°)
= \(\frac{1}{2}\)[2 – 4 sin A sin B sin C]
= 1 – 2 sin A sin B sin C

ii) A + B + C = 90°
⇒ 2A + 2B + 2C = 180°
2A + 2B + 2C 180°
sin 2A + sin 2B + sin 2C
= 4 cos A cos B cos C
By Problem No. 57(i).

Question 59.
If A + B + C = 3π/2, prove that cos 2A + cos 2B + cos 2C = 1 – 4 sin A sin B sin C.
Solution:
A + B + C = 3π/2 —— (1)
L.H.S. = cos 2A + cos 2B + cos 2C
= 2 cos (A + B). cos(A – B) +1 – 2 sin2 C
= 2 cos (270° – C). cos (A – B) + 1 – 2 sin2 C
= 1 – 2 sin C cos (A – B) – 2 sin2C
= 1 – 2 sin C [cos(A – B) + sin C]
= 1 – 2 sin C[cos (A – B) + sin (270° – \(\overline{A+B}\))]
= 1 – 2 sin C[cos(A – B) – cos(A + B)]
= 1 – 2 sin C[2 sin A sin B]
= 1 – 4 sin A sin B sin C

Question 60.
If A B, C are angles of a triangle, then prove that
sin2 \(\frac{A}{2}\) + sin2 \(\frac{B}{2}\) – sin2 \(\frac{C}{2}\) = 1 – 2 cos \(\frac{A}{2}\) cos \(\frac{B}{2}\) sin \(\frac{C}{2}\) (Mar. ’16, May ’12, ’11)
Solution:
Inter 1st Year Maths 1A Trigonometric Ratios up to Transformations Important Questions 57

Inter 1st Year Maths 1A Trigonometric Ratios up to Transformations Important Questions

Question 61
If A. B, C are angles of a triangle, then prove that sin \(\frac{\mathbf{A}}{\mathbf{2}}\) + sin \(\frac{\mathbf{B}}{\mathbf{2}}\) + sin \(\frac{\mathbf{C}}{\mathbf{2}}\) = 1 + 4 sin \(\frac{\pi-\mathbf{A}}{4}\) sin \(\frac{\pi-B}{4}\) sin \(\frac{\mathrm{C}}{2}\)
Solution:
A + B + C = 180° ——- (1)
Inter 1st Year Maths 1A Trigonometric Ratios up to Transformations Important Questions 58
Inter 1st Year Maths 1A Trigonometric Ratios up to Transformations Important Questions 59
Inter 1st Year Maths 1A Trigonometric Ratios up to Transformations Important Questions 60

Question 62.
If A + B + C = 0, then prove that cos2 A + cos2 B + cos2 C = 1 + 2 cos A cos B cos C.
Solution:
A + B + C = 0 —— (1)
L.H.S. = cos2 A + cos2 B + cos2 C
= cos2 A + (1 – sin2 B) + cos2 C
= 1 + (cos2 A – sin2 B) + cos2 C
= 1 + cos (A + B) cos (A – B) + cos2 C
= 1 + cos(-C) cos(A – B) + cos2 C
By (1)
= 1 + cos C cos (A – B) + cos2 C
= 1 + cos C [cos (A – B) + cos c]
= 1 + cos C[cos(A – B) + cos(-B – A)]
By (1)
1 + cos C[cos (A – B) + cos(A + B)]
= 1 + cos C (2 cos A cos B)
= 1 + 2 cos A cos B cos C = R.H.S.

Question 63.
If A + B + C = 25, then prove that cos (S — A) + cos (S — B) + cos (S — C) + cos (S) = 4 cos \(\frac{A}{2}\) cos \(\frac{B}{2}\) cos \(\frac{C}{2}\).
Solution:
Inter 1st Year Maths 1A Trigonometric Ratios up to Transformations Important Questions 61

Inter 1st Year Maths 1A Hyperbolic Functions Solutions Ex 9(a)

Practicing the Intermediate 1st Year Maths 1A Textbook Solutions Inter 1st Year Maths 1A Hyperbolic Functions Solutions Exercise 9(a) will help students to clear their doubts quickly.

Intermediate 1st Year Maths 1A Hyperbolic Functions Solutions Exercise 9(a)

Question 1.
If sinh x = \(\frac{3}{4}\), find cosh (2x) and sinh (2x).
Solution:
Inter 1st Year Maths 1A Hyperbolic Functions Solutions Ex 9(a) Q1

Question 2.
If sinh x = 3, then show that x = loge(3 + √10).
Solution:
Inter 1st Year Maths 1A Hyperbolic Functions Solutions Ex 9(a) Q2

Inter 1st Year Maths 1A Hyperbolic Functions Solutions Ex 9(a)

Question 3.
Prove that
(i) tanh (x – y) = \(\frac{\tanh x-\tanh y}{1-\tanh x \tanh y}\)
Solution:
Inter 1st Year Maths 1A Hyperbolic Functions Solutions Ex 9(a) Q3(i)
Inter 1st Year Maths 1A Hyperbolic Functions Solutions Ex 9(a) Q3(i).1

(ii) coth (x – y) = \(\frac{{coth} x \cdot {coth} y-1}{{coth} y-{coth} x}\)
Solution:
Inter 1st Year Maths 1A Hyperbolic Functions Solutions Ex 9(a) Q3(ii)

Question 4.
Prove that
(i) (cosh x – sinh x)n = cosh (nx) – sinh (nx), for any n ∈ R.
Solution:
Inter 1st Year Maths 1A Hyperbolic Functions Solutions Ex 9(a) Q4(i)

(ii) (cosh x + sinh x)n = cosh (nx) + sinh (nx), for any n ∈ R.
Solution:
Inter 1st Year Maths 1A Hyperbolic Functions Solutions Ex 9(a) Q4(ii)

Inter 1st Year Maths 1A Hyperbolic Functions Solutions Ex 9(a)

Question 5.
Prove that \(\frac{\tanh x}{{sech} x-1}+\frac{\tanh x}{{sech} x+1}\) = -2 cosech x, for x ≠ 0
Solution:
Inter 1st Year Maths 1A Hyperbolic Functions Solutions Ex 9(a) Q5

Question 6.
Prove that \(\frac{\cosh x}{1-\tanh x}+\frac{\sinh x}{1-{coth} x}\) = sinh x + cosh x, for x ≠ 0
Solution:
Inter 1st Year Maths 1A Hyperbolic Functions Solutions Ex 9(a) Q6
Inter 1st Year Maths 1A Hyperbolic Functions Solutions Ex 9(a) Q6.1

Question 7.
For any x ∈ R, prove that cosh4x – sinh4x = cosh (2x)
Solution:
L.H.S = cosh4x – sinh4x
= (cosh2x)2 – (sinh2x)2
= [cosh2x – sinh2x] [cosh2x + sinh2x]
= (1) cosh (2x)
= cosh (2x)
∴ cosh4x – sinh4x = cosh (2x)

Inter 1st Year Maths 1A Hyperbolic Functions Solutions Ex 9(a)

Question 8.
If u = \(\log _{e}\left(\tan \left(\frac{\pi}{4}+\frac{\theta}{2}\right)\right)\) and if cos θ > 0,then prove that cosh u = sec θ.
Solution:
Inter 1st Year Maths 1A Hyperbolic Functions Solutions Ex 9(a) Q8

Inter 1st Year Maths 1A Inverse Trigonometric Functions Solutions Ex 8(a)

Practicing the Intermediate 1st Year Maths 1A Textbook Solutions Inter 1st Year Maths 1A Inverse Trigonometric Functions Solutions Exercise 8(a) will help students to clear their doubts quickly.

Intermediate 1st Year Maths 1A Inverse Trigonometric Functions Solutions Exercise 8(a)

I.

Question 1.
Evaluate the following.
(i) \(\sin ^{-1}\left(\frac{-\sqrt{3}}{2}\right)\)
Solution:
Inter 1st Year Maths 1A Inverse Trigonometric Functions Solutions Ex 8(a) I Q1(i)

(ii) \(\cos ^{-1}\left(\frac{1}{\sqrt{2}}\right)\)
Solution:
\(\cos ^{-1}\left(\frac{1}{\sqrt{2}}\right)=\cos ^{-1}\left(\cos \frac{\pi}{4}\right)=\frac{\pi}{4}\)

Inter 1st Year Maths 1A Inverse Trigonometric Functions Solutions Ex 8(a)

(iii) sec-1(-√2)
Solution:
Inter 1st Year Maths 1A Inverse Trigonometric Functions Solutions Ex 8(a) I Q1(iii)

(iv) cot-1(-√3)
Solution:
Inter 1st Year Maths 1A Inverse Trigonometric Functions Solutions Ex 8(a) I Q1(iv)

(v) \(\sin \left(\frac{\pi}{3}-\sin ^{-1}\left(\frac{-1}{2}\right)\right)\)
Solution:
Inter 1st Year Maths 1A Inverse Trigonometric Functions Solutions Ex 8(a) I Q1(v)

(vi) \(\sin ^{-1}\left(\sin \frac{5 \pi}{6}\right)\)
Solution:
Inter 1st Year Maths 1A Inverse Trigonometric Functions Solutions Ex 8(a) I Q1(vi)

(vii) \(\cos ^{-1}\left(\cos \frac{5 \pi}{4}\right)\)
Solution:
Inter 1st Year Maths 1A Inverse Trigonometric Functions Solutions Ex 8(a) I Q1(vii)

Question 2.
Find the values of
(i) \(\sin \left(\cos ^{-1} \frac{3}{5}\right)\)
Solution:
\(\sin \left(\cos ^{-1} \frac{3}{5}\right)=\sin \left(\sin ^{-1} \frac{4}{5}\right)=\frac{4}{5}\)

(ii) \(\tan \left({cosec}^{-1} \frac{65}{63}\right)\)
Solution:
\(\tan \left({cosec}^{-1} \frac{65}{63}\right)=\tan \left(\tan ^{-1} \frac{63}{16}\right)\) = \(\frac{63}{16}\)

(iii) \(\sin \left(2 \sin ^{-1} \frac{4}{5}\right)\)
Solution:
Inter 1st Year Maths 1A Inverse Trigonometric Functions Solutions Ex 8(a) I Q2(iii)

(iv) \(\sin ^{-1}\left(\sin \frac{33 \pi}{7}\right)\)
Solution:
Inter 1st Year Maths 1A Inverse Trigonometric Functions Solutions Ex 8(a) I Q2(iv)

(v) \(\cos ^{-1}\left(\cos \frac{17 \pi}{6}\right)\)
Solution:
Inter 1st Year Maths 1A Inverse Trigonometric Functions Solutions Ex 8(a) I Q2(v)

Inter 1st Year Maths 1A Inverse Trigonometric Functions Solutions Ex 8(a)

Question 3.
Simplify each of the following.
(i) \(\tan ^{-1}\left[\frac{\sin x}{1+\cos x}\right]\)
Solution:
Inter 1st Year Maths 1A Inverse Trigonometric Functions Solutions Ex 8(a) I Q3(i)

(ii) tan-1(sec x + tan x)
Solution:
Inter 1st Year Maths 1A Inverse Trigonometric Functions Solutions Ex 8(a) I Q3(ii)

(iii) \(\tan ^{-1} \sqrt{\frac{1-\cos x}{1+\cos x}}\)
Solution:
Inter 1st Year Maths 1A Inverse Trigonometric Functions Solutions Ex 8(a) I Q3(iii)

(iv) sin-1(2 cos2θ – 1) + cos-1(1 – 2 sin2θ)
Solution:
sin-1(cos 2θ) + cos-1(cos 2θ)
= sin-1[sin (90° – 2θ)] + cos-1(cos 2θ)
= 90° – 2θ + 2θ
= 90°

(v) \(\tan ^{-1}\left(x+\sqrt{1+x^{2}}\right)\); x ∈ R
Solution:
Inter 1st Year Maths 1A Inverse Trigonometric Functions Solutions Ex 8(a) I Q3(v)
Inter 1st Year Maths 1A Inverse Trigonometric Functions Solutions Ex 8(a) I Q3(v).1

II.

Question 1.
Prove that
(i) \(\sin ^{-1} \frac{3}{5}+\sin ^{-1} \frac{8}{17}=\cos ^{-1} \frac{36}{85}\)
Solution:
Inter 1st Year Maths 1A Inverse Trigonometric Functions Solutions Ex 8(a) II Q1(i)
Inter 1st Year Maths 1A Inverse Trigonometric Functions Solutions Ex 8(a) II Q1(i).1

(ii) \(\sin ^{-1} \frac{3}{5}+\cos ^{-1} \frac{12}{13}=\cos ^{-1} \frac{33}{65}\)
Solution:
Inter 1st Year Maths 1A Inverse Trigonometric Functions Solutions Ex 8(a) II Q1(ii)
Inter 1st Year Maths 1A Inverse Trigonometric Functions Solutions Ex 8(a) II Q1(ii).1

(iii) \(\tan \left[\cot ^{-1} 9+{cosec}^{-1} \frac{\sqrt{41}}{4}\right]=1\)
Solution:
Inter 1st Year Maths 1A Inverse Trigonometric Functions Solutions Ex 8(a) II Q1(iii)

(iv) \(\cos ^{-1} \frac{4}{5}+\sin ^{-1} \frac{3}{\sqrt{34}}=\tan ^{-1} \frac{27}{11}\)
Solution:
Inter 1st Year Maths 1A Inverse Trigonometric Functions Solutions Ex 8(a) II Q1(iv)
Inter 1st Year Maths 1A Inverse Trigonometric Functions Solutions Ex 8(a) II Q1(iv).1

Inter 1st Year Maths 1A Inverse Trigonometric Functions Solutions Ex 8(a)

Question 2.
Find the values of
(i) \(\sin \left(\cos ^{-1} \frac{3}{5}+\cos ^{-1} \frac{12}{13}\right)\)
Solution:
Inter 1st Year Maths 1A Inverse Trigonometric Functions Solutions Ex 8(a) II Q2(i)

(ii) \(\tan \left(\sin ^{-1} \frac{3}{5}+\cos ^{-1} \frac{5}{\sqrt{34}}\right)\)
Solution:
Inter 1st Year Maths 1A Inverse Trigonometric Functions Solutions Ex 8(a) II Q2(ii)

(iii) \(\cos \left(\sin ^{-1} \frac{3}{5}+\sin ^{-1} \frac{5}{13}\right)\)
Solution:
Let \(\sin ^{-1} \frac{3}{5}\) = α and \(\sin ^{-1} \frac{5}{13}\) = β
Inter 1st Year Maths 1A Inverse Trigonometric Functions Solutions Ex 8(a) II Q2(iii)

Question 3.
Prove that
(i) \(\cos \left[2 \tan ^{-1} \frac{1}{7}\right]=\sin \left[2 \tan ^{-1} \frac{3}{4}\right]\)
Solution:
Inter 1st Year Maths 1A Inverse Trigonometric Functions Solutions Ex 8(a) II Q3(i)

(ii) \(\tan \left[2 \tan ^{-1}\left(\frac{\sqrt{5}-1}{2}\right)\right]=2\)
Solution:
Inter 1st Year Maths 1A Inverse Trigonometric Functions Solutions Ex 8(a) II Q3(ii)

(iii) \(\cos \left\{2\left[\tan ^{-1}\left(\frac{1}{4}\right)+\tan ^{-1}\left(\frac{2}{9}\right)\right]\right\}=\frac{3}{5}\)
Solution:
Inter 1st Year Maths 1A Inverse Trigonometric Functions Solutions Ex 8(a) II Q3(iii)

Question 4.
Prove that
(i) \(\tan ^{-1} \frac{1}{7}+\tan ^{-1} \frac{1}{13}-\tan ^{-1} \frac{2}{9}=0\)
Solution:
Inter 1st Year Maths 1A Inverse Trigonometric Functions Solutions Ex 8(a) II Q4(i)
Inter 1st Year Maths 1A Inverse Trigonometric Functions Solutions Ex 8(a) II Q4(i).1

(ii) \(\tan ^{-1} \frac{1}{2}+\tan ^{-1} \frac{1}{5}+\tan ^{-1} \frac{1}{8}=\frac{\pi}{4}\)
Solution:
Inter 1st Year Maths 1A Inverse Trigonometric Functions Solutions Ex 8(a) II Q4(ii)

(iii) \(\tan ^{-1} \frac{3}{4}+\tan ^{-1} \frac{3}{5}-\tan ^{-1} \frac{8}{19}=\frac{\pi}{4}\)
Solution:
Inter 1st Year Maths 1A Inverse Trigonometric Functions Solutions Ex 8(a) II Q4(iii)
Inter 1st Year Maths 1A Inverse Trigonometric Functions Solutions Ex 8(a) II Q4(iii).1

(iv) \(\tan ^{-1} \frac{1}{7}+\tan ^{-1} \frac{1}{8}\) = \(\cot ^{-1} \frac{201}{43}+\cot ^{-1} 18\)
Solution:
Inter 1st Year Maths 1A Inverse Trigonometric Functions Solutions Ex 8(a) II Q4(iv)

Inter 1st Year Maths 1A Inverse Trigonometric Functions Solutions Ex 8(a)

Question 5.
Show that
(i) sec2 (tan-1 2) + cosec2 (cot-1 2) = 10
Solution:
Let a = tan-1 2 ⇒ tan α = 2
sec2 α = 1 + tan-1 α = 1 + 4 = 5
Let β = cot-1 2 ⇒ cot β = 2
cosec2 β = 1 + cot2 β = 1 + 4 = 5
LHS = sec2 (tan-1 2) + cosec2 (cot-1 2)
= 5 + 5
= 10
= RHS

(ii) Find the value of \(\left(\cos ^{-1} \frac{4}{5}+\tan ^{-1} \frac{2}{3}\right)\)
Solution:
Inter 1st Year Maths 1A Inverse Trigonometric Functions Solutions Ex 8(a) II Q5(ii)

(iii) If sin-1 x – cos-1 x = \(\frac{\pi}{6}\) then find x.
Solution:
Inter 1st Year Maths 1A Inverse Trigonometric Functions Solutions Ex 8(a) II Q5(iii)

III.

Question 1.
Prove that
(i) \(2 \sin ^{-1} \frac{3}{5}-\cos ^{-1} \frac{5}{13}=\cos ^{-1} \frac{323}{325}\)
Solution:
Inter 1st Year Maths 1A Inverse Trigonometric Functions Solutions Ex 8(a) III Q1(i)

(ii) \(\sin ^{-1} \frac{4}{5}+2 \tan ^{-1} \frac{1}{3}=\frac{\pi}{2}\)
Solution:
Inter 1st Year Maths 1A Inverse Trigonometric Functions Solutions Ex 8(a) III Q1(ii)
Inter 1st Year Maths 1A Inverse Trigonometric Functions Solutions Ex 8(a) III Q1(ii).1

(iii) \(4 \tan ^{-1} \frac{1}{5}+\tan ^{-1} \frac{1}{99}-\tan ^{-1} \frac{1}{70}=\frac{\pi}{4}\)
Solution:
Inter 1st Year Maths 1A Inverse Trigonometric Functions Solutions Ex 8(a) III Q1(iii)
Inter 1st Year Maths 1A Inverse Trigonometric Functions Solutions Ex 8(a) III Q1(iii).1

Question 2.
(i) If α = \({tan}^{-1}\left(\frac{\sqrt{1+x^{2}}-\sqrt{1-x^{2}}}{\sqrt{1+x^{2}}+\sqrt{1-x^{2}}}\right)\) then prove that x2 = sin 2α.
Solution:
Inter 1st Year Maths 1A Inverse Trigonometric Functions Solutions Ex 8(a) III Q2(i)
Inter 1st Year Maths 1A Inverse Trigonometric Functions Solutions Ex 8(a) III Q2(i).1

(ii) Prove that tan\(\left\{2-\tan ^{-1}\left(\frac{\sqrt{1+x^{2}}-1}{x}\right)\right\}=\mathbf{x}\)
Solution:
Inter 1st Year Maths 1A Inverse Trigonometric Functions Solutions Ex 8(a) III Q2(ii)

(iii) Prove that \(\sin \left[\cot ^{-1} \frac{2 x}{1-x^{2}}+\cos ^{-1}\left(\frac{1-x^{2}}{1+x^{2}}\right)\right]\) = 1
Solution:
Inter 1st Year Maths 1A Inverse Trigonometric Functions Solutions Ex 8(a) III Q2(iii)

(iv) Prove that \(\left\{\frac{\pi}{4}+\frac{1}{2} \cos ^{-1}\left(\frac{a}{b}\right)\right\}\)
Solution:
Inter 1st Year Maths 1A Inverse Trigonometric Functions Solutions Ex 8(a) III Q2(iv)

Question 3.
(i) If cos-1 p + cos-1 q + cos-1 r = π, then prove that p2 + q2 + r2 + 2pqr = 1
Solution:
Let cos-1 p = A, cos-1 q = B and cos-1 r = C
then A + B + C = π ………(1)
and p = cos A, q = cos B and r cos C
Now p2 + q2 + r2 = cos2 A + cos2 B + cos2 C
= cos2 A + (1 – sin2 B + cos2 C)
= 1 + (cos2 A – sin2 B) + cos2 C
= 1 + cos (A + B) . cos (A – B) + cos2 C
= 1 + cos (π – C) cos (A – B) + cos2 C (By (1))
= 1 – cos C cos (A – B) + cos2 C
= 1 – cos C [cos (A – B) – cos C]
= 1 – cos C [cos (A – B) – cos(180° – \(\overline{A+B}\)]
= 1 – cos C [cos (A – B) + cos (A + B)]
= 1 – cos C [2 cos A cos B]
= 1 – 2 pqr
∴ p2 + q2 + r2 + 2pqr = 1

Inter 1st Year Maths 1A Inverse Trigonometric Functions Solutions Ex 8(a)

(ii) If \(\sin ^{-1}\left(\frac{2 p}{1+p^{2}}\right)-\cos ^{-1}\left(\frac{1-q^{2}}{1+q^{2}}\right)\) = \(\tan ^{-1}\left(\frac{2 x}{1-x^{2}}\right)\), then prove that x = \(\frac{p-q}{1+p q}\)
Solution:
Inter 1st Year Maths 1A Inverse Trigonometric Functions Solutions Ex 8(a) III Q3(ii)

(iii) If a, b, c are distinct non-zero real numbers having the same sign, prove that \(\cot ^{-1}\left(\frac{a b+1}{a-b}\right)+\cot ^{-1}\left(\frac{b c+1}{b-c}\right)\) + \(\cot ^{-1}\left(\frac{c a+1}{c-a}\right)\) = π or 2π
Solution:
Since (a – b) + (b – c) + (c – a) = 0.
(a – b), (b – c), (c – a) all cannot have the same sign.
Now two cases arise, namely, either two of these numbers are positive and one negative (or) two of these numbers are negative and one is positive.
Case (i): Without loss of generality, we assume that (a – b), (b – c) are both positive and (c – a) is negative
Inter 1st Year Maths 1A Inverse Trigonometric Functions Solutions Ex 8(a) III Q3(iii)
Case (ii): Without loss of generality, we assume that (a – b) and (b – c) are both negative and (c – a) is positive.
Inter 1st Year Maths 1A Inverse Trigonometric Functions Solutions Ex 8(a) III Q3(iii).1

(iv) If sin-1 (x) + sin-1 (y) + sin-1 (z) = π, then prove that \(x \sqrt{1-x^{2}}+y \sqrt{1-y^{2}}+z \sqrt{1-z^{2}}=2 x y z\)
Solution:
Let sin-1 (x) = A, sin-1 (y) = B and sin-1 (z) = C
Then A + B + C = π …………(1)
and x = sin A, y = sin B and z = sin C
Now LHS = \(x \sqrt{1-x^{2}}+y \sqrt{1-y^{2}}+z \sqrt{1-z^{2}}\)
= sin A \(\sqrt{1-\sin ^{2} A}\) + sin B \(\sqrt{1-\sin ^{2} B}\) + sin C \(\sqrt{1-\sin ^{2} C}\)
= sin A cos A + sin B cos B + sin C cos C
= \(\frac{1}{2}\) [sin 2A + sin 2B + sin 2C]
= \(\frac{1}{2}\) [2 . sin (A + B) cos (A – B) + sin 2C]
= \(\frac{1}{2}\) [2 sin (π – c). cos (A – B) + sin 2C]
= \(\frac{1}{2}\) [2 sin C cos (A – B) + 2 sin C cos C]
= \(\frac{1}{2}\) (2 sin C) [cos (A – B) + cos C]
= sin C [cos (A – B) + cos (180° – \(\overline{A+B}\)]
= sin C [cos (A – B) – cos (A + B)]
= sin C [2 sin A sin B]
= 2 xyz
∴ \(x \sqrt{1-x^{2}}+y \sqrt{1-y^{2}}+z \sqrt{1-z^{2}}=2 x y z\)

(v) (a) If tan-1 x + tan-1 y + tan-1 z = π, then prove that x + y + z = xyz.
Solution:
Let A = tan-1 x, B = tan-1 y, C = tan-1 z
tan A = x, tan B = y, tan C = z
Given A + B + C = π ……….(1)
A + B = π – C
tan (A + B) = tan (π – C)
\(\frac{\tan A+\tan B}{1-\tan A \tan B}\) = -tan C
\(\frac{x+y}{1-x y}\) = -z
x + y = -z + xyz
∴ x + y + z = xyz

(b) If tan-1 x + tan-1 y + tan-1 z = \(\frac{\pi}{2}\), then prove that xy + yz + zx = 1.
Solution:
Let tan-1 x = A, tan-1 y = B and tan-1 z = C
Then A + B + C = \(\frac{\pi}{2}\) …….(1)
and x = tan A, y = tan B and z = tan C
∵ A + B + C = \(\frac{\pi}{2}\)
A + B = \(\frac{\pi}{2}\) – C
⇒ tan (A + B) = tan(\(\frac{\pi}{2}\) – C)
⇒ \(\frac{\tan A+\tan B}{1-\tan A \cdot \tan B}\) = cot C
⇒ \(\frac{x+y}{1-x y}=\frac{1}{z}\)
⇒ zx + yz = 1 – xy (or) xy + yz + zx = 1

Inter 1st Year Maths 1A Inverse Trigonometric Functions Solutions Ex 8(a)

Question 4.
Solve the following equations for x:
(i) \({Tan}^{-1}\left(\frac{x-1}{x-2}\right)+{Tan}^{-1}\left(\frac{x+1}{x+2}\right)=\frac{\pi}{4}\)
Solution:
Inter 1st Year Maths 1A Inverse Trigonometric Functions Solutions Ex 8(a) III Q4(i)

(ii) \(\tan ^{-1}\left(\frac{1}{2 x+1}\right)+\tan ^{-1}\left(\frac{1}{4 x+1}\right)\) = \(\tan ^{-1} \frac{2}{x^{2}}\)
Solution:
Given that
Inter 1st Year Maths 1A Inverse Trigonometric Functions Solutions Ex 8(a) III Q4(ii)
⇒ x2(3x + 1) = 2x(4x + 3)
⇒ x [x(3x + 1) – 2(4x + 3)] = 0
⇒ x = 0 (or) 3x2 – 7x – 6 = 0
⇒ x = 0 (or) 3x2 – 9x + 2x – 6 = 0
⇒ x = 0 (or) 3x(x – 3) + 2(x – 3) = 0
⇒ x = 0 (or) (3x + 2) (x – 3) = 0
⇒ x = 0 (or) 3 (or) \(\frac{-2}{3}\)

(iii) \(3 \sin ^{-1}\left(\frac{2 x}{1+x^{2}}\right)-4 \cos ^{-1}\left(\frac{1-x^{2}}{1+x^{2}}\right)\) + \(2 \tan ^{-1}\left(\frac{2 x}{1-x^{2}}\right)=\frac{\pi}{3}\)
Solution:
Inter 1st Year Maths 1A Inverse Trigonometric Functions Solutions Ex 8(a) III Q4(iii)
Inter 1st Year Maths 1A Inverse Trigonometric Functions Solutions Ex 8(a) III Q4(iii).1

(iv) sin-1(1 – x) – 2 sin-1 x = \(\frac{\pi}{2}\)
Solution:
Given sin-1(1 – x) – 2 sin-1x = \(\frac{\pi}{2}\)
Let sin-1(1 – x) = α and sin-1(x) = β
Then sin α = 1 – x and sin β = x
cos α = \(\sqrt{1-(1-x)^{2}}\) and cos β = \(\sqrt{1-x^{2}}\)
Now sin-1(1 – x) – 2 sin-1(x) = \(\frac{\pi}{2}\)
⇒ α – 2β = \(\frac{\pi}{2}\)
⇒ α = \(\frac{\pi}{2}\) + 2β
⇒ sin α = sin (\(\frac{\pi}{2}\) + 2β)
⇒ sin α = cos 2β
⇒ 1 – x = 1 – 2 sin2β
⇒ 1 – x = 1 – 2x2
⇒ 2x2 – x = 0
⇒ x(2x – 1) = 0
⇒ x = 0 (or) x = \(\frac{1}{2}\)
But when x = \(\frac{1}{2}\)
Inter 1st Year Maths 1A Inverse Trigonometric Functions Solutions Ex 8(a) III Q4(iv)
Hence x = 0 is the only solution for the given equation.

Inter 1st Year Maths 1A Inverse Trigonometric Functions Solutions Ex 8(a)

Question 5.
Solve the following equations.
(i) \(\cot ^{-1}\left(\frac{1+x}{1-x}\right)=\frac{1}{2} \cot ^{-1}\left(\frac{1}{x}\right)\), x > 0 and x ≠ 1
Solution:
Inter 1st Year Maths 1A Inverse Trigonometric Functions Solutions Ex 8(a) III Q5(i)

(ii) \(\tan \left[\cos ^{-1} \frac{1}{x}\right]=\sin \left[\cot ^{-1} \frac{1}{2}\right]\); x ≠ 0
Solution:
Let \(\cos ^{-1}\left(\frac{1}{x}\right)=\alpha, \cot ^{-1} \frac{1}{2}=\beta\)
Inter 1st Year Maths 1A Inverse Trigonometric Functions Solutions Ex 8(a) III Q5(ii)

(iii) \(\cos ^{-1} x+\sin ^{-1} \frac{x}{2}=\frac{\pi}{6}\)
Solution:
Inter 1st Year Maths 1A Inverse Trigonometric Functions Solutions Ex 8(a) III Q5(iii)
Inter 1st Year Maths 1A Inverse Trigonometric Functions Solutions Ex 8(a) III Q5(iii).1

(iv) \(\cos ^{-1}(\sqrt{3} \cdot x)+\cos ^{-1} x=\frac{\pi}{2}\)
Solution:
Inter 1st Year Maths 1A Inverse Trigonometric Functions Solutions Ex 8(a) III Q5(iv)
Inter 1st Year Maths 1A Inverse Trigonometric Functions Solutions Ex 8(a) III Q5(iv).1

(v) \(\sin \left[\sin ^{-1}\left(\frac{1}{5}\right)+\cos ^{-1} x\right]=1\)
Solution:
Inter 1st Year Maths 1A Inverse Trigonometric Functions Solutions Ex 8(a) III Q5(v)

Inter 1st Year Maths 1B Direction Cosines and Direction Ratios Solutions Ex 6(b)

Practicing the Intermediate 1st Year Maths 1B Textbook Solutions Inter 1st Year Maths 1B Direction Cosines and Direction Ratios Solutions Exercise 6(b) will help students to clear their doubts quickly.

Intermediate 1st Year Maths 1B Direction Cosines and Direction Ratios Solutions Exercise 6(b)

I.

Question 1.
Find the direction ratios of the line joining the points (3, 4, 0) and (4, 4, 4).
Solution:
A(3, 4, 0) and B(4, 4, 4) are the given points.
d.rs of AB are (x2 – x1, y2 – y1, z2 – z1)
(4-3, 4-4, 4-0) i.e.,(1, 0, 4)

Question 2.
The direction ratios of a line are (-6, 2, 3). Find its direction cosines.
Solution:
D.Rs of the line are -6, 2, 3
Dividing with \(\sqrt{36+4+9}\) = 7
Direction cosines of the line are – \(\frac{6}{7}, \frac{2}{7}, \frac{3}{7}\)

Inter 1st Year Maths 1B Direction Cosines and Direction Ratios Solutions Ex 6(b)

Question 3.
Find the cosine of the angle between the lines whose direction cosines are (\(\frac{1}{\sqrt{3}}, \frac{1}{\sqrt{3}}, \frac{1}{\sqrt{3}}\)) and (\(\frac{1}{\sqrt{2}}, \frac{1}{\sqrt{2}}\), 0)
Solution:
cos θ = l1 + l2 + m1 m2 + n1 n2
Inter 1st Year Maths 1B Direction Cosines and Direction Ratios Solutions Ex 6(b) 1

Question 4.
Find the angle between the lines whose direction ratios are (1, 1, 2) (√3, -√3, 0).
Solution:
Inter 1st Year Maths 1B Direction Cosines and Direction Ratios Solutions Ex 6(b) 2

Question 5.
Show that the lines with direction cosines (\(\frac{12}{13}, \frac{-3}{13}, \frac{-4}{13}\)) and (\(\frac{4}{13}, \frac{12}{13}, \frac{3}{13}\)) are perpendicular to each other.
Solution:
If two lines, are perpendicular, then
l1l2 + m1m2 + n1n2 = 0
l1l2 + m1m2 + n1n2
\(\frac{12}{13} \cdot \frac{4}{13}-\frac{3}{13} \cdot \frac{12}{13}-\frac{4}{13} \cdot \frac{3}{13}\)
∴ The given lines are perpendicular.

Question 6.
O is the origin, P(2, 3, 4) and Q(1, k, 1) are points such that \(\overline{\mathrm{O P}}\) ⊥ \(\overline{\mathrm{O Q}}\). Find k.
Solution:
d.rs of OP and 2, 3,4
d.rs of OQ are 1, k, 1
OP and OQ are perpendicular
⇒ a1a2 + b1b2 + c1c2 = 0
2 + 3k + 4 = 0
3k = -6
k = -2.

II.

Question 1.
If the direction ratios of a line are (3, 4, 0), find its direction cosines are also the angles made with the co-ordinate axes.
Solution:
Direction ratios of the line are (3, 4, 0)
Dividing with \(\sqrt{9+16+0}\) = 5
D.C’s of the line are (\(\frac{3}{5}, \frac{4}{5}\), 0)
If αc, βc² are the angles made By the line with the co-ordinate axes, then
cos α = \(\frac{3}{5}\) cos β = \(\frac{4}{5}\) cos γ = 0
α = cos-1\(\frac{3}{5}\), β = cos-1\(\frac{4}{5}\), γ = \(\frac{\pi}{2}\)
Angles made with co-ordinate axes are
cos-1\(\frac{3}{5}\), cos-1\(\frac{4}{5}\), γ = \(\frac{\pi}{2}\)

Inter 1st Year Maths 1B Direction Cosines and Direction Ratios Solutions Ex 6(b)

Question 2.
Show that the line through the points (1, -1,2) (3, 4, -2) is perpendicular to the line through the points (0, 3, 2) and (3, 5, 6).
Solution:
A(1, -1 2) B(3, 4, -2) C(0,3,2) and D(3, 5,6) are the given points
d.rs of AB are 3 – 1, 4 + 1, -2 -2 i.e., 2, 5, -4
d.rs of CD are 3 – 0, 5 – 3, 6 – 2 i.e., 3, 2 4
a1a2 + b1b2 + c1c2 = 2.3 + 5.2 – 4.4
= 6 + 10 – 16 = 0
AB and CD are perpendicular.

Question 3.
Find the angle between \(\overline{\mathrm{D C}}\) and \(\overline{\mathrm{A B}}\) where A = (3, 4, 5), B =(4, 6, 3) C = (-1, 2, 4) are D(l, 0, 5).
Solution:
A(3, 4, 5), B(4, 6, 3), C(-1, 2, 4), D(1, 0, 5) are the given points.
d.rs of AB are 4 – 3, 6 – 4, 3 – 5 i.e., 1, 2, -2
d.rs of CD are 1 + 1, 0 – 2, 5 – 4 i.e., 2, -2, 1
Inter 1st Year Maths 1B Direction Cosines and Direction Ratios Solutions Ex 6(b) 3

Question 4.
Find the direction cosines of a line which is perpendicular to the lines, whose direction ratios are (1, -1, 2) and (2, 1, -1).
Solution:
Let l, m, n be the d.cs of the required line it is perpendicular to the lines with
d.rs (1, -1, 2) and (2, 1, -1)
l – m + 2n = 0
2l + m – n = 0
Inter 1st Year Maths 1B Direction Cosines and Direction Ratios Solutions Ex 6(b) 4
d.rs of the line are – 1, 5, 3
Dividing with \(\sqrt{1+25+9} =\sqrt{35}\)
d.cs of the required line are ’
– \(\frac{1}{\sqrt{35}}, \frac{5}{\sqrt{35}}, \frac{3}{\sqrt{35}}\)

Question 5.
Show that the points (2, 3, -4), (1, -2, 3) and (3, 8, -11) are collinear.
Solution:
A(2, 3, -4),B( 1, -2, 3) and C(3, 8, -11) are the given points.
Inter 1st Year Maths 1B Direction Cosines and Direction Ratios Solutions Ex 6(b) 5
∴ A, B, C are collinear.

Question 6.
Show that the points (4, 7, 8), (2, 3, 4), (-1, -2,1), (1,2,5) are vertices of apare- llelogram.
Solution:
A(4, 7, 8), B(2, 3, 4), C(-1, -2, 1) and D (1, 2, 5) . are the given points.
Inter 1st Year Maths 1B Direction Cosines and Direction Ratios Solutions Ex 6(b) 6
∴ AB = CD and BC = DA
∴ A, B, C, D are the vertical of parallelogram.

III.

Question 1.
Show that the lines whose direction co – sines are given by l + m + n = 0, 2mn + 3nZ – 5Zm = 0 are perpendicular to each other.
Solution:
Given l + m + n = 0 ………… (1)
2mn + 3nl – 5lm = 0 …………… (2)
From (1), l = -(m + n)
Substituting in (2)
2mn – 3n(m + n) + 5m(m + n) = 0
2mn – 3mn – 3n2 + 5m2 + 5mn = 0
5m² + 4mn – 3n² = 0
\(\frac{m_{1} m_{2}}{n_{1} n_{2}}=-\frac{3}{5} \Rightarrow \frac{m_{1} m_{2}}{-3}=\frac{n_{1} n_{2}}{5}\) ………. (1)
From (1), n = -(l + m).
Substituting is (2)
-2m (l + m) – 3l(l + m) – 5lm = 0
– 2lm – 2m2 – 3l² – 3lm – 5lm = 0
3l² + 10lm + 2m² = 0
\(\frac{l_{1} l_{2}}{m_{1} m_{2}}=\frac{2}{3} \Rightarrow \frac{l_{1} l_{2}}{2}=\frac{m_{1} m_{2}}{3}\) ……… (2)
From (1) and (2) we get
\(\frac{I_{1} I_{2}}{2}=\frac{m_{1} m_{2}}{3}=\frac{n_{1} n_{2}}{-5}=1\)
l1l2 = 2k, m1m2 = 3k, n1 n2 = -5k
∴ l1l2 + m1m2 + n1n2 = 2k + 3k – 5k = 0
The two lines are perpendicular.

Inter 1st Year Maths 1B Direction Cosines and Direction Ratios Solutions Ex 6(b)

Question 2.
Find the angle between the lines whose direction cosines satisfy the equations l + m + n = 0, l² + m² – n² = 0.
Solution:
Given l + m + n = 0
l² + m² – n² = 0
From (1), l = – (m + n)
Substituting in (2)
(m + n)² + m² – n² = 0
m² + n² + 2mn + m² – n² = 0
2m² + 2mn = 0
2m(m + n) = 0
∴ m = 0 or m + n = 0

Case (i) : m = 0, Substituting in (1) l + n = 0
l = -n ⇒ \(\frac{l}{1}\) = \(\frac{n}{-1}\)
D.Rs of the first line l1 are (1, 0, – 1)

Case (ii) :m + n = 0 ⇒ m = -n ⇒ \(\frac{m}{1}\) = \(\frac{n}{-1}\)
Substituting in (1) l = 0
D.Rs of the second line l2 are (0, 1 – 1)
Suppose θ is the angle between the two lines
Inter 1st Year Maths 1B Direction Cosines and Direction Ratios Solutions Ex 6(b) 7

Question 3.
If a ray makes angles α, α, γ and δ with the four diagonals of a cube find cos² α + cos² β + cos² γ + cos² δ.
Solution:
Let each side of the cube be of length a. Let one of the vertices of the cube be the origin O and the co-ordinate axes be along the three edges \(\overline{\mathrm{O A}}\), \(\overline{\mathrm{O B}}\) and \(\overline{\mathrm{O C}}\) passing through the origin. The co-ordinates of the vertices of the cube with respect to the frame of reference OABC are as shown in figure. The diagonals of he cubeare \(\overline{\mathrm{O P}}\) \(\overline{\mathrm{C D}}\), \(\overline{\mathrm{A E}}\) and \(\overline{\mathrm{B F}}\) .(a, a, a), (a, a, -a), (-a, a, a) and (a, -a, a) are direction . ratios of these diagonals respectively.
Inter 1st Year Maths 1B Direction Cosines and Direction Ratios Solutions Ex 6(b) 8

Let the direction cosines of the given ray be (l, m, n). If this fay makes the angles α, β, γ and δ with the four diagonals of the cube, then
Inter 1st Year Maths 1B Direction Cosines and Direction Ratios Solutions Ex 6(b) 9
cos² α + cos² β + cos² γ + cos² δ
\(\frac{1}{3}\){|l + m + n|² +|l + m – n|² + |-l + m + n|² + |l – m + n|²}
\(\frac{1}{3}\)[(l + m + n)² + (l + m – n)² + (-l + m + n)² + (l – m + n)²]
\(\frac{1}{3}\)[4(l² + m² + n²)] = \(\frac{4}{3}\) (since l² + m² + n² = 1)

Question 4.
If (l1, m1, n1), (l2, m2, n2) are d.c.s of two intersecting lines, show that d.c.s of two lines, bisecting the angles between them are proportional to l1 ± l2, m1 ± m2 n1 ± n2.
Inter 1st Year Maths 1B Direction Cosines and Direction Ratios Solutions Ex 6(b) 10
Solution:
OA and OB are the given lines
A and B are points at unit distances from O
Co-ordinates of A are (l1, m1, n1)
Co-ordinates of B are (l2, m2, n2)
P is the mid point of AB
Co-ordinates of P are \(\left(\frac{l_{1}+l_{2}}{2}, \frac{m_{1}+m_{2}}{2}, \frac{n_{1}+n_{2}}{2}\right)\)
∴ OP is the bisector of ∠AOB
D.Rs of OP are l1 + l2, m1+ m2, n1+ n2
Suppose B’ is a point are OB such that OB’ OB = 1
Co-ordinates of B’ are (-l2, -m2, -n2)
Q is the mid point of AB’
Co-ordinates of Q are \(\left(\frac{l_{1}-l_{2}}{2}, \frac{m_{1}-m_{2}}{2}, \frac{n_{1}-n_{2}}{2}\right)\)
OQ is the other bisector of ∠AOB
D.Rs of OQ are l1 – l1, m1 – m2, n1 – n2

Inter 1st Year Maths 1B Direction Cosines and Direction Ratios Solutions Ex 6(b)

Question 5.
A (-1, 2, -3), B(5, 0, -6), C(0, 4, -1) are three points. Show that the direction cosines of the bisectors of ∠BAC are propotional to (25, 8, 5) and (-11, 20, 23).
Solution:
A (-1, 2, -3), B (5, 0, -6) and C (0, 4, -1) are the given points.
D.Rs of AB are 5 +1, 0 -2, -6 +3 ie., 6, -2, -3
D.Rs of AB are \(\frac{6}{7}, \frac{-2}{7}, \frac{-3}{7}\)
D.Rs of AC are 0 + 1, 4 – 2, -1 + 3 i.e., 1, 2, 2
D.Rs of AC are \(\frac{1}{3}, \frac{2}{3}, \frac{2}{3}\)
D.Rs of one of the bisectors are
Inter 1st Year Maths 1B Direction Cosines and Direction Ratios Solutions Ex 6(b) 11
D.Rs of one of the bisectos are (25, 8, 5)
D.Rs of the other bisectors are
Inter 1st Year Maths 1B Direction Cosines and Direction Ratios Solutions Ex 6(b) 12
D.Rs of the second bisector are (-11, 20, 23)

Question 6.
If (6,10,10), (1, 0, -5), (6, -10, 0) are vertices of a triangle, find the direction ratios of its sides. Determine whether it is right angled or isosceles.
Solution:
A (6, 10, 10), B (1, 0, -5), C (6, -10, 0) are the vertices of ∆ABC
D.Rs of AB are 5, 10, 15 i.e., 1, 2, 3
D.Rs of BC are -5, 10, -5 i.e., 1, -2, 1
D.Rs of AC are 0,20,10, i.e., 0, 2, 1
Inter 1st Year Maths 1B Direction Cosines and Direction Ratios Solutions Ex 6(b) 13
∴ The given triangle is right angle a.

Question 7.
The vertices of a triangle are A (1, 4, 2), B (-2, 1, 2) C (2, 3, -4). Find ∠A, ∠B, ∠C.
Inter 1st Year Maths 1B Direction Cosines and Direction Ratios Solutions Ex 6(b) 14
Solution:
A (1, 4, 2), B (-2, 1, 2), C (2, 3, -4) and the vertices of OABC.
D.Rs of AB are 3, 3, 0 i.e., 1, 1, 0
D.Rs of BC are -4, -2, 6 i.e., 2, 1, -3
D.Rs of AC are -1, 1, 6.
Inter 1st Year Maths 1B Direction Cosines and Direction Ratios Solutions Ex 6(b) 15
Inter 1st Year Maths 1B Direction Cosines and Direction Ratios Solutions Ex 6(b) 16

Question 8.
Find the angle between the lines whose direction cosines are given by the equation 3l + m + 5n = 0 and 6mn – 2nl + 5lm = 0
Solution:
Given 3l + m + 5n = 0 ……….. (1)
6mn – 2nl + 5lm = 0 …………. (2)
From (1), m = – (3l + 5n)
Substituting in (2)
-6n(3l + 5n) -2nl – 5l(3l + 5n) = 0
-18ln – 30n² – 2nl – 15l² – 25ln = 0
-15l² – 45ln – 30n² = 0
l² + 3ln + 2n² = 0
(l + 2n) (l + n) — 0
l + 2n = 0 or l + n = 0

Case (i) : l1 + n1 = 0 ⇒ n1 = – l1; ⇒ \(\frac{l_{1}}{1}=\frac{n_{1}}{-1}\)
But m1 = -(3l1 + 5n1) = -(-3n1 + 5n1) = -2n1
∴ \(\frac{m_{1}}{+2}=\frac{n_{1}}{-1}\)
∴ \(\frac{l_{1}}{1}=\frac{m_{1}}{2}=\frac{n_{1}}{-1}\)
D.Rs of the first line lx are (1,2, -1)
Case (ii): l2 + 2n2 = 0
l2 = —2n2 ⇒ \(\frac{l_{2}}{-2}=\frac{n_{2}}{1}\)
m2 = -(3l2 + 5n2) = – (-6n2 + 5n2) = n2
\(\frac{m_{2}}{1}=\frac{n_{2}}{1}\)
∴ \(\frac{l_{2}}{-2}=\frac{m_{2}}{1}=\frac{n_{2}}{1}\)
D.Rs of the second line Z2 are (-2, 1, 1)
Suppose ‘θ’ is the angle between the lines l1 and l2
Inter 1st Year Maths 1B Direction Cosines and Direction Ratios Solutions Ex 6(b) 17

Inter 1st Year Maths 1B Direction Cosines and Direction Ratios Solutions Ex 6(b)

Question 9.
If a variable line in two adjecent positions has direction cosines (l, m, n) and (l + δl, m + δm, n + δn), show that the small angle δθ between two position. Is given by (δθ)² = (δl)² + (δm)² + (δn)².
Solution:
Since (l, m, n) and (l + δl, m + δm, n + δn) are direction cosines, we have
l² + m² + n² = 1 …………. (1)
(l + δl)² + (m + δm)² + (n + δn)² — 1 ………. (2)
(2) – (1) gives (l + δl)² + (m + δm)² + (n + 8n)² – (l² + m² + n²) = 0
2(l.δl + m.δm + n.δn) = -((δl)² + (δm)² + (δn)²) …………. (3)
cos θ . δθ = l(l + δl) + m(m + δm) + n(n + δn)
= (l² + m² + n²) + (l.δl + m.δm + n.δn)
= 1 – \(\frac{1}{2}\) [(δl)² + (δm)² + (δn)²]
(δl)² + (δm)² + (δn)² = 1 = ² (1 – cos θ.δ θ)
δθ being small, sin \(\frac{\delta \theta}{2}\) = \(\frac{\delta \theta}{2}\)
∴ sin²θ \(\frac{\delta \theta}{2}\) = 4(\(\frac{\delta \theta}{2}\))² = (δθ)²
∴ (δθ)² = (δl)² + (δm)² + (δn)².

Inter 1st Year Maths 1B Direction Cosines and Direction Ratios Solutions Ex 6(a)

Practicing the Intermediate 1st Year Maths 1B Textbook Solutions Inter 1st Year Maths 1B Direction Cosines and Direction Ratios Solutions Exercise 6(a) will help students to clear their doubts quickly.

Intermediate 1st Year Maths 1B Direction Cosines and Direction Ratios Solutions Exercise 6(a)

I.

Question 1.
A line makes angles 90°, 60° and 30° with positive directions of X, Y, Z – axes respectively. Find the direction cosines.
Solution:
Suppose l, m, n are the direction cosines of the line.
l = cos α = cos 90° = 0
m = cos β = cos 60° = \(\frac{1}{2}\)
n = cos γ = cos 30° = \(\frac{\sqrt{3}}{2}\)
Direction cosines of the line are (0, \(\frac{1}{2}\), \(\frac{\sqrt{3}}{2}\))

Question 2.
If a line makes angles α, β, γ with the positive direction of x, y, z axes, what is the value of sin² α + sin² β + sin² γ?
Solution:
We know that cos² α + cos² β + cos² γ = 1
1 – sin² α + 1 – sin² β + 1 – sin² γ = 1
sin² a + sin² P + sin² γ = 3 – 1 = 2.

Inter 1st Year Maths 1B Direction Cosines and Direction Ratios Solutions Ex 6(a)

Question 3.
If P(√3, 1, 2√3) is a point in space, find the direction cosines of \(\overrightarrow{O P}\).
Solution:
Direction ratios of P are (√3, 1, 2√3)
a² + b² + c² = 3 + 1 + 12 = 16
⇒ \(\sqrt{a^{2}+b^{2}+c^{2}}\) = 4
Direction cosines of \(\overrightarrow{O P}\) are
Inter 1st Year Maths 1B Direction Cosines and Direction Ratios Solutions Ex 6(a) 1

Question 4.
Find the direction cosines of the line joining the points (-4, 1, 7) are (2, -3, 2).
Solution:
A(- 4, 1, 2) and B(2, -3, 2) are the given points
d.rs of PQ are (x2 – x1, y2 – y1, z2 – z1)
(2 + 4, 1 + 3, 2 – 7) ie., (6, 4, -5)
Dividing with
Inter 1st Year Maths 1B Direction Cosines and Direction Ratios Solutions Ex 6(a) 2

II.

Question 1.
Find the direction cosines of the sides of the triangle whose vertices are (3, 5, -4), (-1, 1, 2) and (-5, -5, -2).
Solution:
A(3, 5, -4), B(-1, 1, 2) and C(-5, -5, -2) are the vertices of ∆ABC
Inter 1st Year Maths 1B Direction Cosines and Direction Ratios Solutions Ex 6(a) 3
d.rs of AB are (-1 -3, 1 – 5, 2 + 4) (-4, -4, 6)
Dividing with \(\sqrt{16+16+36}=\sqrt{68}=2 \sqrt{17}\)
D.Rs of AB are \(\frac{-4}{2 \sqrt{17}}, \frac{-4}{2 \sqrt{17}}, \frac{6}{2 \sqrt{17}}\)
ie., \(\frac{-2}{\sqrt{17}}, \frac{-2}{\sqrt{17}}, \frac{3}{\sqrt{17}}\)
D.Rs of BC are (-5 + 1, -5 -1, -2 -2)
i.e., (-4, -6, -4)

Dividing with \(\sqrt{16+16+36}=\sqrt{68}=2 \sqrt{17}\)
d.cs of BC are \(\frac{-4}{2 \sqrt{17}}, \frac{-6}{2 \sqrt{17}}, \frac{-4}{2 \sqrt{17}}\)
ie., \(\frac{-2}{\sqrt{17}}, \frac{-3}{\sqrt{17}}, \frac{-2}{\sqrt{17}}\)

d.rs of CA are 3 + 5, 5 + 5, -4 + 2
= 8, 10, -2
Dividing with \(\sqrt{64+100+4}=\sqrt{168}=2 \sqrt{42}\)
d.cs of CA are \(\frac{8}{2 \sqrt{42}}, \frac{10}{2 \sqrt{42}}, \frac{-2}{2 \sqrt{42}}\)
ie., \(\frac{4}{\sqrt{42}}, \frac{5}{\sqrt{42}}, \frac{-1}{\sqrt{42}}\)

Inter 1st Year Maths 1B Direction Cosines and Direction Ratios Solutions Ex 6(a)

Question 2.
Show that the lines \(\stackrel{\leftrightarrow}{P Q}\) and \(\stackrel{\leftrightarrow}{R S}\) are parallel where P, Q, R, S are two points (2, 3, 4), (4, 7, 8), (-1, -2, 1) and (1, 2, 5) respectively.
Solution:
P(2, 3, 4), Q(4, 7, 8), R(-1, -2, 1)
and S(1, 2, 5) are the given points.
d.rs of PQ are 4 -2, 7 -3, 8 – 4 i.e., 2, 4, 4
d.rs of RS are 1 + 1, 2 + 2, 5 – 1 i.e., 2, 4, 4
d.rs of PQ are RS are proportional
∴ PQ and RS are parallel.

III.

Question 1.
Find the direction cosines of two lines which are connected by the relations l – 5m + 3n = 0 and 7l² + 5m² – 3n² = 0.
Solution:
Given l – 5m + 3n = 0
⇒ l = 5m – 3n ………….. (1)
7l² + 5m² – 3n² = 0 …………. (2)
Substituting the value of l in (2)
7(5m – 3n)² + 5m² – 3n² = 0
7(25m² + 9n² – 30 mn) + 5m² – 3n² = 0
175 m² + 63n² – 210 mn + 5m² – 3n² = 0
180m² – 210mn + 60n² = 0
Dividing with 30.
6m² – 7mn + 2n² = 0
(3m – 2n) (2m – n) = 0
3m = 2n or 2m = n

Case (i): 3m1 = 2n1 ⇒ \(\frac{m_{1}}{2}=\frac{n_{1}}{3}\)
and m1 = \(\frac{2}{3}\) n1
From (1) l1 = 5m1 – 3n1 = \(\frac{10}{3}\)n1 – 3n1
d.rs of the first line are (1, 2, 3)
Dividing with \(\sqrt{1+4+9}=\sqrt{14}\)
d.cs of the first line are (\(\frac{1}{\sqrt{14}}, \frac{2}{\sqrt{14}}, \frac{3}{\sqrt{14}}\))

Case (ii): 2m2 = n2
From (1) l2 – 5m2 + 3n2 = 0
l2 – 5m2 + 6m2 = 0
l2 = m2
∴ \(\frac{l_{2}}{-1}=\frac{m_{2}}{1}=\frac{n_{2}}{2}\)
d.rs of the second line are -1, 1, 2
Dividing with \(\sqrt{1+1+4}=\sqrt{6}\)
d.cs of the second line are (\(\frac{-1}{\sqrt{6}}, \frac{1}{\sqrt{6}}, \frac{2}{\sqrt{6}}\))

Inter 1st Year Maths 1A Trigonometric Equations Solutions Ex 7(a)

Practicing the Intermediate 1st Year Maths 1A Textbook Solutions Inter 1st Year Maths 1A Trigonometric Equations Solutions Exercise 7(a) will help students to clear their doubts quickly.

Intermediate 1st Year Maths 1A Trigonometric Equations Solutions Exercise 7(a)

I.

Question 1.
Find the principal solutions of the angles in the equations
(i) 2 cos2θ = 1
Solution:
cos2θ = \(\frac{1}{2}\)
θ = 45°, 135°

(ii) √3 sec θ + 2 = 0
Solution:
sec θ = \(\frac{-2}{\sqrt{3}}\)
⇒ cos θ = \(\frac{-\sqrt{3}}{2}\)
⇒ θ = 150°

(iii) 3 tan2θ = 1
Solution:
tan2θ = \(\frac{1}{3}\)
θ = ±\(\frac{\pi}{6}\)

Inter 1st Year Maths 1A Trigonometric Equations Solutions Ex 7(a)

Question 2.
Solve the following equations.
(i) cos 2θ = \(\frac{\sqrt{5}+1}{4}\), θ ∈ [0, 2π]
Solution:
cos 2θ = \(\frac{\sqrt{5}+1}{4}\)
⇒ cos 2θ = cos 36° = cos (\(\frac{\pi}{5}\))
and \(\frac{\pi}{5}\) ∈ [0, 2π]
2θ = \(\frac{\pi}{5}\) ⇒ θ = \(\frac{\pi}{10}\) is the principal solution
and 2θ = 2nπ ± \(\frac{\pi}{5}\) where n ∈ Z is the general solution
⇒ θ = nπ ± \(\frac{\pi}{10}\)
The values of θ in [0, 2π] are \(\left\{\frac{\pi}{10}, \frac{9 \pi}{10}, \frac{11 \pi}{10}, \frac{19 \pi}{10}\right\}\)

(ii) tan2θ = 1, θ ∈ [-π, π]
Solution:
tan2θ = 1 ⇒ tan θ = ±1
tan θ = ±1 = tan (\(\pm \frac{\pi}{4}\))
The principal solutions are θ = \(\pm \frac{\pi}{4}\)
and the general solution is given by nπ ± \(\frac{\pi}{4}\), n ∈ Z
Put n = -1, 0, 1
\(\left\{\frac{-3 \pi}{4}, \frac{-\pi}{4}, \frac{\pi}{4}, \frac{3 \pi}{4}\right\}\) is the solution set for the given equation in [-π, π]

(iii) sin 3θ = \(\frac{\sqrt{3}}{2}\), θ ∈ [-π, π]
Solution:
sin 3θ = \(\frac{\sqrt{3}}{2}\) = sin \(\frac{\pi}{3}\)
and \(\frac{\pi}{3}\) ∈ [-π, π]
∴ 3θ = \(\frac{\pi}{3}\) is the principal solution
and 3θ = \(n \pi+(-1)^{n} \frac{\pi}{3}\), n ∈ Z
⇒ θ = \(\frac{n \pi}{3}+(-1)^{n} \cdot \frac{\pi}{9}\), n ∈ Z is the general solution.
The solution of θ in [-π, π] are \(\left\{\frac{-5 \pi}{9}, \frac{-4 \pi}{9}, \frac{\pi}{9}, \frac{2 \pi}{9}, \frac{7 \pi}{9}, \frac{8 \pi}{9}\right\}\)

(iv) cos2θ = \(\frac{3}{4}\), θ ∈ [0, π]
Solution:
cos2θ = \(\frac{3}{4}\)
⇒ cos θ = ±\(\frac{\sqrt{3}}{4}\)
The general solution is given by
θ = nπ ± \(\frac{\pi}{6}\), n ∈ Z
Put n = 0, 1
The solution set for the given equation in [0, π] is \(\left\{\frac{\pi}{6}, \frac{5 \pi}{6}\right\}\)

(v) 2 sin2θ = sin θ, θ ∈ (0, π)
Solution:
2 sin2θ – sin θ = 0
sin θ (2 sin θ – 1) = 0
sin θ = 0 or sin θ = \(\frac{1}{2}\)
since θ ∈ (0, π)
∴ The solution of θ = \(\left\{\frac{\pi}{6}+\frac{5 \pi}{6}\right\}\)

Inter 1st Year Maths 1A Trigonometric Equations Solutions Ex 7(a)

Question 3.
Find general solutions to the following equations.
(i) sin θ = \(\frac{\sqrt{3}}{2}\), cos θ = \(\frac{-1}{2}\)
Solution:
sin θ = \(\frac{\sqrt{3}}{2}\), cos θ = \(\frac{-1}{2}\)
∵ sin θ is +ve and cos θ is -ve
⇒ θ lies in II quadrant
sin θ = \(\frac{\sqrt{3}}{2}=\sin \left(\frac{2 \pi}{3}\right)\)
cos θ = \(\frac{-1}{2}=\cos \left(\frac{2 \pi}{3}\right)\)
⇒ θ = 2nπ ± \(\frac{2 \pi}{3}\), n ∈ Z is the general solution.

(ii) tan x = \(\frac{-1}{\sqrt{3}}\), sec x = \(\frac{2}{\sqrt{3}}\)
Solution:
∵ tan x = \(\frac{-1}{\sqrt{3}}\) and sec x = \(\frac{2}{\sqrt{3}}\)
⇒ x lies in IV quadrant
tan x = \(\frac{-1}{\sqrt{3}}=\tan \left(\frac{-\pi}{6}\right)\)
sec x = \(\frac{2}{\sqrt{3}}=\sec \left(\frac{-\pi}{6}\right)\)
∴ θ = 2nπ + \(\left(\frac{-\pi}{6}\right)\), n ∈ Z is the general solution.

(iii) cosec θ = -2, cot θ = -√3
Solution:
cosec θ = -2, cot θ = -√3
⇒ θ lies in IV quadrant
cosec θ = -2
⇒ sin θ = \(-\frac{1}{2}=\sin \left(-\frac{\pi}{6}\right)\)
cot θ = -√3
⇒ tan θ = \(-\frac{1}{\sqrt{3}}=\tan \left(-\frac{\pi}{6}\right)\)
∴ θ = 2nπ + \(\left(\frac{-\pi}{6}\right)\), n ∈ Z is the general solution.

Question 4.
(i) If sin (270° – x) = cos 292°, then find x in (0, 360°).
Solution:
sin (270° – x) = cos (292°)
⇒ -cos x = cos (180° + 112°)
⇒ -cos x = -cos 112°
⇒ cos x = cos 112°
⇒ x = 112° or x = 360° – 112° = 248°

(ii) If x < 90°and sin (x + 28°) = cos (3x – 78°), then find x.
Solution:
sin (x + 28°) = cos (3x – 78°)
= sin (90° – 3x + 78°)
= sin (168° – 3x)
x + 28° = 168° – 3x + 28° (180°) or
= 180° – (168° – 3x) + 2x (180°)
⇔ there exists n ∈ Z such that
4x = 140° + 2x (180°)
2x = 16° – 2x (180°)
⇔ there exists n ∈ z such that
x = 35° + x(90°) or x = 8° – x (180°)
Hence x = 8° and x = 35° are the only values of x that lie (0, 90°) and satisfy the given equation.

Inter 1st Year Maths 1A Trigonometric Equations Solutions Ex 7(a)

Question 5.
Find general solutions to the following equations.
(i) 2 sin2θ = 3 cos θ
Solution:
2 sin2θ = 3 cos θ
⇒ 2(1 – cos2θ) = 3 cos θ
⇒ 2 cos2θ + 3 cos θ – 2 = 0
⇒ 2 cos2θ + 4 cos θ – cos θ – 2 = 0
⇒ 2 cos θ (cos θ + 2) – 1 (cos θ + 2) = 0
⇒ (2 cos θ – 1) (cos θ + 2) = 0
⇒ cos θ = \(\frac{1}{2}\) (or) cos θ = -2
∴ The range of cos θ is [-1, 1]
cos θ = -2 is not admissible
∴ cos θ = \(\frac{1}{2}=\cos \frac{\pi}{3}\)
⇒ θ = \(\frac{\pi}{3}\) is the principal solution and
θ = 2nπ ± \(\frac{\pi}{3}\), n ∈ Z is the general solution.

(ii) sin2θ – cos θ = \(\frac{1}{4}\)
Solution:
sin2θ – cos θ = \(\frac{1}{4}\)
⇒ 4(1 – cos2θ) – 4 cos θ = 1
⇒ 4 cos2θ + 4 cos θ – 3 = 0
⇒ 4 cos2θ + 6 cos θ – 2 cos θ – 3 = 0
⇒ 2 cos θ (2 cos θ + 3) – (2 cos θ + 3) = 0
⇒ (2 cos θ – 1) (2 cos θ + 3) = 0
∴ cos θ = \(\frac{1}{2}\) (or) cos θ = \(\frac{-3}{2}\)
∵ The range of cos θ is [-1, 1]
cos θ = \(\frac{-3}{2}\) is not admisable
∴ cos θ = \(\frac{1}{2}=\cos \left(\frac{\pi}{3}\right)\)
θ = \(\frac{\pi}{3}\) is the principal solution and
θ = 2nπ ± \(\frac{\pi}{3}\), n ∈ Z is the general solution.

(iii) 5 cos2θ + 7 sin2θ = 6
Solution:
5 cos2θ + 7 sin2θ = 6
Dividing by cos2θ
⇒ 5 + 7 tan2θ = 6 sec2θ
⇒ 5 + 7 tan2θ = 6(1 + tan2θ)
⇒ tan2θ = 1
⇒ tan θ = ±1
∴ θ = nπ ± \(\frac{\pi}{4}\), n ∈ Z is the general solution.

(iv) 3 sin4x + cos4x = 1
Solution:
3 sin4x + cos4x = 1
⇒ 3 sin4x + (cos2x)2 = 1
⇒ 3 sin4x + (1 – sin2x)2 = 1
⇒ 3 sin4x + 1 + sin4x – 2 sin2x = 1
⇒ 4 sin4x – 2 sin2x = 0
⇒ 2 sin2x (2 sin2x – 1) = 0
⇒ sin x = 0 (or) sin x = ±\(\frac{1}{\sqrt{2}}\)
If sin x = 0
⇔ x = nπ, n ∈ Z is the general solution.
If sin x = ±\(\frac{1}{\sqrt{2}}\)
⇒ x = nπ ± \(\frac{\pi}{4}\), n ∈ Z is the general solution.
∴ General solution is x = nπ (or) nπ ± \(\frac{\pi}{4}\), n ∈ Z.

Inter 1st Year Maths 1A Trigonometric Equations Solutions Ex 7(a)

II.

Question 1.
Solve the following equations and write a general solution.
(i) 2 sin2θ – 4 = 5 cos θ
Solution:
2(1 – cos2θ) – 4 = 5 cos θ
2 – 2cos2θ – 4 = 5 cos θ
2 cos2θ + 5 cos θ + 2 = 0
2 cos2θ + 4 cos θ + cos θ + 2 = 0
2 cos θ (cos θ + 2) + 1 (cos θ + 2) = 0
(cos θ + 2) (2 cos θ + 1) = 0
cos θ = -2 or cos θ = \(-\frac{1}{2}\)
cos θ = -2 is not possible
∴ cos θ = \(-\frac{1}{2}\) = cos \(\frac{2 \pi}{3}\)
∴ General solution is θ = 2nπ ± \(\frac{2 \pi}{3}\), n ∈ Z

(ii) 2 + √3 sec x – 4 cos x = 2√3
Solution:
2 + √3 sec x – 4 cos x = 2√3
\(\frac{2 \cos x+\sqrt{3}-4 \cos ^{2} x}{\cos x}\) = 2√3
2 cos x + √3 – 4 cos2x = 2√3 cos x
4 cos2x + 2√3 cos x – 2 cos x – √3 = 0
2 cos x (2 cos x + √3) – 1 (2 cos x + √3) = 0
(2 cos x – 1) (2 cos x + √3) = 0
cos x = \(\frac{1}{2}\) (or) cos x = \(\frac{-\sqrt{3}}{2}\)
If cos x = \(\frac{1}{2}\) = cos \(\frac{\pi}{3}\), n ∈ Z
General solution x = 2nπ ± \(\frac{\pi}{3}\)
If cos x = \(\frac{-\sqrt{3}}{2}\) = cos \(\frac{5 \pi}{3}\)
General solution x = 2nπ ± \(\frac{5 \pi}{3}\), n ∈ Z

(iii) 2 cos2θ + 11 sin θ = 7
Solution:
2 (1 – sin2θ) + 11 sin θ = 7
2 – 2 sin2θ + 11 sin θ = 7
2 sin2θ – 11 sin θ + 5 = 0
2 sin2θ – 10 sin θ – sin θ + 5 = 0
2 sin θ (sin θ – 5) – 1(sin θ – 5) = 0
(sin θ – 5) (2 sin θ – 1) = 0
sin θ = 5 or sin θ = \(\frac{1}{2}\)
If sin θ = 5 is not possible
∴ sin θ = \(\frac{1}{2}\) = sin \(\frac{\pi}{6}\)
General solultion θ = nπ + (-1)n \(\frac{\pi}{6}\), n ∈ Z

Inter 1st Year Maths 1A Trigonometric Equations Solutions Ex 7(a)

(iv) 6 tan2x – 2 cos2x = cos 2x
Solution:
6 tan2x – 2 cos2x = cos 2x
⇒ 6(sec2x – 1) – 2 cos2x = 2 cos2x – 1
⇒ 6 sec2x – 6 – 4 cos2x + 1 = 0
⇒ 6 sec2x – 4 cos2x – 5 = 0
⇒ \(\frac{6}{\cos ^{2} x}\) – 4 cos2x – 5 = 0
⇒ 6 – 4 cos4x – 5 cos2x = 0
⇒ 4 cos4x + 5 cos2x – 6 = 0
⇒ 4 cos4x + 8 cos2x – 3 cos2x – 6 = 0
⇒ 4 cos2x (cos2x + 2) – 3(cos2x + 2) = 0
⇒ (4 cos2x – 3) (cos2x + 2) = 0
⇒ 4 cos2x = 3, cos2x ≠ -2
⇒ cos x = ±\(\frac{\sqrt{3}}{2}\)
∴ x = nπ ± \(\frac{\pi}{6}\), n ∈ Z is the general solution.

(v) 4 cos2θ + √3 = 2(√3 + 1) cos θ
Solution:
4 cos2θ – 2(√3 + 1) cos θ + √3 = 0
⇒ 4 cos2θ – 2√3 cos θ – 2 cos θ + √3 = 0
⇒ 2 cos θ (2 cos θ – √3) – 1(2 cos θ – √3) = 0
⇒ (2 cos θ – 1) (2 cos θ – √3) = 0
⇒ cos θ = \(\frac{1}{2}\) (or) cos θ = \(\frac{\sqrt{3}}{2}\)
If cos θ = \(\frac{1}{2}\) = cos \(\left(\frac{\pi}{3}\right)\)
∴ θ = 2nπ ± \(\frac{\pi}{3}\), n ∈ Z is the general solution.
If cos θ = \(\frac{\sqrt{3}}{2}\) = cos \(\left(\frac{\pi}{6}\right)\)
∴ θ = 2nπ ± \(\frac{\pi}{6}\), n ∈ Z is the general solution.

(vi) 1 + sin 2x – (sin 3x – cos 3x)2
Solution:
1 + sin 2x = (sin 3x – cos 3x)2
⇒ 1 + sin 2x = sin23x + cos23x – 2 sin 3x cos 3x
⇒ 1 + sin 2x = 1 – sin (2 × 3x)
⇒ sin 6x + sin 2x = 0
⇒ \(2 \sin \left(\frac{6 x+2 x}{2}\right) \cdot \cos \left(\frac{6 x-2 x}{2}\right)=0\)
⇒ sin (4x) . cos (2x) = 0
⇒ cos 2x = 0 (or) sin 4x = 0
If cos 2x = 0 = cos \(\frac{\pi}{2}\)
⇒ 2x = \(\frac{\pi}{2}\) is the principal solution and
2x = (2n + 1) \(\frac{\pi}{2}\), n ∈ Z is the general solution,
so that x = \(\frac{n \pi}{2}+\frac{\pi}{4}\), n ∈ Z
If sin 4x = 0 = sin(nπ), n ∈ Z
4x = nπ, n ∈ Z is the general solution.
So that 4x = nπ
⇒ x = \(\frac{n \pi}{4}\), n ∈ Z
∴ x = \(\frac{n \pi}{4}\); \(\frac{n \pi}{2}+\frac{\pi}{4}\), n ∈ Z is the general solution.

(vii) 2 sin2x + sin22x = 2
Solution:
2 sin2x + sin2(2x) = 2
⇒ 2 sin2x + (2 sin x cos x)2 – 2 = 0
⇒ sin2x + 2 sin2x cos2x – 1 = 0
⇒ 2 sin2x cos2x – (1 – sin2x) = 0
⇒ 2 sin2x cos2x – cos2x = 0
⇒ cos2x (2 sin2x – 1) = 0
⇒ cos x = 0 (or) sin x = ±\(\frac{1}{\sqrt{2}}\)
If cos x = 0 = cos \(\frac{\pi}{2}\)
⇒ x = \(\frac{\pi}{2}\) is the principal solution
and cos x = 0 ⇔ \(\sin \left(x-\frac{\pi}{2}\right)=0\)
⇔ x – \(\frac{\pi}{2}\) = nπ, n ∈ Z
⇔ x = nπ + \(\frac{\pi}{2}\)
⇔ x = (2n + 1) \(\frac{\pi}{2}\), n ∈ Z
∴ x = (2n + 1) \(\frac{\pi}{2}\), n ∈ Z is the general solution of cos x = 0
If sin x = ±\(\frac{1}{\sqrt{2}}\) = sin(±\(\frac{\pi}{4}\))
x = ±\(\frac{\pi}{4}\) are principal solutions
and x = nπ ± \(\frac{\pi}{4}\), n ∈ Z is the general solution
∴ The general solutions are \(\left[\left\{(2 n+1) \frac{\pi}{2}\right\},\left\{n \pi \pm \frac{\pi}{4}\right\} n \in Z\right]\)

Inter 1st Year Maths 1A Trigonometric Equations Solutions Ex 7(a)

Question 2.
Solve the following equations.
(i) √3 sin θ – cos θ = √2
Solution:
Inter 1st Year Maths 1A Trigonometric Equations Solutions Ex 7(a) II Q2(i)

(ii) cot x + cosec x = √3
Solution:
Inter 1st Year Maths 1A Trigonometric Equations Solutions Ex 7(a) II Q2(ii)
Inter 1st Year Maths 1A Trigonometric Equations Solutions Ex 7(a) II Q2(ii).1

(iii) sin x + √3 cos x = √2
Solution:
Inter 1st Year Maths 1A Trigonometric Equations Solutions Ex 7(a) II Q2(iii)

Question 3.
Solve the following equations:
(i) tan θ + sec θ = √3, 0 ≤ θ ≤ 2π
Solution:
Inter 1st Year Maths 1A Trigonometric Equations Solutions Ex 7(a) II Q3(i)
Inter 1st Year Maths 1A Trigonometric Equations Solutions Ex 7(a) II Q3(i).1

(ii) cos 3x + cos 2x = sin \(\frac{3 x}{2}\) + sin \(\frac{x}{2}\), 0 ≤ x ≤ 2π
Solution:
Inter 1st Year Maths 1A Trigonometric Equations Solutions Ex 7(a) II Q3(ii)
Inter 1st Year Maths 1A Trigonometric Equations Solutions Ex 7(a) II Q3(ii).1

(iii) cot2x – (√3 + 1) cot x + √3 = 0, 0 < x < \(\frac{\pi}{2}\)
Solution:
cot2x – (√3 + 1) cot x + √3 = 0
⇔ cot2x – √3 cot x – cot x + √3 = 0
⇔ cot x (cot x – √3) – 1(cot x – √3) = 0
⇔ cot x = √3 (or) cot x = 1
case (i): cot x = 1 ⇒ tan x = 1
∴ x = \(\left\{\frac{\pi}{4}\right\}\)
case (ii): cot x = √3 ⇒ tan x = \(\frac{1}{\sqrt{3}}\)
∴ x = \(\left\{\frac{\pi}{6}\right\}\)
∴ Solutions are \(\left\{\frac{\pi}{6}, \frac{\pi}{4}\right\}\)

Inter 1st Year Maths 1A Trigonometric Equations Solutions Ex 7(a)

(iv) sec x . cos 5x + 1 = 0; 0 < x < 2π
Solution:
Inter 1st Year Maths 1A Trigonometric Equations Solutions Ex 7(a) II Q3(iv)

III.

Question 1.
(i) Solve sin x + sin 2x + sin 3x = cos x + cos 2x + cos 3x
Solution:
(sin 3x + sin x) + sin 2x = (cos 3x + cos x) + cos 2x
⇔ 2 . sin(\(\frac{3 x+x}{2}\)) . cos (\(\frac{3 x-x}{2}\)) + sin 2x = 2 . cos (\(\frac{3 x+x}{2}\)) . cos (\(\frac{3 x-x}{2}\)) + cos 2x
⇔ 2 sin 2x cos x + sin 2x = 2 cos 2x cos x + cos 2x
⇔ sin 2x (2 cos x + 1) = cos 2x (2 cos x + 1)
⇔ (2 cos x + 1) (sin 2x – cos 2x ) = 0
⇔ cos x = \(-\frac{1}{2}\) (or) sin 2x = cos 2x (i.e.,) tan (2x) = 1
Case (i):
cos x = \(-\frac{1}{2}\) = cos (\(\frac{2 \pi}{3}\))
Principal solution is x = \(\frac{2 \pi}{3}\)
and General solution is x = 2nπ ± \(\frac{2 \pi}{3}\), n ∈ z
Case (ii):
tan 2x = 1 = tan \(\frac{\pi}{4}\)
∴ Principal solution is 2x = \(\frac{\pi}{4}\) (i.e.,) x = \(\frac{\pi}{2}\)
General solution is 2x = nπ + \(\frac{\pi}{4}\), n ∈ z
⇒ x = \(\frac{\mathrm{n} \pi}{2}+\frac{\pi}{8}\), n ∈ Z
∴ General solution is \(\left.\left[\left\{2 n \pi \pm \frac{2 \pi}{3}\right\},\left\{\frac{n \pi}{2}+\frac{\pi}{8}\right\} / n \in Z\right\}\right]\)

(ii) If x + y = \(\frac{2 \pi}{3}\) and sin x + sin y = \(\frac{3}{2}\), find x and y.
Solution:
Inter 1st Year Maths 1A Trigonometric Equations Solutions Ex 7(a) III Q1(ii)

(iii) If sin 3x + sin x + 2 cos x = sin 2x + 2 cos2x, find the general solution.
Solution:
Given sin 3x + sin x + 2 cos x = sin 2x + 2 cos2x
⇒ 2 . sin (\(\frac{3 x+x}{2}\)) . cos (\(\frac{3 x-x}{2}\)) + 2 cos x = 2 sin x cos x + 2 cos2x
⇒ 2 . sin 2x . cos x + 2 cos x = 2 cos x (sin x + cos x)
⇒ 2 cos x (sin 2x + 1) = 2 cos x (sin x + cos x)
⇒ 2 cos x [sin 2x + 1 – sin x – cos x] = 0
⇒ cos x = 0 (or) sin 2x – sin x + 1 – cos x = 0
Inter 1st Year Maths 1A Trigonometric Equations Solutions Ex 7(a) III Q1(iii)
Inter 1st Year Maths 1A Trigonometric Equations Solutions Ex 7(a) III Q1(iii).1

(iv) Solve cos 3x – cos 4x = cos 5x – cos 6x
Solution:
-2 sin 5x . sin x = -2 sin 4x . sin x
⇒ 2 sin x [sin 5x – sin 4x] = 0
⇒ 4 sin x . cos \(\frac{9 x}{2}\) . sin \(\frac{x}{2}\) = 0
Inter 1st Year Maths 1A Trigonometric Equations Solutions Ex 7(a) III Q1(iv)

Question 2.
Solve the following equations.
(i) cos 2θ + cos 8θ = cos 5θ
Solution:
cos 2θ + cos 8θ = cos 5θ
2 cos (\(\frac{2 \theta+8 \theta}{2}\)) cos (\(\frac{2 \theta-8 \theta}{2}\)) – cos 5θ = 0
2 cos 5θ . cos 3θ – cos 5θ = 0
cos 5θ (2 cos 3θ – 1) = 0
If cos 5θ = 0
Solution is 5θ = (2n + 1) \(\frac{\pi}{2}\)
θ = (2n + 1) \(\frac{\pi}{10}\), n ∈ z
If 2 cos 3θ – 1 = 0
cos 3θ = \(\frac{1}{2}\) = cos \(\frac{\pi}{3}\)
Solution is 3θ = 2nπ ± \(\frac{\pi}{3}\)
θ = \(\frac{2 n \pi}{3} \pm \frac{\pi}{9}\), n ∈ Z

(ii) cos θ – cos 7θ = sin 4θ
Solution:
-2 sin (\(\frac{\theta+7 \theta}{2}\)) sin (\(\frac{\theta-7 \theta}{2}\)) – sin 4θ = 0
2 sin 4θ sin 3θ – sin 4θ = 0
sin 4θ (2 sin 3θ – 1) = 0
If sin 4θ = 0
∴ Solution is 4θ = nπ
θ = \(\frac{n \pi}{4}\), n ∈ z
If 2 sin 3θ – 1 = 0
sin 3θ = \(\frac{1}{2}\) = sin \(\frac{\pi}{6}\)
Solution is 3θ = nπ + (-1)n \(\frac{\pi}{6}\)
θ = \(\frac{n \pi}{3}+(-1)^{n} \frac{\pi}{18}\), n ∈ z

Inter 1st Year Maths 1A Trigonometric Equations Solutions Ex 7(a)

(iii) sin θ + sin 5θ = sin 3θ, 0 < θ < π
Solution:
sin θ + sin 5θ = sin 3θ
sin θ + sin 5θ – sin 3θ = 0
sin θ + 2 cos (\(\frac{5 \theta+3 \theta}{2}\)) sin (\(\frac{5 \theta-3 \theta}{2}\)) = 0
sin θ + 2 cos 4θ . sin θ = 0
sin θ (1 + 2 cos 4θ) = 0
sin θ = 0, cos 4θ = \(\frac{-1}{2}\)
If sin θ = 0, solution is θ = nπ, n ∈ Z
If cos 4θ = \(\frac{-1}{2}\) = cos(\(\frac{2 \pi}{3}\))
Solution is 4θ = 2nπ ± \(\frac{2 \pi}{3}\)
θ = \(\frac{2 n \pi}{4} \pm \frac{2 \pi}{12}\), n ∈ Z
θ = \(\frac{n \pi}{2} \pm \frac{\pi}{6}\), n ∈ Z
Since 0 < θ < π
Then π = \(\frac{\pi}{6}, \frac{\pi}{3}, \frac{2 \pi}{3}, \frac{5 \pi}{6}\)

Question 3.
(i) If tan pθ = cot qθ and p ≠ -q show that the solutions are in A.P. with common difference \(\frac{\pi}{p+q}\)
Solution:
tan pθ = cot qθ = tan \(\frac{\pi}{2}\) – qθ
pθ = nπ + \(\frac{\pi}{2}\) – qθ
(p + q) θ = (2n + 1) \(\frac{\pi}{2}\)
θ = \(\frac{(2 n+1)}{p+q} \frac{\pi}{2}\), n is an integer
The Solutions \(\frac{\pi}{2(p+q)}, \frac{3 \pi}{2(p+q)}, \frac{5 \pi}{2(p+q)}\) + …………
∴ The solution form an Arithematical proportion With common difference \(\frac{2 \pi}{2(p+q)}=\frac{\pi}{p+q}\)

(ii) Show that the solutions of cos pθ = sin qθ form two series each of which is an A.P. Find also the common difference of each A.P. (p ≠ ±q).
Solution:
cos pθ = sin qθ
cos pθ – sin qθ = 0
cos pθ + \(\cos \left[\frac{\pi}{2}+9 \theta\right]\) = 0
Inter 1st Year Maths 1A Trigonometric Equations Solutions Ex 7(a) III Q3(ii)

(iii) Find the number of solutions of the equation tan x + sec x = 2 cos x; cos x ≠ 0, lying in the interival (0, π).
Solution:
tan x + sec x = 2 cos x
\(\frac{\sin x}{\cos x}+\frac{1}{\cos x}\) = 2 cos x
sin x + 1 = 2 cos2x
sin x + 1 = 2(1 – sin2x)
sin x + 1 = (2 – 2 sin2x)
2 sin2x + sin x – 1 = 0
2 sin2x + 2 sin x – sin x – 1 = 0
2 sin x (sin x + 1) – 1 (sin x + 1) = 0
(sin x + 1)(2 sin x – 1) = 0
sin x = -1 (or) sin x = \(\frac{1}{2}\)
If sin x = -1
x = \(\frac{-\pi}{2} \text { (or) } \frac{3 \pi}{2}\)
If sin x = \(\frac{1}{2}\)
x = \(\frac{\pi}{6} \text { (or) } \frac{5 \pi}{6}\)
In the interval (0, π)
Number of solutions = 2

(iv) Solve sin 3α = 4 sin α sin(x + α) sin(x – α) where α ≠ nπ, n ∈ Z
Solution:
3 sin α – 4 sin3α = 4 sin α (sin2x – sin2α)
Dividing with sin α
3 – 4 sin2α = 4 (sin2x – sin2α)
3 – 4 sin2α = 4 sin2x – 4 sin2α
4 sin2x = 3
2 sin2x = \(\frac{3}{2}\)
1 – cos 2x = \(\frac{3}{2}\)
cos 2x = \(-\frac{1}{2}=\cos \frac{2 \pi}{3}\)
2x = 2nπ ± \(\frac{2 \pi}{3}\), ∀ n ∈ Z
x = nπ ± \(\frac{\pi}{3}\), n ∈ Z

Inter 1st Year Maths 1A Trigonometric Equations Solutions Ex 7(a)

Question 4.
(i) If tan(π cos θ) = cot(π sin θ), then prove that \(\cos \left(\theta-\frac{\pi}{4}\right)=\pm \frac{1}{2 \sqrt{2}}\)
Solution:
Inter 1st Year Maths 1A Trigonometric Equations Solutions Ex 7(a) III Q4(i)

(ii) Find the range of θ if cos θ + sin θ is positive.
Solution:
Inter 1st Year Maths 1A Trigonometric Equations Solutions Ex 7(a) III Q4(ii)

Question 5.
If α, β are the solutions of the equation a cos θ + b sin θ = c, where a, b, c ∈ R and if a2 + b2 > 0, cos α ≠ cos β and sin α ≠ sin β, then show that
(i) sin α + sin β = \(\frac{2 b c}{a^{2}+b^{2}}\)
(ii) cos α + cos β = \(\frac{2 a c}{a^{2}+b^{2}}\)
(iii) cos α . cos β = \(\frac{c^{2}-b^{2}}{a^{2}+b^{2}}\)
(iv) sin α . sin β = \(\frac{c^{2}-a^{2}}{a^{2}+b^{2}}\)
Solution:
a cos θ = b sin θ = c
First write this as a quadratic equation in sin θ
⇔ a cos θ = c – b sin θ
By squaring on both sides, we get
a2 cos2θ = (c – b sin θ)2
⇔ a2 (1 – sin2θ) = c2 + b2 sin2θ – 2 bc sin θ
⇔ (a2 + b2) sin2θ – 2bc sin θ + (c2 – a2) = 0
It is a quadratic equation in sin θ,
It has sin α and sin β as roots since α and β are solutions for the given equation
(i) Sum of the roots = sin α + sin β = \(\frac{2 b c}{a^{2}+b^{2}}\)
Again a cos θ + b sin θ = c
Write this as a quadratic equation in cos θ
⇔ b sin θ = c – a cos θ
By squaring on both sides
⇔ b2 sin2θ = (c – a cos θ)2
⇔ b2(1 – cos2θ) = c2 + a2 cos2θ – 2 ca cos θ
⇔ (a2 + b2) cos2θ – 2 ca cos θ + (c2 – b2) = 0
It is a quadratic equation in cos θ. It has cos α, cos β be its roots.
(ii) Sum of the roots = cos α + cos β = \(\frac{2 c a}{a^{2}+b^{2}}\)
(iii) Product of the roots = cos α . cos β = \(\frac{c^{2}-b^{2}}{a^{2}+b^{2}}\)
(iv) Product of the roots = sin α . sin β = \(\frac{c^{2}-a^{2}}{a^{2}+b^{2}}\)

Inter 1st Year Maths 1A Trigonometric Equations Solutions Ex 7(a)

Question 6.
(i) Find the common roots of the equations cos 2x + sin 2x = cot x and 2 cos2x + cos22x = 1.
Solution:
Let tan x = A
Given that cos 2x + sin 2x = cot x
⇔ \(\frac{1-\tan ^{2} x}{1+\tan ^{2} x}+\frac{2 \tan x}{1+\tan ^{2} x}=\frac{1}{\tan x}\)
⇔ \(\frac{1-A^{2}}{1+A^{2}}+\frac{2 A}{1+A^{2}}=\frac{1}{A}\)
⇔ (1 – A2 + 2A) A = (1 + A2)
⇔ A – A3 + 2A2 = 1 + A2
⇔ A3 – A2 – A + 1 = 0
⇔ A = 1
A = 1 satisfy this equation
Inter 1st Year Maths 1A Trigonometric Equations Solutions Ex 7(a) III Q6(i)
∴ A3 – A2 – A + 1 = 0
⇔ (A – 1) (A2 – 1) = 0
⇔ (A – 1) (A – 1) (A + 1) = 0
⇔ A = 1, A = -1
∴ tan x = ±1
⇒ x = (2n + 1) \(\frac{\pi}{4}\), n ∈ z
Given 2 cos2x + cos2 2x = 1
⇔ (2 cos2x – 1) + cos2(2x) = 0
⇔ cos 2x + cos2(2x) = 0
⇔ cos 2x (1 + cos 2x) = 0
⇔ cos 2x = 0 (or) cos 2x = -1
Case (i): cos 2x = 0
⇔ 2x = (2n + 1) \(\frac{\pi}{2}\)
∴ x = (2n + 1) \(\frac{\pi}{4}\), n ∈ z
∴ (2n + 1) \(\frac{\pi}{4}\), n ∈ z is the common root for the given two trigonometric equations.

(ii) Solve the equation \(\sqrt{6-\cos x+7 \sin ^{2} x}+\cos x=0\)
Solution:
\(\sqrt{6-\cos x+7 \sin ^{2} x}+\cos x=0\)
6 – cos x + 7 sin2x ≥ 0
⇒ 7(1 – cos2x) – cos x + 6 ≥ 0
⇒ 7 – 7 cos2x – cos x + 6 ≥ 0
⇒ 7 cos2x + cos x – 13 ≤ 0
Consider 7 cos2x + cos x – 13 = 0
Inter 1st Year Maths 1A Trigonometric Equations Solutions Ex 7(a) III Q6(ii)
cos x values do not lie in [-1, 1]
Hence the given equation has no solution.

(iii) If |tan x| = tan x + \(\frac{1}{\cos x}\) and x ∈ [0, 2π], find the value of x.
Solution:
Case (i):
|tan x| = tan x, if x lies either in I (or) in III quadrant
Then |tan x| = tan x + \(\frac{1}{\cos x}\)
⇒ tan x = tan x + sin x
⇒ sec x = 0 which is impossible, since sec x ∉ (-1, 1)
Case (ii):
|tan x| = -tan x, if x lies in II & IV quadrants
Then |tan x| = tan x + \(\frac{1}{\cos x}\)
⇒ -tan x = tan x + sec x
⇒ -2 tan x = sec x
⇒ \(-2 \frac{\sin x}{\cos x}-\frac{1}{\cos x}\) = 0
⇒ -2 sin x – 1 = 0
⇒ sin x = \(\frac{-1}{2}=\sin \left(\frac{-\pi}{6}\right)=\sin \left(2 \pi-\frac{\pi}{6}\right)\)
∴ x = \(\frac{11 \pi}{6}\)

Inter 1st Year Maths 1B Three Dimensional Coordinates Solutions Ex 5(b)

Practicing the Intermediate 1st Year Maths 1B Textbook Solutions Inter 1st Year Maths 1B Three Dimensional Coordinates Solutions Exercise 5(b) will help students to clear their doubts quickly.

Intermediate 1st Year Maths 1B Three Dimensional Coordinates Solutions Exercise 5(b)

I.

Question 1.
Find the ratio in which the XZ-plane divides the line joining A(-2, 3, 4) and B(l, 2,3).
Solution:
Ratio in which XZ plane divides
AB = -y1 : y2
= -3 : 2

Question 2.
Find the co-ordinates of the vertex ‘C’ of ∆ABC if its centroid is the origin and the vertices A, B are (1, 1, 1) and (-2, 4, 1) respectively.
Solution:
A(1, 1, 1) B(-2, 4, 1) and (x, y, z) are the vertices of ∆ABC.
G is the centroid of ∆ABC
Co-ordinates of G are
(\(\frac{1-2+x}{3}\), \(\frac{1+4+y}{3}\), \(\frac{1+1+z}{3}\)) = (0, 0, 0)
\(\frac{x-1}{3}\) = 0, \(\frac{y+5}{3}\) = 0, \(\frac{z+2}{3}\) = 0
x – 1 = 0, y +5 =0, z + 2 = 0
x = 1, y = – 5, z = -2
∴ Co-ordinates of c are (1, -5, -2)

Inter 1st Year Maths 1B Three Dimensional Coordinates Solutions Ex 5(b)

Question 3.
If (3, 2, -1), (4, 1, 1) and (6, 2, 5) are three vertices and (4, 2, 2) is the centroid of a tetrahedron, find the fourth vertex.
Solution:
A(3, 2, -1), B(4, 1, 1), C(6, 2, 5), D(x, y, z) are the vertices of the tetrahedron.
Co-ordinates of the centroid G are
Inter 1st Year Maths 1B Three Dimensional Coordinates Solutions Ex 5(b) 1
Co-ordinates of D are (3, 3, 3)

Question 4.
Find the distance between the mid point of the line segment \(\overline{\mathrm{A B}}\) and the point (3, -1, 2) where A = (6, 3, -4) and B = (-2, -1, 2).
Solution:
A(6, 3, – 4) B(-2, -1,2) are the given points.
Q is the midpoint of AB.
Co-ordinates of Q are (\(\frac{6-2}{2}\), \(\frac{3-1}{2}\), \(\frac{-4+2}{2}\)) = (2, 1, -1)
Co-ordinates of P are (3, -1, 2)
Inter 1st Year Maths 1B Three Dimensional Coordinates Solutions Ex 5(b) 2

II.

Question 1.
Show that the points (5, 4,2) (6,2, -1) and (8, -2, -7) are collinear.
Solution:
A(5, 4, 2), B(6, 2, -1) c(8, -2, -7) are the given points.
AB = \(\sqrt{(5-6)^{2}+(4-2)^{2}+(2+1)^{2}}\)
Inter 1st Year Maths 1B Three Dimensional Coordinates Solutions Ex 5(b) 3
∴ A, B, C are collinear.

Question 2.
Show that the points A(3, 2, -4), B(5, 4, -6) and C(9, 8, -10) are collinear and find the ratio in which B divides \(\overline{\mathrm{A C}}\).
Solution:
A(3, 2, -4), B(5, 4, -6) and C(9, 8, -10) are the given points.
Inter 1st Year Maths 1B Three Dimensional Coordinates Solutions Ex 5(b) 4
A, B, C are collinear
Ratio in which B divides AC = AB : BC
= 2√3 : 4√3 = 1 : 2

III.

Question 1.
If A(4, 8, 12) B(2, 4, 6) C(3, 5, 4) and D(5, 8, 5) are four points, show that the line \(\stackrel{\leftrightarrow}{A B}\) and \(\stackrel{\leftrightarrow}{C D}\) intersect.
Solution:
A(4, 8, 12), B(2, 4, 6) C(3, 5, 4) and D(5, 8, 5) are the given points
Co-ordinates of the point dividing AB in the ratio λ : 1 is
\(\left[\frac{2 \lambda+4}{\lambda+1} \cdot \frac{4 \lambda+8}{\lambda+1} \cdot \frac{6 \lambda+12}{\lambda+1}\right]\) ……….. (1)
Co-ordinates of the point dividing CD in the ratio p : 1 is
\(\left[\frac{5 \mu+3}{\mu+1}, \frac{8 \mu+5}{\mu+1}, \frac{5 \mu+4}{\mu+1}\right]\) ……….. (2)
If the given lines intersects these two points must coincide.
\(\frac{2 \lambda+4}{\lambda+1}=\frac{5 \mu+3}{\mu+1}\)
(2λ + 4) (µ + 1) = (5µ + 3) (λ + 1)
2λµ + 2λ + 4µ + 4 = 5λµ + 5µ + 3λ + 3
3λ + λ + µ – 1 = 0
λ(3µ+1) = -(µ – 1)
λ = –\(\frac{(\mu-1)}{3 \mu+1}\)
\(\frac{4 \lambda+8}{\lambda+1}=\frac{8 \mu+5}{\mu+1}\)
(4λ + 8) (µ + 1) = (8µ + 5) (λ + 1)
4λµ + 4λ + 8µ + 8 = 8λµ + 8µ + 5λ + 5
4Aµ + λ – 3 = 0
(4µ + 1) λ = 3
\(-\frac{(4 \mu+1)(\mu-1)}{3 \mu+1}=3\)
4µ2 – 4µ + µ – 1 = -9µ – 3
4µ2 4- 6µ + 2 = 0
2µ2 + 3µ + 1=0
(2µ + 1) (µ – 1) = 0
µ = – \(\frac{1}{2}\) or – 1
µ = -1 is not possible
µ = – \(\frac{1}{2}\)
Inter 1st Year Maths 1B Three Dimensional Coordinates Solutions Ex 5(b) 5
Co-ordinates of (2) are
Inter 1st Year Maths 1B Three Dimensional Coordinates Solutions Ex 5(b) 6
Co-ordinates of (1) are
\(\frac{-6+4}{-3+1}, \frac{-12+8}{-3+1}, \frac{-18+12}{-3+1}\) = (1, 2, 3)
Since these two points coincide the given lines intersect.

Inter 1st Year Maths 1B Three Dimensional Coordinates Solutions Ex 5(b)

Question 2.
Find the point of instersection of the lines \(\stackrel{\leftrightarrow}{A B}\) and \(\stackrel{\leftrightarrow}{C D}\) where A = (7, -6, 1) B = (17, -18, -3), C = (1, 4, -5) and D = (3, -4, 11)
Inter 1st Year Maths 1B Three Dimensional Coordinates Solutions Ex 5(b) 7
Solution:
A(7, -6, 1), B(17, -18, -3), C(1, 4, -5) and D(3, -4, 11) are the given points.
Co-ordinates of the point dividing AB in the ratio λ: 1 are
Inter 1st Year Maths 1B Three Dimensional Coordinates Solutions Ex 5(b) 8
Co-ordinates of the point dividing CD in the ratio µ : 1 are
Inter 1st Year Maths 1B Three Dimensional Coordinates Solutions Ex 5(b) 9
(17λ + 7) (µ + 1) = (3µ + 1) (λ + 1)
17λµ + 17λ + 7µ + 7 = 3λµ + 3µ + λ + 1
14λµ + 16λ + 4µ + 6 = 0.
Inter 1st Year Maths 1B Three Dimensional Coordinates Solutions Ex 5(b) 10
-18λµ – 6µ – 18λ – 6
= -4λµ + 4λ – 4µ + 4
14λµ + 22λ + 2µ + 10 = 0
14λµ + 16λ + 4µ + 6 = 0 ………. (1)
14λµ + 22λ + 2µ + 10 = 0……….(2)
Subtraction – 6λ + 2µ – 4 = 0
2µ = 6λ + 4
µ = 3λ + 2
Substituting in (3)
14λ(3λ + 2) + 16λ + 4(3λ + 2) + 6 = 0
42λ² + 28λ + 16λ + 12λ+ 8 + 6 = 0
42λ² + 56λ + 14 = 0
3λ² + 4λ + 1 = 0
(λ + 1) (3λ + 1) = 0
λ = -1 or λ = –\(\frac{1}{3}\)
λ = -1 is not possible
λ = –\(\frac{1}{3}\)
Inter 1st Year Maths 1B Three Dimensional Coordinates Solutions Ex 5(b) 11
µ = 3λ + 2 = -1 + 2 = 1
µ = 1 ⇒
\(\frac{3+1}{1+1}, \frac{-4+4}{1+1}, \frac{11-5}{1+1}\) = (2, 0, 3)
∴ These two points coincide
⇒ The given lines AB and CD intersect
Point of intersection is (2, 0, 3)

Question 3.
A(3, 2, 0), B(5, 3, 2), C(-9, 6, -3) are vertices of a triangle. \(\overline{\mathrm{A D}}\), the bisector of ∠BAC meets \(\overline{\mathrm{B C}}\) at D. Find the co-ordinates of D.
Solution:
A(3, 2, 0),B(5, 3, 2) C(-9, 6, -3) are the vertices of ∆ABC
Inter 1st Year Maths 1B Three Dimensional Coordinates Solutions Ex 5(b) 12
Inter 1st Year Maths 1B Three Dimensional Coordinates Solutions Ex 5(b) 13
AB is the bisector of ∠DAC
‘D’ divides BC in the ratio 3 : 13
Co-ordinates of D are
Inter 1st Year Maths 1B Three Dimensional Coordinates Solutions Ex 5(b) 14

Inter 1st Year Maths 1B Three Dimensional Coordinates Solutions Ex 5(b)

Question 4.
Show that the points 0(0,0,0), A(2, -3, 3), B(-2, 3, -3) are collinear. Find the ratio in which each point divides the segment joining the other two.
Solution:
0(0, 0, 0), A(2, -3, 3), B(-2, 3, -3) are the given points.
Inter 1st Year Maths 1B Three Dimensional Coordinates Solutions Ex 5(b) 15
∴ O, A, B are collinear.
Inter 1st Year Maths 1B Three Dimensional Coordinates Solutions Ex 5(b) 16
Ratio in which ‘O’ divides AB
= OA : OB = √22 : √22 = 1 : 1
Ratio is which A divides OB
= OA:AB = -√22 : √22 = -1 : 2
Ratio in which B divides OA
= AB : BO = -2√22 : √22 =-2 : 1
A and B divide externally.

Inter 1st Year Maths 1B Differentiation Important Questions

Inter 1st Year Maths 1B Differentiation Important Questions

Students get through Maths 1B Important Questions Inter 1st Year Maths 1B Differentiation Important Questions which are most likely to be asked in the exam.

Intermediate 1st Year Maths 1B Differentiation Important Questions

Question 1.
If f(x) = x2 (x ∈ R), prove that f is differentiable on R and find its derivative.
Solution:
Given that f(x) = x2
for x, h ∈ R, f(x + h) – f(x)(x + h)2 – x2
= x2 + h2 + 2hx – x2
= 2hx + h2 = h(2x + h)
Inter 1st Year Maths 1B Differentiation Important Questions 1
∴ f is differentiable on R and f'(x) = 2x for each x ∈ R

Inter 1st Year Maths 1B Differentiation Important Questions

Question 2.
Suppose f(x) = \(\sqrt{x}\) (x > 0). Prove that f is differentiable on (0, ∞) and find f(x).
Solution:
Let x ∈ (0, ∞) h ≠ 0 and |h| < 0
Inter 1st Year Maths 1B Differentiation Important Questions 2

Question 3.
If f(x) = \(\frac{1}{x^{2}+1}\) (x ∈ R), prove that f is differentiable on R and find f'(x).
Solution:
Let x ∈ R and h ≠ 0
Inter 1st Year Maths 1B Differentiation Important Questions 3
= –\(\frac{2 x}{\left(x^{2}+1\right)^{2}}\)
∴ f is differentiable and f'(x) = –\(\frac{2 x}{\left(x^{2}+1\right)^{2}}\) for each x ∈ R .

Question 4.
If f(x) = sin x (x ∈ R), then show that f is differentiable on R and f'(x) = cosx.
Solution:
Let x ∈ R and h ≠ 0
Inter 1st Year Maths 1B Differentiation Important Questions 4
∴ f is differentiable on R and f'(x) = cos x for each x ∈ R.

Inter 1st Year Maths 1B Differentiation Important Questions

Question 5.
Show that f(x) = |x| (x ∈ R) is not differentiable at zero and is differentiable at any x ≠ 0.
Solution:
Given f(x) = |x|
∴ f(x) = x if x ≥ 0
if h ≠ 0
\(\frac{f(0+h)-f(0)}{h}\) = \(\frac{f(h)}{h}\) = \(\left\{\begin{array}{r}
1 \text { if } h>0 \\
-1 \text { if } h<0
\end{array}\right.\)
f'(0+) = 1, f'(0) = -1
∴ is not differentiable at zero it can be easily proved that f is differentiable at any x ≠ 0 and that f'(x) = \(\left\{\begin{array}{l}
1 \quad \text { if } x>0 \\
-1 \text { if } x<0
\end{array}\right.\)

Question 6.
Check whether the following function is differentiable at zero f(x) = \(\left\{\begin{array}{l}
3+x \text { if } x \geq 0 \\
3-x \text { if } x<0
\end{array}\right.\)
Solution:
Inter 1st Year Maths 1B Differentiation Important Questions 5
f has the left hand derivative at zero and f'(0) = -1
∴ f'(0+) ≠ f'(0)
f(x) is not differentiable at zero.

Inter 1st Year Maths 1B Differentiation Important Questions

Question 7.
Show that the derivative of a constant function on an interval is zero.
Solution:
let f be a constant function on an interval I.
f(x) = C ∀ x ∈ I for some constant.
Let a ∈ I, for h ≠ 0 \(\frac{f(a+h)-f(a)}{h}\) = \(\frac{c-c}{h}\) = 0
for sufficiently small (h)
Inter 1st Year Maths 1B Differentiation Important Questions 6
∴ f is differentiable 0 and f'(0).

Question 8.
Suppose for all x, y ∈ R f(x + y) = f(x). f(y) and f'(0) exists. Then show that f(x) exists and equals to f(x) f'(0)for all x ∈ R.
Solution:
Let x ∈ R, for h ≠ 0, we have
\(\frac{f(x+h)-f(x)}{h}\) = \(\frac{f(x) f(h)-f(x)}{h}\)
= f(x) \(\frac{[f(h)-1]}{h}\) ………………….. (1)
f(0) = f(0 + 0) = f(0) f(0) ⇒ f(0) (1 —f(0)) = 0
∴ f(0) = 0 or f(0) = 1
Case (1) : Suppose f(0) = 0
f(x) = f(x + 0) = f(x) f(0) = 0 ∀ x ∈ R
∴ f(x) is a constant function = f'(x) = 0 for all x ∈ R
∴ f'(x) = 0 = f(x) . f'(0)

Case (ii): Suppose f(0) = 1
Inter 1st Year Maths 1B Differentiation Important Questions 7
∴ f is differentiable and f'(x) = f'(x) f'(0).

Question 9.
If f(x) = (ax + b)n, (x > –\(\frac{b}{a}\)), then find f'(x).
Sol:
Let u = ax + b so that y = un
f'(x) = \(\frac{\mathrm{d}}{\mathrm{dx}}\) (un) \(\frac{\mathrm{du}}{\mathrm{dx}}\)
= n.un-1a.
= an(ax+b)n – 1

Question 10.
Find the derivative of f(x) = ex (x2 + 1)
Solution:
Let u = ex, V = x2 + 1
\(\frac{\mathrm{du}}{\mathrm{dx}}\) = ex, \(\frac{\mathrm{dv}}{\mathrm{dx}}\) = 2x
f'(x) = u(x) v'(x) + u'(x) . v(x)
= ex2x + (x2 + 1) ex
= ex(2x + x2 + 1)
= ex(x + 1)2

Inter 1st Year Maths 1B Differentiation Important Questions

Question 11.
If y = \(\frac{a-x}{a+x}\) (x ≠ -a), find \(\frac{\mathrm{d} y}{\mathrm{dx}}\).
Solution:
Let u = a – x and v = a + x so that y = \(\frac{\mathrm{u}}{\mathrm{v}}\)
Inter 1st Year Maths 1B Differentiation Important Questions 8

Question 12.
If f(x) = e2x . log x (x > 0), then find f'(x).
Solution:
Let u = e2x, v = log x so that
\(\frac{\mathrm{du}}{\mathrm{dx}}\) = 2 . e2x , \(\frac{\mathrm{dv}}{\mathrm{dx}}\) = 1
f(x) = u.v
f'(x) = u . \(\frac{\mathrm{dv}}{\mathrm{dx}}\) + y . \(\frac{\mathrm{du}}{\mathrm{dx}}\)
= e2x . \(\frac{1}{x}\) + log x (2e2x)
= e2x (\(\frac{1}{x}\) + 2 logx)

Question 13.
If f(x) = \(\sqrt{\frac{1+x^{2}}{1-x^{2}}}\) (|x| < 1), then find f'(x)
Solution:
Inter 1st Year Maths 1B Differentiation Important Questions 9

Question 14.
If f(x) = x2, 2x log x (x > 0), find f'(x).
Solution:
u = x2, v = 2x, w = logx
\(\frac{\mathrm{du}}{\mathrm{dx}}\) = 2x, \(\frac{\mathrm{dv}}{\mathrm{dx}}\) = 2x . log2, \(\frac{\mathrm{dw}}{\mathrm{dx}}\) = \(\frac{1}{x}\)
f'(x) = uv . \(\frac{\mathrm{dw}}{\mathrm{dx}}\) + vw . \(\frac{\mathrm{du}}{\mathrm{dx}}\) + uw . \(\frac{\mathrm{dv}}{\mathrm{dx}}\)
= x22x . \(\frac{1}{x}\) + 2x . logx(2x) + x2 . logx . 2x log2
= x . 2x (logx2 + xlogx (log 2) + 1)

Inter 1st Year Maths 1B Differentiation Important Questions

Question 15.
If y = \(\left|\begin{array}{l}
f(x) g(x) \\
\phi(x) \psi(x)
\end{array}\right|\) then show that \(\frac{\mathrm{dy}}{\mathrm{dx}}\) = \(\left|\begin{array}{l}
f^{\prime}(x) g^{\prime}(x) \\
\phi(x) \psi(x)
\end{array}\right|\) + \(\left|\begin{array}{l}
f(x) g \backslash(x) \\
\phi^{\prime}(x) \Psi(x)
\end{array}\right|\)
Solution:
Given y = \(\left|\begin{array}{l}
f(x) g(x) \\
\phi(x) \psi(x)
\end{array}\right|\)
= f(x) ψ(x) – Φ(x) g(x)
\(\frac{\mathrm{dy}}{\mathrm{dx}}\) = f(x) ψ'(x) + ψ(x) f'(x) – [Φ(x). g'(x) + g(x). Φ'(x)]
= [f(x) ψ'(x) – g(x) Φ’x)] + [f'(x) ψ(x) – Φ(x).g'(x)]
= \(\left|\begin{array}{l}
f(x) g(x) \\
\phi^{\prime}(x) \psi^{\prime}(x)
\end{array}\right|\) + \(\left|\begin{array}{cc}
f^{\prime}(x) & g^{\prime}(x) \\
\phi(x) & \psi(x)
\end{array}\right|\)

Question 16.
If f(x) = 7x2+3x (x > 0), then find f'(x).
Solution:
Let u = x3 + 3x ⇒ \(\frac{\mathrm{du}}{\mathrm{dx}}\) = 3x2 + 3 = 3(x2 + 1)
f(X) = 7u
f'(x) = \(\frac{d f}{d u}\) . \(\frac{\mathrm{du}}{\mathrm{dx}}\) (7u . l0g 7) [3(x2 + 1)]
= 3(x2 + 1) 7x2+3x log 7

Question 17.
If f(x) = x ex sin x, then find f(x).
Solution:
Let u = x ,v = ex, w = sin x
\(\frac{\mathrm{dy}}{\mathrm{dx}}\) = 1, \(\frac{\mathrm{dv}}{\mathrm{dx}}\) = ex . \(\frac{\mathrm{dw}}{\mathrm{dx}}\) = cos x
f(x) = u.v.w
f'(x) = uv . \(\frac{\mathrm{dw}}{\mathrm{dx}}\) + uw \(\frac{\mathrm{dv}}{\mathrm{dx}}\) + vw \(\frac{\mathrm{du}}{\mathrm{dx}}\)
= xex cos x + x . sinx ex + ex sin x.

Inter 1st Year Maths 1B Differentiation Important Questions

Question 18.
If f(x) = sin (log x), (x > 0), find f'(x).
Solution:
Let u = logx, y = f(x) so that y = sin u
\(\frac{\mathrm{dy}}{\mathrm{dx}}\) = \(\frac{d y}{d u}\) . \(\frac{\mathrm{du}}{\mathrm{dx}}\)
\(\frac{\mathrm{dy}}{\mathrm{dx}}\) = cos u, \(\frac{\mathrm{du}}{\mathrm{dx}}\) = \(\frac{1}{x}\)
f'(x) = \(\frac{1}{x}\) . cos u = \(\frac{1}{x}\) cos (log x)

Question 19.
If f(x) =(x3 + 6x2 + 12x – 13)100; find f'(x).
Solution:
u = x3 + 6x2 + 12x – 13
⇒ \(\frac{\mathrm{du}}{\mathrm{dx}}\) = 3x2 + 12x + 12
= 3(x2 + 4x + 4)
= 3(x + 2)2
f(x) = u100
f'(x) = 100 . u99 . \(\frac{\mathrm{du}}{\mathrm{dx}}\)
= 100 (x3 + 6x2 + 12x – 13)99 . 3(x + 2)2
= 300 (x + 2)2 (x3 + 6x2 + 12x – 13)99

Question 20.
Find the derivative of f(x) = \(\frac{x \cos x}{\sqrt{1+x^{2}}}\)
Solution:
Let u = x cos x, and v = \(\sqrt{1+x^{2}}\) so that
Inter 1st Year Maths 1B Differentiation Important Questions 10

Question 21.
li f(x) = log (secx + tan x), find f'(x). [Mar 14, May 11]
Solution:
Let u = sec x + tan x and y = log u
\(\frac{\mathrm{dy}}{\mathrm{dx}}\) = \(\frac{1}{u}\), \(\frac{\mathrm{du}}{\mathrm{dx}}\) = sec x. tan x + sec2 x
= sec x (sec x + tan x)
\(\frac{\mathrm{dy}}{\mathrm{dx}}\) = \(\frac{\mathrm{dy}}{\mathrm{dx}}\) . \(\frac{\mathrm{du}}{\mathrm{dx}}\)
= \(\frac{1}{\sec x+\tan x}\) . sec x(sec x + tan x) = sec x

Inter 1st Year Maths 1B Differentiation Important Questions

Question 22.
If y = sin-1\(\sqrt{x}\), find \(\frac{\mathrm{dy}}{\mathrm{dx}}\)
Solution:
u = \(\sqrt{x}\), y = sin-1 x.
Inter 1st Year Maths 1B Differentiation Important Questions 11

Question 23.
If y = sec (\(\sqrt{\tan x}\)), find \(\frac{\mathrm{dy}}{\mathrm{dx}}\).
Solution:
u = \(\sqrt{\tan x}\), v = tanx
Then y = sec u, u = \(\sqrt{\mathrm{v}}\), v = tan x
Inter 1st Year Maths 1B Differentiation Important Questions 12

Question 24.
If y = \(\frac{x \sin ^{-1} x}{\sqrt{1-x^{2}}}\), find \(\frac{\mathrm{dy}}{\mathrm{dx}}\).
Solution:
Let u = x sin-1x, v = \(\sqrt{1-x^{2}}\)
Inter 1st Year Maths 1B Differentiation Important Questions 13

Inter 1st Year Maths 1B Differentiation Important Questions

Question 25.
If y = log (cosh 2x), find \(\frac{\mathrm{dy}}{\mathrm{dx}}\).
Solution:
Let u = cosh 2x, so that y = log u
\(\frac{\mathrm{dy}}{\mathrm{dx}}\) = \(\frac{1}{u}\); \(\frac{\mathrm{du}}{\mathrm{dx}}\) = 2 sin h2x
\(\frac{\mathrm{dy}}{\mathrm{dx}}\) = \(\frac{d y}{d u}\) . \(\frac{\mathrm{du}}{\mathrm{dx}}\)
= 2 sin h 2x . \(\cosh 2 x\) = 2 tan h 2x

Question 26.
If y = log (sin (log x)), find \(\frac{\mathrm{dy}}{\mathrm{dx}}\).
Solution:
Let v = log x, u = sin v so that y = log u.
\(\frac{\mathrm{dy}}{\mathrm{dx}}\) = \(\frac{1}{u}\); \(\frac{d u}{d v}\) = cos u; \(\frac{\mathrm{dv}}{\mathrm{dx}}\) = \(\frac{1}{x}\)
\(\frac{\mathrm{dy}}{\mathrm{dx}}\) = \(\frac{d y}{d u}\) . \(\frac{d u}{d v}\) . \(\frac{\mathrm{dv}}{\mathrm{dx}}\)
= \(\frac{1}{\sin (\log x)}\) . cos (logx) \(\frac{1}{x}\) = \(\frac{\cot (\log x)}{x}\)

Question 27.
If y = (cot-1x3)2, find \(\frac{\mathrm{dy}}{\mathrm{dx}}\).
Solution:
u = cot-1x3, u = x3, y = u2
Inter 1st Year Maths 1B Differentiation Important Questions 14

Inter 1st Year Maths 1B Differentiation Important Questions

Question 28.
If y = cosec-1(e2x+1), find \(\frac{\mathrm{dy}}{\mathrm{dx}}\).
Solution:
u = e2x+1, y = cosec-1u
Inter 1st Year Maths 1B Differentiation Important Questions 15

Question 29.
If y = tan-1 (cos \(\sqrt{x}\)), find \(\frac{\mathrm{dy}}{\mathrm{dx}}\).
Solution:
v = \(\sqrt{x}\) and u = cos v, y = tan-1u
\(\frac{\mathrm{dv}}{\mathrm{dx}}\) = \(\frac{1}{2 \sqrt{x}}\), \(\frac{d u}{d v}\) = – sin u; \(\frac{\mathrm{dy}}{\mathrm{dx}}\) = \(\frac{1}{1+u^{2}}\)
= – sin \(\sqrt{x}\) = \(\frac{1}{1+\cos ^{2}(\sqrt{x})}\)
Inter 1st Year Maths 1B Differentiation Important Questions 16

Question 30.
If y = Tan-1 \(\sqrt{42}\) for 0 < |x| < 1, find \(\frac{\mathrm{dy}}{\mathrm{dx}}\). [May, Mar 12]
Solution:
Put x2 = cos 2θ
Inter 1st Year Maths 1B Differentiation Important Questions 17

Inter 1st Year Maths 1B Differentiation Important Questions

Question 31.
If y = x2exsin x, find \(\frac{\mathrm{dy}}{\mathrm{dx}}\).
Solution:
log y = log x2. ex. sin x
= log x2 + log ex + log sin x
= 2 log x + log ex + log sin x
Differentiating w.r.to by sin x
\(\frac{1}{y}\) . \(\frac{\mathrm{dy}}{\mathrm{dx}}\) = \(\frac{2}{x}\) + 1 + \(\frac{1}{sin x}\) . cos x
\(\frac{\mathrm{dy}}{\mathrm{dx}}\) = y(\(\frac{2}{x}\) + 1 + cot x)

Question 32.
If y = xtanx + (sin x)cos x, find \(\frac{\mathrm{dy}}{\mathrm{dx}}\) [Mar. 14, 11]
Solution:
Let u = xtanx and v = (sin x)cos x
log u logx tanx = (tan x) log x
Inter 1st Year Maths 1B Differentiation Important Questions 18
Inter 1st Year Maths 1B Differentiation Important Questions 19

Question 33.
If x = a(cos t + log tan (\(\frac{t}{2}\))), y = a sin t, find \(\frac{\mathrm{dy}}{\mathrm{dx}}\).
Solution:
Inter 1st Year Maths 1B Differentiation Important Questions 20
Inter 1st Year Maths 1B Differentiation Important Questions 21

Inter 1st Year Maths 1B Differentiation Important Questions

Question 34.
If xy = ex-y, than show that \(\frac{\mathrm{dy}}{\mathrm{dx}}\) = \(\frac{\log x}{(1+\log x)^{2}}\)    [May 07]
Solution:
xy = ex-y
log xy = log ex-y
y log x = (x – y) (log e = 1)
y(1 + log x) = x
Inter 1st Year Maths 1B Differentiation Important Questions 22

Question 35.
If siny = x sin (a + y), then show that \(\frac{\mathrm{dy}}{\mathrm{dx}}\) = \(\frac{\sin ^{2}(a+y)}{\sin a}\) (a is not a multiple of π)
Solution:
Inter 1st Year Maths 1B Differentiation Important Questions 23

Question 36.
If y = x4 + tan x, then find y”.
Solution:
y = x4 + tan x
\(\frac{\mathrm{dy}}{\mathrm{dx}}\) = 4x3 + sec2 x
\(\frac{d^{2} y}{d x^{2}}\) = 12x2 + 2 sec x (sec x tan x)
= 12x2 + 2 sec2x . tan x

Inter 1st Year Maths 1B Differentiation Important Questions

Question 37.
If f(x) = sinx, sin 2x sin 3x, find f”(x).
Solution:
f(x) = \(\frac{1}{2}\) sin 2x(2 sin 3x sin x)
= \(\frac{1}{2}\) (sin 2x) (cos 2x – cos4x)
= \(\frac{1}{4}\) (2 sin 2x cos 2x – 2 sin 2x cos 4x)
= \(\frac{1}{4}\) (sin2x + sin4x – sin6x)
Therefore,
f'(x)= \(\frac{1}{4}\)[2 cos 2x+ 4cos 4x – 6cos 6x]
Hence,
f”(x) = \(\frac{1}{4}\) (-4 sin 2x – 16 sin 4x + 36 sin 6x)
= 9 sin 6x – 4 sin 4x – sin 2x.

Question 38.
Show that y = x + tan x satisfies cos2x \(\frac{d^{2} y}{d x^{2}}\) + 2x = 2y.
Solution:
y = x + tan x implies that y’ = 1 + sec2 x
That is, y’ cos2x = 1 + cos2x.
Differentiating both sides of the above equation we get
y” cos2x + y’ . 2 cos x (-sin x) = 2 cos x (- sin x)
∴ y” cos2 x = 2(y’ – 1) sin x cos x
= 2 sec2x sin x cos x = 2 tan x = 2(y – x)
This proves the result.

Question 39.
If x = a(t – sin t),y = a(1 + cost), find \(\frac{d^{2} y}{d x^{2}}\).
Solution:
Inter 1st Year Maths 1B Differentiation Important Questions 24
Inter 1st Year Maths 1B Differentiation Important Questions 25

Inter 1st Year Maths 1B Differentiation Important Questions

Question 40.
Find the second order derivative of y = tan-1(\(\frac{2 x}{1-x^{2}}\))
Solution:
Put x = tan θ, Then
y = tan-1 (\(\frac{2 \tan \theta}{1-\tan ^{2} \theta}\))
= tan-1 (tan 2θ)
= 2θ = 2 tan-1x
∴ \(\frac{\mathrm{dy}}{\mathrm{dx}}\) = \(\frac{2}{1+x^{2}}\) and \(\frac{d^{2} y}{d x^{2}}\) = \(\frac{-4 x}{\left(1+x^{2}\right)^{2}}\).

Question 41.
If y = sin (sin x), show that y” + (tan x) y’ + y cos2x = 0.
Solution:
y = sin (sin x) implies that .
y’ = cos x . cos (sin x) and
y” = -cos2x sin (sin x) – sin x cos (sin x)
= – y cos2x – sin x (\(\frac{y^{\prime}}{\cos x}\))
= -y cos2x – y’ tan x
∴ y” + (tan x)y’ + y cos2 x = 0.

Question 42.
If f(x) = ex(x ∈ R), then show that f(x) = ex by first principle.
Solution:
From f(x) = ex we have for h ≠ 0
Inter 1st Year Maths 1B Differentiation Important Questions 26

Inter 1st Year Maths 1B Differentiation Important Questions

Question 43.
If f(x) = log x (x > 0), then show that f(x) = \(\frac{1}{x}\) by first principle.
Solution:
Now for h ≠ 0
Inter 1st Year Maths 1B Differentiation Important Questions 27
\(\frac{d}{dx}\) (log x) = \(\frac{1}{x}\)

Question 44.
If 1(x) = ax (x ∈ R) (a > 0), then show that f'(x) = ax log a by first principle.
Solution:
For h ≠ 0
\(\frac{f(x+h)-f(x)}{h}\) = \(\frac{a^{x+h}-a^{x}}{h}\) = ax [latex]\frac{a^{h}-1}{h}[/latex]
We know that \(\frac{a^{h}-1}{h}\) → log a as h → 0
Hence f'(x) = ax . log a.
\(\frac{d}{d x}\) = (ax) = ax log a

Question 45.
If y = Tan-1 \(\sqrt{\frac{1-x}{1+x}}\) (|x| < 1), we shall find \(\frac{\mathrm{dy}}{\mathrm{dx}}\).
Solution:
Substituting x cos u (u ∈ (0, π)) in y, we get
Inter 1st Year Maths 1B Differentiation Important Questions 28
Inter 1st Year Maths 1B Differentiation Important Questions 29
observe that Tan-1x, \(\sqrt{\frac{1-x}{1+x}}\) and cos u are the functions that stand for f(x), g(x) and h(u) respectively, mentioned in the method.

Inter 1st Year Maths 1B Differentiation Important Questions

Question 46.
If y = Tan-1[latex]\frac{2 x}{1-x^{2}}[/latex] (|x| < 1) then we shall \(\frac{\mathrm{dy}}{\mathrm{dx}}\).
Solution:
Substituting x = tan u
Inter 1st Year Maths 1B Differentiation Important Questions 30

Question 47.
If x = a cos3t, y = a sin3t, find \(\frac{\mathrm{dy}}{\mathrm{dx}}\).
Solution:
Here \(\frac{\mathrm{dy}}{\mathrm{dx}}\) = 3a cos2t (-sin t) and
\(\frac{d y}{d t}\) = 3a sin2t. cost.
∴ \(\frac{\mathrm{dy}}{\mathrm{dx}}\) = \(\frac{\frac{\mathrm{dy}}{\mathrm{dt}}}{\frac{\mathrm{dx}}{\mathrm{dt}}}\) = -tan t

Question 48.
If y = et +cost, x = log t + sin t find \(\frac{\mathrm{dy}}{\mathrm{dx}}\)
Solution:
Here \(\frac{\mathrm{dy}}{\mathrm{dx}}\) = et – sin t and \(\frac{d x}{d t}\) = \(\frac{1}{t}\) + cos t
∴ \(\frac{\mathrm{dy}}{\mathrm{dx}}\) = \(\frac{t\left(e^{t}-\sin t\right)}{(1+t \cos t)}\)

Inter 1st Year Maths 1B Differentiation Important Questions

Question 49.
To find the derivative of f(x) = x\(\sin ^{\frac{1}{x}}\) with respect to g(x) = sin-1x, we have to compute \(\frac{d f}{d g}\)
Solution:
Now f(x) = x\(\sin ^{\frac{1}{x}}\) implies that
log f(x) = sin-1x . log x so that
Inter 1st Year Maths 1B Differentiation Important Questions 31

Question 50.
If x3 + y3 – 3axy = 0, find \(\frac{\mathrm{dy}}{\mathrm{dx}}\).
Solution:
Let the given equation define the function.
y = 1(x) that is x3 + (f(x))3 – 3axf(x) = 0
Differentiating both sides of this equation with respect to x, we get
3x2 + 3 (f(x))2 f'(x) – [3a. f(x) + 3axf'(x)] = 0
Hence 3x2 + 3y2 f'(x) – [3ay + 3ax f'(x)] = 0
∴ f'(x) = \(\frac{\mathrm{dy}}{\mathrm{dx}}\) = \(\frac{a y-x^{2}}{y^{2}-a x}\)

Question 51.
If 2x2 – 3xy + y2 + x + 2y – 8 = 0, find \(\frac{\mathrm{dy}}{\mathrm{dx}}\).
Solution:
Treating y as a function of x and then differentiating with respect to x,
we get 4x – 3y – 3xy’ + 2yy’ + 1 + 2y’ = 0
∴ \(\frac{\mathrm{dy}}{\mathrm{dx}}\) = y’ = \(\frac{3 y-4 x-1}{2 y-3 x+2}\)

Inter 1st Year Maths 1B Differentiation Important Questions

Question 52.
If y = xx (x > 0), we shall find \(\frac{\mathrm{dy}}{\mathrm{dx}}\).
Solution:
Taking logarithms on both the sides of
y = xx we obtain log y = x log x
Differentiating with respect to x,
We get \(\frac{y^{\prime}}{y}\) = x . \(\frac{1}{x}\) + log x = 1 + log x
∴ \(\frac{\mathrm{dy}}{\mathrm{dx}}\) = y’ = y(1 + log x) = xx (1 + log x)

Question 53.
If y = (tan x)sin x [o < x < \(\frac{\pi}{2}\) ] compute \(\frac{\mathrm{dy}}{\mathrm{dx}}\).
Solution:
Taking logarthms on both sides of
y= (tan x)sin x, we get
log y = sin x . log (tan x)
Differentiating with respect to x, we get
\(\frac{y^{\prime}}{y}\) = \(\frac{\sin x}{\tan x}\) . sec2x + cosx . log (tan x)
= sec x + cos x . log (tan x)
Hence \(\frac{\mathrm{dy}}{\mathrm{dx}}\) = (tan x)sin x [sec x + cos x log (tan x)]

Inter 1st Year Maths 1A Products of Vectors Important Questions

Products of Vectors Important Questions

Students get through Maths 1A Important Questions Inter 1st Year Maths 1A Products of Vectors Important Questions which are most likely to be asked in the exam.

Intermediate 1st Year Maths 1A Products of Vectors Important Questions

Question 1.
If \(\bar{a}\) = 6\(\bar{i}\) + 2\(\bar{j}\) + 3\(\bar{k}\) and \(\bar{b}\) = 2\(\bar{i}\) – 9\(\bar{j}\) + 6\(\bar{k}\), then find \(\bar{a}\) . \(\bar{b}\) and the angle between \(\bar{a}\) and \(\bar{b}\).
Solution:
Inter 1st Year Maths 1A Products of Vectors Important Questions 1

Question 2.
If a = i + 2j – 3k, b = 3i – j + 2k then show that a + b and a – b are perpendicular to each other. (A.P) (Mar. ’15, May ’11)
Solution:
a + b = (1 + 3)i + (2 – 1)j+ (-3 + 2) k
= 4i + j – k
Similarly a – b = – 2i + 3j – 5k
(a + b). (a – b) = 4(-2) + 1(3) + (-1)(-5) = 0
Hence (a + b) and (a – b) are mutually perpendicular.

Inter 1st Year Maths 1A Products of Vectors Important Questions

Question 3.
Let \(\bar{a}\) and \(\bar{b}\) be non- zero, non-collinear vectors. If |\(\bar{a}\) + \(\bar{b}\) | = | \(\bar{a}\) – \(\bar{b}\) |, then find the angle between \(\bar{a}\) and \(\bar{b}\).
Solution:
| \(\bar{a}\) + \(\bar{b}\) |2 = | a – b |2.
⇒ |\(\bar{a}\)|2 + |\(\bar{b}\)|2 + 2 (\(\bar{a}\). \(\bar{b}\))
⇒ |\(\bar{a}\)|2 + |\(\bar{b}\)|2 – 2 (\(\bar{a}\). \(\bar{b}\))
⇒ 4(\(\bar{a}\) . \(\bar{b}\)) = 0 ⇒ \(\bar{a}\) . \(\bar{b}\) = 0
⇒ Angle between \(\bar{a}\) and \(\bar{b}\) is 90°.

Question 4.
If |\(\bar{a}\)| = 11, |\(\bar{b}\)| = 23 and | \(\bar{a}\) – \(\bar{b}\) | = 30, then find the angle between the vectors \(\bar{a}\), \(\bar{b}\) and also find | \(\bar{a}\) + \(\bar{b}\) |.
Solution:
Given that | \(\bar{a}\) | = 11, | \(\bar{b}\) | = 23 and | \(\bar{a}\) – \(\bar{b}\) | = 30, Let (\(\bar{a}\), \(\bar{b}\)) = θ.
Inter 1st Year Maths 1A Products of Vectors Important Questions 2

Question 5.
If \(\bar{a}\) = \(\bar{i}\) – \(\bar{j}\) – \(\bar{k}\) and \(\bar{b}\) = 2\(\bar{i}\) – 3\(\bar{j}\) + \(\bar{k}\), then find the projection vector of \(\bar{b}\) on \(\bar{a}\) and its magnitude.
Solution:
Inter 1st Year Maths 1A Products of Vectors Important Questions 3

Question 6.
If P, Q, R and S are points whose position vectors are \(\bar{i}\) – \(\bar{k}\), –\(\bar{i}\) +2\(\bar{j}\), 2\(\bar{i}\) – 3\(\bar{k}\) and 3\(\bar{i}\) – 2\(\bar{j}\) – \(\bar{k}\) respectively, then find the component of \(\overline{\mathbf{R S}}\) on \(\overline{\mathbf{P Q}}\).
Solution:
Let ‘O’ be the origin.
\(\overline{\mathrm{OP}}\) = \(\overline{\mathbf{i}}\) – \(\overline{\mathbf{k}}\), \(\overline{\mathrm{OQ}}\) = –\(\overline{\mathbf{i}}\) + 2\(\overline{\mathbf{j}}\), \(\overline{\mathrm{OR}}\) = 2\(\overline{\mathbf{i}}\) – 3\(\overline{\mathbf{k}}\) and \(\overline{\mathrm{OR}}\) = 3\(\overline{\mathbf{i}}\) – 2\(\overline{\mathbf{j}}\) – \(\overline{\mathbf{k}}\)
Inter 1st Year Maths 1A Products of Vectors Important Questions 4
The component of \(\overline{\mathrm{RS}}\) and \(\overline{\mathrm{PQ}}\)
Inter 1st Year Maths 1A Products of Vectors Important Questions 5

Question 7.
If the vectors λ\(\overline{\mathbf{i}}\) – 3\(\overline{\mathbf{j}}\) + 5\(\overline{\mathbf{k}}\) and 2 λ\(\overline{\mathbf{i}}\) – λ\(\overline{\mathbf{j}}\) + \(\overline{\mathbf{k}}\) are perpendicular to each other, find λ. (T.S) (Mar. ’16)
Solution:
Since the vectors are perpendicular
⇒ (λ\(\overline{\mathbf{i}}\) – 3\(\overline{\mathbf{j}}\) + 5\(\overline{\mathbf{k}}\)) . (2λ\(\overline{\mathbf{i}}\) – λ\(\overline{\mathbf{j}}\) – \(\overline{\mathbf{k}}\)) = 0
⇒ λ(2λ) + (-3)(-λ) + 5(-1) = 0
⇒ 2λ2 + 3λ – 5 = 0
⇒ (2λ + 5)(λ – 1) = 0
∴ λ = 1 (or) \(\frac{-5}{2}\)

Inter 1st Year Maths 1A Products of Vectors Important Questions

Question 8.
Let \(\overline{\mathbf{a}}\) = 2\(\overline{\mathbf{i}}\) + 3\(\overline{\mathbf{j}}\) + \(\overline{\mathbf{k}}\), \(\overline{\mathbf{b}}\) = 4\(\overline{\mathbf{i}}\) + \(\overline{\mathbf{j}}\) and \(\overline{\mathbf{c}}\) = \(\overline{\mathbf{i}}\) – 3\(\overline{\mathbf{j}}\) – 7\(\overline{\mathbf{k}}\). Find the vector \(\overline{\mathbf{r}}\) such that \(\overline{\mathbf{r}}\) . \(\overline{\mathbf{a}}\) = 9, \(\overline{\mathbf{r}}\) . \(\overline{\mathbf{b}}\) = 7 and \(\overline{\mathbf{r}}\) . \(\overline{\mathbf{c}}\) = 6.
Solution:
Let \(\overline{\mathbf{r}}\) = x\(\overline{\mathbf{i}}\) + y\(\overline{\mathbf{j}}\) + z\(\overline{\mathbf{k}}\)
∴ \(\overline{\mathbf{r}}\) . \(\overline{\mathbf{a}}\) = 9 ⇒ 2x + 3y + z = 9
\(\overline{\mathbf{r}}\) . \(\overline{\mathbf{b}}\) = 7 ⇒ 4x + y = 7 and \(\overline{\mathbf{r}}\) . \(\overline{\mathbf{c}}\) = 6 ⇒ x – 3y – 7z = 6
By solving these equations, we get x = 1, y = 3 and z = -2
∴ \(\overline{\mathbf{r}}\) = \(\overline{\mathbf{i}}\) + 3\(\overline{\mathbf{j}}\) – 2\(\overline{\mathbf{k}}\)

Question 9.
Show that the points 2\(\overline{\mathbf{i}}\) – \(\overline{\mathbf{j}}\) + \(\overline{\mathbf{k}}\), \(\overline{\mathbf{i}}\) – 3\(\overline{\mathbf{j}}\) – 5\(\overline{\mathbf{k}}\) and 3\(\overline{\mathbf{i}}\) – 4\(\overline{\mathbf{j}}\) – 4\(\overline{\mathbf{k}}\) are the vertices of a right angle triangle. Also, find the other angles.
Solution:
Let ‘O’ be the origin and A, B, C be the given points, then
Inter 1st Year Maths 1A Products of Vectors Important Questions 6

Question 10.
Prove that the angle θ between any two diagonals of a cube is given by cos θ = \(\frac{1}{3}\). (May ’11)
Solution:
Let us suppose that the cube is a unit cube
Inter 1st Year Maths 1A Products of Vectors Important Questions 7
Inter 1st Year Maths 1A Products of Vectors Important Questions 8

Question 11.
Let \(\overline{\mathbf{a}}\), \(\overline{\mathbf{b}}\), \(\overline{\mathbf{c}}\) be non—zero mutually orthogonal vectors, if x \(\overline{\mathbf{a}}\) + y\(\overline{\mathbf{b}}\) + z = 0, then x = y = z = 0.
Solution:
x\(\overline{\mathbf{a}}\) + y\(\overline{\mathbf{b}}\) + z\(\overline{\mathbf{c}}\) = 0
⇒ \(\overline{\mathbf{a}}\) • (x\(\overline{\mathbf{a}}\) + y\(\overline{\mathbf{b}}\) + z\(\overline{\mathbf{c}}\)) = 0
⇒ x(\(\overline{\mathbf{a}}\) • \(\overline{\mathbf{a}}\)) = 0
⇒ x = 0 (∵ \(\overline{\mathbf{a}}\) • \(\overline{\mathbf{a}}\) ≠ 0)
Similarly y = 0, z = 0.

Question 12.
Let \(\overline{\mathbf{a}}\), \(\overline{\mathbf{b}}\) and \(\overline{\mathbf{c}}\) be mutually orthogonal vectors of equal magnitudes. Prove that the vector \(\overline{\mathbf{a}}\) + \(\overline{\mathbf{b}}\) + \(\overline{\mathbf{c}}\) is equally inclined to each of \(\overline{\mathbf{a}}\), \(\overline{\mathbf{b}}\) and \(\overline{\mathbf{c}}\), the angle of inclination being cos-1\(\left(\frac{1}{\sqrt{3}}\right)\).
Solution:
Inter 1st Year Maths 1A Products of Vectors Important Questions 9
Similarly, it can be proved that \(\overline{\mathbf{a}}\) + \(\overline{\mathbf{b}}\) + \(\overline{\mathbf{c}}\) inclines at an angle of cos-1\(\left(\frac{1}{\sqrt{3}}\right)\) with \(\overline{\mathbf{b}}\) and \(\overline{\mathbf{c}}\).

Question 13.
The vectors \(\overline{\mathbf{A B}}\) = 3\(\bar{i}\) – 2\(\bar{j}\) + 2\(\bar{k}\) and \(\overline{\mathbf{A D}}\) = \(\bar{i}\) – 2\(\bar{k}\) represent the adjacent sides of a parallelogram ABCD. Find the angle between the diagonals.
Solution:
Inter 1st Year Maths 1A Products of Vectors Important Questions 10
Inter 1st Year Maths 1A Products of Vectors Important Questions 11

Question 14.
For any two vectors a and b, show that
i) |a • b| (Cauchy – Schwartz inequality)
ii) || a + b | ≤ ||a|| + ||b| (triangle inequality
Solution:
i)
If a = 0 or b = 0, the inequalities hold trivially.
Inter 1st Year Maths 1A Products of Vectors Important Questions 12
ii)
Inter 1st Year Maths 1A Products of Vectors Important Questions 13

Question 15.
Find the cartesian equation of the plane passing through the point (-2, 1, 3) and perpendicular to the vector 3\(\bar{i}\) + \(\bar{j}\) + 5\(\bar{k}\).
Solution:
Let A = -2\(\bar{i}\) + \(\bar{j}\) + 3\(\bar{k}\) and \(\bar{r}\) = x\(\bar{i}\) + y\(\bar{j}\) + z\(\bar{k}\) be any point in the plane
∴ \(\overline{\mathrm{AP}}\) = (x + 2)\(\bar{i}\) + (y – 1)\(\bar{j}\) + (z – 3)\(\bar{k}\)
∴ \(\overline{\mathrm{AP}}\) is perpendicular to 3\(\bar{i}\) + \(\bar{j}\) + 5\(\bar{k}\)
⇒ (x + 2)3 + (y – 1)1 + (z – 3)5 = 0
⇒ 3x + y + z – 10 = 0

Inter 1st Year Maths 1A Products of Vectors Important Questions

Question 16.
Find the cartesian equation of the plane through the point A (2, -1, -4) and parallel to the plane 4x – 12y – 3z – 7 = 0.
Solution:
The equation of the plane parallel to
4x – 12y – 3z – 7 = 0 be
4x -12y – 3z = p
∴ If passes through the point A(2, -1, -4)
⇒ 4(2) – 12(-1) – 3(-4) = p
⇒ p = 32
∴ The equation of the required plane is
4x – 12y – 3z = 32

Question 17.
Find the angle between the planes 2x – 3y – 6z = 5 and 6x + 2y – 9z = 4.
Solution:
Equation to the planes is 2x – 3y – 6z = 5
⇒ \(\overline{\mathbf{r}}\) .(2\(\overline{\mathbf{i}}\) – 3\(\overline{\mathbf{j}}\) – 6\(\overline{\mathbf{k}}\)) = 5 ———- (1)
and 6x + 2y – 9z = 4 —— (2)
∴ The angle between the planes \(\overline{\mathbf{r}} \cdot \overline{\mathbf{n}}_{1}\) = p, and \(\overline{\mathbf{r}} \cdot \overline{\mathbf{n}}_{2}\) = q is θ, then
Inter 1st Year Maths 1A Products of Vectors Important Questions 14

Question 18.
Find unit vector orthogonal to the vector 3\(\bar{i}\) + 2\(\bar{j}\) +6\(\bar{k}\) and coplanar with thevectors 2\(\bar{i}\) + \(\bar{j}\) + \(\bar{k}\) and \(\bar{i}\) – \(\bar{j}\) + \(\bar{k}\).
Solution:
Inter 1st Year Maths 1A Products of Vectors Important Questions 15
⇒ (2x + y)3 + (x – y)2 + (x + y)6 = 0
⇒ 6x + 3y + 2x – 2y + 6x + 6y = 0
⇒ 14x + 7y = 0
⇒ y = -2x —— (1)
∵ |\(\overline{\mathbf{r}}\)| = 1
⇒ (2x + y)2 + (x – y)2 + (x + y)2 = 1
⇒ 4x2 + 4xy + 9 + 2(x2 + y2) = 1
⇒ 6x2 + 4x (-2x) + 3(-2x)2 = 1 (By using (1))
⇒ 10x2 = 1

Question 19.
If \(\overline{\mathbf{a}}\) = 2\(\overline{\mathbf{i}}\) – 3\(\overline{\mathbf{j}}\) + 5\(\overline{\mathbf{k}}\), \(\overline{\mathbf{b}}\) = –\(\overline{\mathbf{i}}\) + 4\(\overline{\mathbf{j}}\) + 2\(\overline{\mathbf{k}}\), then find \(\overline{\mathbf{a}}\) × \(\overline{\mathbf{b}}\) and unit vector perpendicular to both \(\overline{\mathbf{a}}\) and \(\overline{\mathbf{b}}\).
Solution:
\(\overline{\mathbf{a}}\) = 2\(\overline{\mathbf{i}}\) – 3\(\overline{\mathbf{j}}\) + 5\(\overline{\mathbf{k}}\) and \(\overline{\mathbf{b}}\) = –\(\overline{\mathbf{i}}\) + 4\(\overline{\mathbf{j}}\) + 2\(\overline{\mathbf{k}}\)
Inter 1st Year Maths 1A Products of Vectors Important Questions 16

Question 20.
If \(\bar{a}\) = 2\(\bar{i}\) – 3\(\bar{j}\) + 5\(\bar{k}\), \(\bar{b}\) = –\(\bar{i}\) + 4\(\bar{j}\) + 2\(\bar{k}\), then find (\(\bar{a}\) + \(\bar{b}\)) × (\(\bar{a}\) – \(\bar{b}\)) and unit vector perpendicular to both \(\bar{a}\) + \(\bar{b}\) and \(\bar{a}\) – \(\bar{b}\).
Solution:
Inter 1st Year Maths 1A Products of Vectors Important Questions 17

Inter 1st Year Maths 1A Products of Vectors Important Questions

Question 21.
Find the area of the parallelogram for which the vectors \(\bar{a}\) = 2\(\bar{i}\) – 3\(\bar{j}\) and \(\bar{b}\) = 3\(\bar{i}\) – \(\bar{k}\) are adjacent sides. (Mar. ; May ‘08)
Solution:
The vector area of the given parallelogram
Inter 1st Year Maths 1A Products of Vectors Important Questions 18

Question 22.
If \(\overline{\mathbf{a}}\), \(\overline{\mathbf{b}}\), \(\overline{\mathbf{c}}\) and \(\overline{\mathbf{d}}\) are vectors such that \(\overline{\mathbf{a}}\) × \(\overline{\mathbf{b}}\) = \(\overline{\mathbf{c}}\) × \(\overline{\mathbf{d}}\) and \(\overline{\mathbf{a}}\) × \(\overline{\mathbf{c}}\) = \(\overline{\mathbf{b}}\) × \(\overline{\mathbf{d}}\) then show that the vectors \(\overline{\mathbf{a}}\) – \(\overline{\mathbf{d}}\) and \(\overline{\mathbf{b}}\) – \(\overline{\mathbf{c}}\) are parallel.
Solution:
Inter 1st Year Maths 1A Products of Vectors Important Questions 19

Question 23.
If \(\overline{\mathbf{a}}\) = \(\overline{\mathbf{i}}\) + 2\(\overline{\mathbf{j}}\) + 3\(\overline{\mathbf{k}}\) and \(\overline{\mathbf{b}}\) = 3\(\overline{\mathbf{i}}\) + 5\(\overline{\mathbf{j}}\) – \(\overline{\mathbf{k}}\) are two sides of a triangle, then find
its area.
Solution:
Area of the triangle = \(\frac{1}{2}|\bar{a} \times \bar{b}|\)
Inter 1st Year Maths 1A Products of Vectors Important Questions 20
Inter 1st Year Maths 1A Products of Vectors Important Questions 21

Question 24.
In Δ ABC, if \(\overline{\mathrm{BC}}\) = \(\bar{a}\), \(\overline{\mathrm{CA}}\) = \(\bar{b}\), \(\overline{\mathrm{AB}}\) = \(\bar{c}\) then show that \(\bar{a}\) × \(\bar{b}\) = \(\bar{b}\) × \(\bar{c}\) = \(\bar{c}\) × \(\bar{a}\).
Solution:
Inter 1st Year Maths 1A Products of Vectors Important Questions 22

Question 25.
Let \(\bar{a}\) = 2\(\bar{i}\) – \(\bar{j}\) + \(\bar{k}\) and \(\bar{b}\) = 3\(\bar{i}\) + 4\(\bar{j}\) – \(\bar{k}\). If θ is the angle between \(\bar{a}\) and \(\bar{b}\) then find sin θ.
Solution:
Inter 1st Year Maths 1A Products of Vectors Important Questions 23

Inter 1st Year Maths 1A Products of Vectors Important Questions

Question 26.
Let \(\bar{a}\), \(\bar{b}\) and \(\bar{c}\) be such that \(\bar{c}\) ≠ 0,
\(\bar{a}\) × \(\bar{b}\) = \(\bar{c}\), \(\bar{b}\) × \(\bar{c}\) = \(\bar{a}\). Show that \(\bar{a}\), \(\bar{b}\), \(\bar{c}\) are pair wise orthogonal vectors and |\(\bar{b}\)| = 1, |\(\bar{c}\)| = |\(\bar{a}\)|.
Solution:
Inter 1st Year Maths 1A Products of Vectors Important Questions 24

Question 27.
Let \(\overline{\mathbf{a}}\) = 2\(\overline{\mathbf{i}}\) + \(\overline{\mathbf{j}}\) – 2\(\overline{\mathbf{k}}\) ; \(\overline{\mathbf{b}}\) = \(\overline{\mathbf{i}}\) + \(\overline{\mathbf{j}}\). If \(\overline{\mathbf{c}}\) is a vector such that \(\overline{\mathbf{a}}\) . \(\overline{\mathbf{c}}\) = |\(\overline{\mathbf{c}}\)|, |\(\overline{\mathbf{c}}\) – \(\overline{\mathbf{a}}\)| = \(2 \sqrt{2}\) and the angle between \(\overline{\mathbf{a}}\) × \(\overline{\mathbf{b}}\) = \(2 \sqrt{2}\) and the angle between \(\overline{\mathbf{a}}\) × \(\overline{\mathbf{b}}\) and \(\overline{\mathbf{c}}\) is 30°, then find the value of |(\(\overline{\mathbf{a}}\) × \(\overline{\mathbf{b}}\)) × \(\overline{\mathbf{c}}\)|
Solution:
Inter 1st Year Maths 1A Products of Vectors Important Questions 25
Inter 1st Year Maths 1A Products of Vectors Important Questions 26

Question 28.
Let \(\overline{\mathbf{a}}\), \(\overline{\mathbf{b}}\) be two non-collinear unit vectors. If \(\bar{\alpha}\) = \(\overline{\mathbf{a}}\) – (\(\overline{\mathbf{a}}\) • \(\overline{\mathbf{b}}\))\(\overline{\mathbf{b}}\) and \(\bar{\beta}\) =
= \(\overline{\mathbf{a}}\) × \(\overline{\mathbf{b}}\), then show that |\(\overline{\boldsymbol{\beta}}\)| = |\(\bar{\alpha}\)|.
Solution:
Inter 1st Year Maths 1A Products of Vectors Important Questions 27

Question 29.
A non-zero vector \(\bar{a}\) is parallel to the line of intersection of the plane determined by the vectors \(\bar{i}\), \(\bar{i}\) + \(\bar{j}\) and the plane determined by the vectors \(\bar{i}\) – \(\bar{j}\), \(\bar{i}\) + \(\bar{k}\). Find the angle between \(\bar{a}\) and the vector \(\bar{i}\) – 2\(\bar{j}\) + 2\(\bar{k}\).
Solution:
Let l be the line of intersection of the planes determined by the pairs \(\bar{i}\), \(\bar{i}\) + \(\bar{j}\) and \(\bar{i}\) – \(\bar{j}\), \(\bar{i}\) + \(\bar{k}\)
Inter 1st Year Maths 1A Products of Vectors Important Questions 28
∴ \(\bar{n}_{1}\) is perpendicular to l and \(\bar{n}_{2}\) is also perpendicular to l.
∴ \(\bar{a}\) is parallel to the line l, follows that \(\bar{a}\) is perpendicularto both \(\bar{n}_{1}\), and \(\bar{n}_{2}\)
Inter 1st Year Maths 1A Products of Vectors Important Questions 29

Question 30.
Let \(\overline{\mathbf{a}}\) = 4\(\overline{\mathbf{i}}\) + 5\(\overline{\mathbf{j}}\) – \(\overline{\mathbf{k}}\), \(\overline{\mathbf{b}}\) = \(\overline{\mathbf{i}}\) – 4\(\overline{\mathbf{j}}\) + 5\(\overline{\mathbf{k}}\) and \(\overline{\mathbf{c}}\) = 3\(\overline{\mathbf{i}}\) + \(\overline{\mathbf{j}}\) – \(\overline{\mathbf{k}}\). Find the vector \(\bar{\alpha}\) which is perpendicular to both \(\overline{\mathbf{a}}\) and \(\overline{\mathbf{b}}\) and \(\bar{\alpha} \cdot \mathbf{c}\) = 21.
Solution:
Inter 1st Year Maths 1A Products of Vectors Important Questions 30

Question 31.
For any vector a, show that |a × i|2 + |a × j|2 + |a × k|2 = 2|a|2.
Solution:
Let a = x i + y j + z k.
Then a × i = -yk + zj.
∴ |a × i|2 = y2 + z2
Similarly |a × j|2 = z2 + x2 and |a × k|2 = x2
+ y2
∴ |a × i|2 + |a × j|2 + |a × k|2 = 2(x2 + y2 + z2) = 2|a|2

Inter 1st Year Maths 1A Products of Vectors Important Questions

Question 32.
If \(\overline{\mathbf{a}}\) is a non-zero vector and \(\overline{\mathbf{b}}\), \(\overline{\mathbf{c}}\) are two vector such that \(\overline{\mathbf{a}}\) × \(\overline{\mathbf{b}}\) = \(\overline{\mathbf{a}}\) × \(\overline{\mathbf{c}}\) and \(\overline{\mathbf{a}}\) • \(\overline{\mathbf{b}}\) = \(\overline{\mathbf{a}}\) • \(\overline{\mathbf{c}}\), then prove that \(\overline{\mathbf{b}}\) = \(\overline{\mathbf{c}}\).
Solution:
Inter 1st Year Maths 1A Products of Vectors Important Questions 31

Question 33.
Prove that the vectors \(\bar{a}\) = 2\(\bar{i}\) – \(\bar{j}\) + \(\bar{k}\), \(\bar{b}\) = \(\bar{i}\) – 3\(\bar{j}\) – 5\(\bar{k}\) and \(\bar{c}\) = 3\(\bar{i}\) – 4\(\bar{j}\) – 4\(\bar{k}\) are coplanar.
Solution:
Inter 1st Year Maths 1A Products of Vectors Important Questions 32

Question 34.
Find the volume of the parallelopiped whose cotersminus edges are represented by the vectors 2\(\bar{i}\) – 3\(\bar{j}\) + \(\bar{k}\), \(\bar{i}\) – \(\bar{j}\) + 2\(\bar{k}\) and 2\(\bar{i}\) + \(\bar{j}\) – \(\bar{k}\).
Solution:
Let \(\bar{a}\) = 2\(\bar{i}\) – 3\(\bar{j}\) + \(\bar{k}\)
Inter 1st Year Maths 1A Products of Vectors Important Questions 33

Question 35.
If the vectors \(\bar{a}\) = 2\(\bar{i}\) – \(\bar{j}\) + \(\bar{k}\), \(\bar{b}\) = \(\bar{i}\) + 2\(\bar{j}\) – 3\(\bar{k}\) and \(\bar{c}\) = 3\(\bar{i}\) + p\(\bar{j}\) + 5\(\bar{k}\) are coplanar, then find p.
Solution:
Inter 1st Year Maths 1A Products of Vectors Important Questions 34

Question 36.
Show that
\(\overline{\mathbf{i}}\) × (\(\overline{\mathbf{a}}\) × \(\overline{\mathbf{i}}\)) + \(\overline{\mathbf{j}}\) × (\(\overline{\mathbf{a}}\) × \(\overline{\mathbf{j}}\)) + k × (\(\overline{\mathbf{a}}\) × \(\overline{\mathbf{k}}\)) = 2\(\overline{\mathbf{a}}\) for any vector \(\overline{\mathbf{a}}\).
Solution:
Inter 1st Year Maths 1A Products of Vectors Important Questions 35

Question 37.
Prove that for any three vectors \(\overline{\mathbf{a}}\), \(\overline{\mathbf{b}}\), \(\overline{\mathbf{c}}\), [\(\overline{\mathbf{b}}\) + \(\overline{\mathbf{c}}\) \(\overline{\mathbf{c}}\) + \(\overline{\mathbf{a}}\) \(\overline{\mathbf{a}}\) + \(\overline{\mathbf{b}}\)] = 2[\(\overline{\mathbf{a}}\) \(\overline{\mathbf{b}}\) \(\overline{\mathbf{c}}\)]
Solution:
Inter 1st Year Maths 1A Products of Vectors Important Questions 36

Question 38
For any three vectors \(\overline{\mathbf{a}}\), \(\overline{\mathbf{b}}\), \(\overline{\mathbf{c}}\) prove that [\(\overline{\mathbf{b}}\) × \(\overline{\mathbf{c}}\) \(\overline{\mathbf{c}}\) × \(\overline{\mathbf{a}}\) \(\overline{\mathbf{a}}\) × \(\overline{\mathbf{b}}\)] = [\(\overline{\mathbf{a}}\) \(\overline{\mathbf{b}}\) \(\overline{\mathbf{c}}\)]2
Solution:
Inter 1st Year Maths 1A Products of Vectors Important Questions 37

Question 39.
Let \(\overline{\mathbf{a}}\), \(\overline{\mathbf{b}}\) and \(\overline{\mathbf{c}}\) be unit vectors such that \(\overline{\mathbf{b}}\) is not parallel to \(\overline{\mathbf{c}}\) and \(\overline{\mathbf{a}}\) × (\(\overline{\mathbf{b}}\) × \(\overline{\mathbf{c}}\)) = \(\frac{1}{2} \bar{b}\). Find the angles made by \(\overline{\mathbf{a}}\) with each of \(\overline{\mathbf{b}}\) and \(\overline{\mathbf{c}}\).
Solution:
Inter 1st Year Maths 1A Products of Vectors Important Questions 38
Since \(\overline{\mathbf{b}}\) and \(\overline{\mathbf{c}}\) are non—collinear vectors. Equating corresponding coeffs. on both sides
Inter 1st Year Maths 1A Products of Vectors Important Questions 39
Inter 1st Year Maths 1A Products of Vectors Important Questions 40

Question 40.
Let \(\overline{\mathbf{a}}\) = \(\overline{\mathbf{i}}\) + \(\overline{\mathbf{j}}\) + \(\overline{\mathbf{k}}\), \(\overline{\mathbf{b}}\) = 2\(\overline{\mathbf{i}}\) – \(\overline{\mathbf{j}}\) + 3\(\overline{\mathbf{k}}\), \(\overline{\mathbf{c}}\) = \(\overline{\mathbf{i}}\) – \(\overline{\mathbf{j}}\) and \(\overline{\mathbf{d}}\) = 6\(\overline{\mathbf{i}}\) + 2\(\overline{\mathbf{j}}\) + 3\(\overline{\mathbf{k}}\). Express \(\overline{\mathbf{d}}\) interms of \(\overline{\mathbf{b}}\) × \(\overline{\mathbf{c}}\), \(\overline{\mathbf{c}}\) × \(\overline{\mathbf{a}}\) and \(\overline{\mathbf{a}}\) × \(\overline{\mathbf{b}}\)  (May. ’12)
Solution:
Inter 1st Year Maths 1A Products of Vectors Important Questions 41
Then from the above problem,
Inter 1st Year Maths 1A Products of Vectors Important Questions 42

Inter 1st Year Maths 1A Products of Vectors Important Questions

Question 41.
For any four vectors \(\overline{\mathbf{a}}\), \(\overline{\mathbf{b}}\), \(\overline{\mathbf{c}}\) and \(\overline{\mathbf{d}}\) prove that
(\(\overline{\mathbf{b}}\) × \(\overline{\mathbf{c}}\)) . (\(\overline{\mathbf{a}}\) × \(\overline{\mathbf{d}}\)) + (\(\overline{\mathbf{c}}\) × \(\overline{\mathbf{a}}\)) . (\(\overline{\mathbf{b}}\) × \(\overline{\mathbf{d}}\)) + (\(\overline{\mathbf{a}}\) × \(\overline{\mathbf{b}}\)) . (\(\overline{\mathbf{c}}\) × \(\overline{\mathbf{d}}\)) = 0.
Solution:
Inter 1st Year Maths 1A Products of Vectors Important Questions 43

Question 42.
Find the equation of the plane passing through the points A = (2, 3, -1), B = (4, 5, 2) and C = (3, 6, 5).
Solution:
Let ‘O’ be the origin
Inter 1st Year Maths 1A Products of Vectors Important Questions 44
Let P be any point on the plane passing through the points A, B, C.
Inter 1st Year Maths 1A Products of Vectors Important Questions 45
Hence, equation of the required plane is
Inter 1st Year Maths 1A Products of Vectors Important Questions 46

Question 43.
Find the equation of the plane passing through the point A (3, -2, -1) and parallel to the vector \(\bar{b}\) = \(\bar{i}\) – 2\(\bar{j}\) + 4\(\bar{k}\) and \(\bar{c}\) = 3\(\bar{i}\) + 2\(\bar{j}\) – 5\(\bar{k}\).
Solution:
The required plane passes through A = (3, -2, -1) and is parallel to the vectors
\(\bar{b}\) = \(\bar{i}\) – 2\(\bar{j}\) + 4\(\bar{k}\) and \(\bar{c}\) = 3\(\bar{i}\) + 2\(\bar{j}\) – 5\(\bar{k}\). Hence if P is a point in the plane, then AP, b and C are coplanar and [AP b c] = 0.
Inter 1st Year Maths 1A Products of Vectors Important Questions 50
i.e., 2x + 17y + 8z + 36 = 0 is the equation of the required plane.

Question 44.
Find the vector equation of the plane passing through the intersection of the planes r. (i + j + k) = 6 and r. (2i + 3j+ 4k) = -5 and the point (1, 1, 1).
Solution:
Inter 1st Year Maths 1A Products of Vectors Important Questions 48
Substituting this value of λ in equation (1), we get
Inter 1st Year Maths 1A Products of Vectors Important Questions 49

Question 45.
Find the distance of a point (2, 5, -3) from the plane r. (6i – 3j + 2k) = 4.
Solution:
Here a = 2i + 5j – 3k, N = 6i – 3j + 2k and d = 4.
∴ The distance of the point (2, 5, -3) from the given plane is
Inter 1st Year Maths 1A Products of Vectors Important Questions 51

Question 46.
Find the angle between the line \(\frac{x+1}{2}\) = \(\frac{y}{3}\) = \(\frac{z-3}{6}\) and the plane 10x + 2y – 11z = 3.
Solution:
Let φ be the angle between the given line and the normal to the plane.
Converting the given equations into vector form, we have
r = (-i + 3k) + λ(2i + 3j + 6k)
and r. (10i + 2j – 11k) = 3
Here b = 2i + 3j + 6k and n = 10i + 2j – 11k
Inter 1st Year Maths 1A Products of Vectors Important Questions 52

Inter 1st Year Maths 1A Products of Vectors Important Questions

Question 47.
For any four vectors a, b, c and d, (a × b) × (c × d) = [a c d]b – [b c d]a and (a × b) × (c × d) = [a b d]c – [a b c]d.
Solution:
Let m = c × d
∴ (a × b) × (c × d) = (a × b) × m
= (a.m)b – (b.m)a
= (a. (c × d))b – (b. (c × d))a
= [a c d]b – [b c d]a.
Again Let a × b = n.
Then(a × b) × (c × d) = n × (c × d)
= (n .d)c – (n.c)d
= ((a × b). d) c – ((a × b). c)d
= [a b d]c – [a b c]d.

Question 48.
Find the shortest distance between the skew line r = (6i + 2j + 2k) + t(i – 2j + 2k) and r = (-4i – k) + s (3i – 2j – 2k).
Solution:
The first line passes through the point
A(6, 2, 2) and is parallel to the vector b = i – 2j + 2k. Second line passes through the point C(-4, 0, -1) and is parallel to the vector d = 3i – 2j – 2k
Inter 1st Year Maths 1A Products of Vectors Important Questions 53

Question 49.
The angle ¡n a semi circle is a right angle. (May ’08)
Solution:
Let O be the centre and AOB be the diameter of the given šemi circle. Let P be a point on the semi circle. With reference to O as the origin, if the position vectors of A and P are taken as \(\overline{\mathrm{a}}\) and \(\overline{\mathrm{p}}\),then
Inter 1st Year Maths 1A Products of Vectors Important Questions 54

Question 50.
The altitude of a triangle are concurrent.
ii) The perpendicular bisectors of the sides of a triangle are concurrent.
Solution:
i) In the given triangle ABC, let the altitudes from A, B drawn to BC. CA respectively intersect them at D, E. Assume that AD and BE intersect at O, join CO and produce it to meet AB at F. With reference to O, let the position vectors of A, B and C be a, b and c respectively.
Inter 1st Year Maths 1A Products of Vectors Important Questions 55
From Fig. we have
Inter 1st Year Maths 1A Products of Vectors Important Questions 56

ii) In the given ABC, let the mid-points of BC, CA and AB be D, E and F respectively. Let the perpendicular bisectors drawn to BC and CA at D and E meet at O. Join OF. With respect to O, let the position vectors of A, B and C be \(\bar{a}\), \(\bar{b}\) and \(\bar{c}\) respectively.
Inter 1st Year Maths 1A Products of Vectors Important Questions 57
Inter 1st Year Maths 1A Products of Vectors Important Questions 58
On adding equations (1) and (2), we obtain
Inter 1st Year Maths 1A Products of Vectors Important Questions 59

Question 51.
Show that the vector area of the quadrilateral ABCD having diagonals \(\overline{\mathbf{A C}}\), \(\overline{\mathbf{B D}}\) is \(\frac{1}{2}(\overline{A C} \times \overline{B D})\)
Solution:
If the point of intersection of the diagonals AC, BD of the given quadrilateral ABCD is assumed as Q, the vector area of the quadrilateral ABCD.
Inter 1st Year Maths 1A Products of Vectors Important Questions 60
= Sum of the vector areas of ΔQAB, ΔQBC, ΔQCD and ΔQDA.
Inter 1st Year Maths 1A Products of Vectors Important Questions 61

Inter 1st Year Maths 1A Products of Vectors Important Questions

Question 52.
For any vectors a, b and c (a × b) × c = (a.c) b – (b.c) a (AP) (Mar. 15 May ‘06; June ’04)
Solution:
Equation (1) is evidently true if a and b are parallel.
Now suppose that a and b are non-parallel.
Let O denote the origin. Choose points A and B such that \(\overline{\mathrm{OA}}\) = \(\overline{\mathbf{a}}\) and \(\overline{\mathrm{OB}}\) = \(\overline{\mathbf{b}}\). Since \(\overline{\mathbf{a}}\) and \(\overline{\mathbf{b}}\) are non-parallel, the points O, A and B are non-collinear. Hence they determine a plane.
Let i denote the unit vector along \(\overline{\mathrm{OA}}\).
Let j be a unit vector in the OAB plane perpendicular to i. Let k = i × j. Then
Inter 1st Year Maths 1A Products of Vectors Important Questions 62

Question 53.
For any four vectors a, b, c and d(a × b). (c × d) = \(\left|\begin{array}{cc}
a \cdot c & a \cdot d \\
b . c & b . d
\end{array}\right|\) and in particular (a × b)2 = a2b2 – (a. b)2.
Solution:
Inter 1st Year Maths 1A Products of Vectors Important Questions 63

Question 54.
In the above fou rmula if \(\overline{\mathbf{a}}\), \(\overline{\mathbf{b}}\), \(\overline{\mathbf{c}}\) and \(\overline{\mathbf{d}}\)(\(\overline{\mathbf{a}}\) × \(\overline{\mathbf{b}}\)). (\(\overline{\mathbf{c}}\) × \(\overline{\mathbf{d}}\)) = (\(\overline{\mathbf{a}}\) . \(\overline{\mathbf{c}}\))(\(\overline{\mathbf{b}}\) . \(\overline{\mathbf{d}}\)) – (\(\overline{\mathbf{a}}\) . \(\overline{\mathbf{d}}\))(\(\overline{\mathbf{b}}\) . \(\overline{\mathbf{c}}\)) = \(\left|\begin{array}{ll}
\overline{\mathbf{a}} \cdot \overline{\mathbf{c}} & \overline{\mathbf{a}} \cdot \overline{\mathbf{d}} \\
\overline{\mathbf{b}} \cdot \overline{\mathbf{c}} & \bar{b} \cdot \overline{\mathbf{d}}
\end{array}\right|\).
Solution:
Inter 1st Year Maths 1A Products of Vectors Important Questions 64
Since this is a scalar, this is also called the scalar product of four vectors.